SlideShare una empresa de Scribd logo
1 de 117
Descargar para leer sin conexión
Universidad La Salle.
Facultad Mexicana de Medicina.
Curso de Extensión Universitaria para la Preparación del Examen Nacional para
Aspirantes a Residencias Médicas.
Examen Final 2016
1.- Se trata de masculino de 9 meses, ingresa al servicio de urgencias por presentar en 24 hrs.,
7 evacuaciones diarreicas, sin sangre, fiebre no cuantificada y vómito 4 veces. La madre refiere
que en las últimas horas el menor está hiporéxico pero bebe con avidez. E.F. : Inquieto,
fontanela anterior deprimida, ojos hundidos, llanto sin lágrimas, lienzo húmedo negativo, pulsos
normales, llenado capilar de 3 segundos.
El agente causal más probable en éste paciente es:
a) Parasito.
b) Virus.
c) Bacteria.
d) Toxina.
Causada por un agente infeccioso
Es el tipo más frecuente. La etiología del 70% de los casos de diarrea infecciosa es la viral.4
Todos los años, las causas principales de gastroenteritis infantil son los Rotavirus, causantes de
600.000-800.000 muertes en todo el mundo. El virus infecta los enterocitos del intestino,
disminuye la actividad de las enzimas que digieren los azúcares, y disminuye la reabsorción del
ión Na+ y del agua en el intestino. Además producen activación del sistema nervioso entérico y
la secreción de iones Cl-. Todo ello produce un exceso de fluidos en la luz intestinal, que tiene
como consecuencia una diarrea acuosa. Otros agentes etiológicos virales son los Norovirus, que
ejercen una acción directa sobre la actividad de las enzimas de los enterocitos.
Gastroenteritis Virales
Agentes etiológicos
Reovirus (Rotavirus)
Calicivirus (Agente Norwalk)
Adenovirus (entéricos)
Astrovirus
Otros: Torovirus, Picornavirus, Picornabirnavirus
Causas de Diarrea Infantil Aguda9
Acuosa
Con sangre
(Disentérica)
Secretora Osmótica Invasiva No invasiva
Rotavirus
Vibrio cholerae
ECET
Vibrios no cólera
Shigella
Virus
G. lamblia
Cryptosporidium
Laxantes
Desnutrición
Shigella
ECEI
Salmonella no tifoidea
Campylobacter jejun
Yersinia
E. coli O157:H7
ECEP
Clostridium difficile
2.- Se trata de femenino de 6 años ingresa al servicio de urgencias, refiere la madre que la
menor ha presentado tos, ardor retroesternal y silbido del pecho. . Tiene tos irritativa, no
productiva, hipertermia no cuantificada. Antecedentes: Originaria de Veracruz, nivel
socioeconómico bajo, habita en un rancho. Malos hábitos de higiene. Exploración física: febril 39
°c, con dificultad para respirar, aleteo nasal, tiraje intercostal y subcostal. Laboratorio: eosinofilia
marcada. Radiografía de tórax muestra infiltrados redondos u ovalados de unos milímetros hasta
varios cm. de diámetro, bilateral.
El tratamiento de elección en éste síndrome es:
a) TMP/SMZ 800mg / 160MG c/12 hrs
b) Amikacina 30 mg. por kg. de peso para 24 hrs.
c) Albendazol 400 mg dosis unica.
d) Clindamicina 300 mg v.0. cada 8 hrs
Tratamiento. Mebendazol (100 mg/12 h/3 dias o 500 mg a dosis unica). Alternativas: pamoato de
pyrantel, 10 mg/kg (maximo 1 g), dosis unica; albendazol, 400 mg, dosis unica
(ninos entre 2-5 anos: 200 mg); piperazina, 75 mg/kg (maximo 3,5 g en adultos o 2,5 g en
Niños entre 2-12 años). La ivermectina tambien es eficaz. (200 μg/kg dosis unica).
Parasitosis intestinales
J. Gascón Brustenga y J. Muñoz Gutiérrez
3.- Se trata de recién nacido pretermino de 30 semanas de gestación, peso de 1200 gr. Cursó
con enfermedad de membranas hialinas por lo que se manejo con una dosis de surfactante y
manejo con ventilación convencional por 3 días. Posteriormente inició con alimentación enteral
por sonda orogástrica con leche humana, sin tolerancia a la misma (residuo gástrico y distensión
abdominal), sangre oculta en heces e ictericia. El diagnóstico más probable es:
a) Alergia a las proteínas de la leche
b) Enterocolitis necrotizante
c) Atresia intestinal
d) Gastroenteritis infecciosa
La enterocolitis necrotizante (ECN) es una enfermedad grave que afecta a recién nacidos, en
especial prematuros, con una incidencia y morbimortalidad elevados. Constituye la urgencia
gastrointestinal más frecuente en las UCI neonatales.
Se presenta como un síndrome gastrointestinal y sistémico que comprende síntomas variados y
variables, como distensión e hipersensibilidad abdominal, sangre en heces, intolerancia a la
alimentación, apnea, letargia, y en casos avanzados acidosis, sepsis, CID y shock.
El síndrome clínico ha sido clasificado en estadios por Bell y col. (1978) y modificado por
Walsh y Klegman (1986) para incluir hallazgos sistémicos, intestinales y radiológicos.
 A. Estadio I : sospecha de enterocolitis necrotizante
 Los hallazgos sistémicos son inespecíficos.
 Los hallazgos intestinales incluye el residuo gástrico y heces guayaco –positivas.
 Los hallazgos radiológicos son normales e inespecíficos.
 B. Estadio II A: enterocolitis necrotizante leve
 Los hallazgos sistémicos son similares al estadio I.
 Los hallazgos intestinales incluyen distensión abdominal prominente con
hipersensibilidad a la palpación o sin ella, ruidos hidroaéreos ausentes, sangre macroscópica
en materia fecal.
 Los hallazgos radiológicos, íleo con asas dilatadas con áreas focales de neumatosis
intestinal.
 C. Estadio II B: enterocolitis necrotizante moderada
 Los hallazgos sistémicos incluyen acidosis leve y trombocitopenia
 Los hallazgos intestinales incluyen edema de la pared abdominal e hipersensibilidad a la
palpación con una masa palpable o sin ella.
 Los hallazgos radiológicos incluyen neumatosis extensa y ascitis temprana.
 Puede haber gas en la vena porta intrahepática.
 D. Estadio IIIA: enterocolitis necrotizante avanzada:
 Los hallazgos sistémicos incluyen acidosis respiratoria y metabólica, ventilación asistida
por apnea, hipotensión arterial, oliguria, neutropenia y coagulación intravascular diseminada.
 Los hallazgos intestinales incluyen edema que disemina, eritema e induración del
abdomen.
 Los hallazgos radiológicos incluyen ascitis prominente y asa centinela persistente sin
perforación.
 E. Estadio IIIB: enterocolitis necrotizante avanzada:
 Los hallazgos sistémicos revelan signos vitales e índices de laboratorio en deterioro,
síndrome de shock y desequilibrio electrolítico.
 Los hallazgos intestinales y radiológicos muestran evidencias de perforación.
Hallazgos radiológicos en la radiografía de abdomen
 Distensión abdominal generalizada
 Íleo paralítico
 Neumatosis intestinal quistoide (patognomónico)
La neumatosis intestinal quistoide se la puede visualizar de varias maneras, cúmulos de gas
lineales, curvilíneos, esponjosos y espumosos.
Este último debe distinguirse de materia fecal o meconio mezclado con aire.
Cualquier lactante con sospecha de enterocolitis necrotizante en el que se encuentren
radiográficamente colecciones de aire lineales, curvilíneas esponjosas o espumosas debe
considerarse que tiene neumatosis intestinal quistoide hasta que se demuestre lo
contrario.
La neumatosis intestinal quistoide suele verse con mayor frecuencia en el colon, pero puede
ocurrir desde estomago hasta recto.
1. Hartmann G. E., Drugas G. T., Shochat S. J. Post-necrotizing enterocolitis strictures
presenting with sepsis of perforation: risk of clinical observation. J. Pediatr. Surg. 1988;
23: 562-6.
2. Kosloske A. M., Burstein J., Bartow S. A. Intestinal obstruction due to colonic stricture
following neonatal necrotizing enterocolitis. Ann Surg. 1980 Aug;192 (2): 202-7.
3. Schwartz M. Z., Hayden C. K., Richardson C. J., Tyson K. R., Lobe T. E. A prospective
evaluation of intestinal stenosis following necrotizing enterocolitis. J. Pediatr. Surg. 1982
Dec; 17 (6): 764-70.
4. Bell M. J., Ternberg J. L., Askin F. B. Intestinal stricture in necroting enterocolitis. J.
Pediatr. Surg. 1976; 11: 319-27.
5. Pokorny W. J., Harr V. L., McGill, C. W., et al; Intestinal stenosis resulting from
necrotizing enterocolitis. Am J. Surg 1981 42: 721-724.
6. Schimpl G., Hollwarth M. E., Fotter R., Becker H. Late intestinal strictures following
successful treatment of necrotizing enterocolitis. Acta Paediatr. Suppl. 1994; 396: 80-3.
7. Bütter A., Flageole H., Laberge J. M. The Changing face of Surgical Indication for
Necrotizing Enterocolitis J. Pediatr. Surg. 2002; 37: 469- 499.
Gobet R. , Sacher P. , Schwobel M. G. Surgical procedures in colonic strictures after necrotizing
enterocolitis. Acta Paediatr. Suppl. 1994;396:77-9.
4.- Ingresa al servicio de urgencias paciente masculino de 2 años que presenta letargo después
de haber estado jugando sin vigilancia. El niño hasta entonces estaba sano, con un desarrollo
normal. En la historia familiar aparece como dato importante que uno de sus hermanos tiene
epilepsia y toma fenobarbital. La exploración revela un niño letárgico sin fiebre que abre
brevemente los ojos ante estímulos dolorosos. No hay signos de traumatismo ni anomalías
focales. Las pupilas son pequeñas pero reactivas. La etiología más probable relacionada al
letargo del niño es:
a) Convulsiones no observadas.
b) Intoxicación.
c) Hemorragia intracraneal.
d) Traumatismo craneoencefálico no observado. .
La causa más común de inicio agudo de somnolencia en un niño que empieza a caminar, es la
intoxicación. La presencia de agentes tóxicos potenciales en la casa es importante en la historia
clínica. La miosis también sugiere intoxicación y la falta de signos de traumatismo o anomalías
focales hace que una hemorragia sea poco probable.
LECTURA RECOMENDADA
PROGRAMA DE ACTUALIZACION CONTINUA EN PEDIATRIA
INTOXICACIONES EN PEDIATRÍA
Dr. Miguel Angel Montoya Cabrera
Jefe del Departamento de Admisión Continua y Toxicología, Hospital de Pediatría, Centro
Médico Nacional Siglo XXI, Instituto Mexicano del Seguro Social. Miembro de la Academia
Nacional de Medicina, de la Academia Mexicana de Cirugía y de la Academia Mexicana de
Pediatría
5.- Femenino de 8 meses refiere la madre que ha presentado cuadros frecuentes de vómitos,
movimientos incordinados y crisis convulsivas de difícil control, se le ha puesto el pelo más claro
que al resto de sus hermanos, piel seborreica, microcefalia, maxilar prominente, dientes
espaciados y retraso en el crecimiento. El diagnóstico más probable de éste paciente es:
a) Citrulinemia
b) Homocistinuria
c) Fenilcetonuria
d) Enfermedad de Jarabe de Maple.
La fenilcetonuria, es un tipo de hiperfenilalaninemia, también conocida como PKU, es una
alteración del metabolismo en el que el organismo no puede metabolizar el aminoácido tirosina a
partir de fenilalanina en el hígado. Esta enfermedad es genética y es provocada por la carencia
de enzima fenilalanina hidroxilasa .
La fenilcetonuria tiene como rasgo principal la herencia genética autosómica recesiva, es decir,
los padres son portadores de los genes defectuosos y al ser traspasados de ambos progenitores,
la enfermedad se expresa en los descendientes.
La causa de la enfermedad es la carencia de la enzima fenilalanina hidroxilasa (FAOH) o de la
dihidropterina reductasa (DHPR) (también llamada tirosina hidroxilasa). Ambas enzimas son
responsables de la hidroxilación del aminoácido fenilalanina en la reacción que produce tirosina.
Por ello, el defecto o falta de alguna de ellas determina un incremento de la concentración
sanguínea de fenilalanina al impedirse su transformación en tirosina. También se aumenta la
transaminación de la fenilalanina como vía metabólica alternativa, y asimismo se acumulan los
metabolitos fenilpiruvato, fenilactato y fenilacetato. El defecto en la síntesis de FAOH se debe a
una anomalía génica localizada en el cromosoma 12, y el de la DHPR en el cromosoma 4.
Existen también formas de la enfermedad con déficits parciales.
El fenilpiruvato es un neurotori que afecta gravemente al cerebro durante el crecimiento y el
desarrollo. Los efectos de su acumulación causan oligofrenia fenilpirúvica, caracterizada por un
cociente intelectual inferior a 20. Los primeros meses de vida, los niños que padecen esta
enfermedad parecen estar sanos. Entre los tres y los seis meses pierden el interés por el
entorno, y al año se evidencia un retraso importante en su desarrollo. Los síntomas suelen ser
retraso psicomotor, cuadros psicóticos de tipo autista, convulsiones, síndrome de West,
convulsiones generalizadas y un eczema facial muy rebelde. Por lo general su desarrollo físico
es bueno, tienden a tener el cabello más claro que sus hermanos, piel clara, y presentan un olor
característico a paja mojada.
Cuadro clínico
La enfermedad se manifiesta, por primera vez, algunas semanas después del nacimiento,
iniciándose con una elevación en el plasma de la fenilalanina hasta un nivel 30 veces superior al
normal y por la excreción de ácido fenilpirúvico por la orina. Transcurridos 6 meses se hace
patente el retraso del desarrollo mental. La mayor parte de los pacientes son deficientes graves o
profundos y en ocasiones se alcanza la deficiencia media.
El portador de esta anomalía, que nace tras un embarazo normal y sin complicaciones, se
desarrolla durante los primeros meses casi siempre sin mostrar anormalidad ninguna. Sin
embargo Partington encontró, casi en la mitad de los lactantes, la existencia de vómitos en los
primeros meses de vida y en un tercio de ellos una irritabilidad inacostumbrada. En una
proporción similar de casos, a los padres ya les había llamado la atención un desagradable olor
del cuerpo del niño. Una parte de ellos mostró dermatosis eczematiformes durante el primer
trimestre, 7 de 36 ya había tenido en el primer año de vida ataques convulsivos. A los 9 meses
llama la atención el retraso en el desarrollo psicomotor.
Datos físicos
El desarrollo corporal cursa casi con normalidad. No obstante puede comprobarse cierta
tendencia al enanismo, aunque también se han descrito casos con tallas superior a la frecuente.
La dentición suele retrasarse hasta después del undécimo mes.
La gran mayoría de los enfermos muestran una piel clara, ojos azules, y color claro del pelo.
Alrededor del 10% poseen cabellos oscuros. La pobreza de pigmentos llama más la atención en
los pueblos de cabellos oscuros. La piel de los portadores además de ser clara es muy suave
aterciopelada y muy sensible. En algunos enfermos se han observado eflorescencias papulosas
en las caras de extensión de las extremidades y en la faz. En ciertos pacientes se puede
encontrar también una tendencia a la acrocianosis.
Datos conductuales
Características clínicas raras
 Cifosis.
 Pies planos.
 Espina bifida.
 Sindactilia en los dedos de los pies.
 Bloqueo cardiaco intraventricular.
 Hipogenitalismo.
 Dermografismo.
 Sensibilidad a la luz.
 Hipersegmentación de las células neutrófilas de la sangre.
 Disminución de la tolerancia a la galactosa.
 Metabolismo basal ligeramente elevado.
Las etapas del desarrollo habitual, la edad en la que el niño se sienta y habla, a veces, se
alcanzan a la edad normal, pero, de ordinario, se retrasa. En la edad límite en que debe
esperarse que el niño normalmente realice estos actos, el 35% no puede andar y el 63% no
puede hablar.
Estos niños, en general, tienen un peso y talla promedio por debajo del correspondiente a su
edad. En la mitad de los casos tiene microcefalia y prominencia del maxilar.
Sus movimientos son lentos y patosos y a menudo suelen adoptar la posición de sastre. Las
anomalías del tono muscular que contribuyen a estos cambios son de origen neurológico.2 de
cada 3 pacientes tienen hiperreflexia tediciosa e hipercinesia sobreañadida estos últimos son
voluntarios y muy variados
Referencias
1. Scriver CR, Kaufman S. 2001. Hyperphenylalaninemia: phenylalanine hydroxylase deficiency.
In Scriver CR,
Beaudet AL, Valle D, Sly WS (eds). The Metabolic and Molecular bases of Inherited Disease.
McGraw-Hill., pp
1667-1724.
2. Martínez-Pardo M, Marchante C. et al: 1998. ”Protocolo de diagnóstico, tratamiento y
seguimiento de las hiperfenilalaninemias”. An. Esp. Pediatría suplemento 114: 3-8.
10
3. Belanger-Quintana A, Morais A, Mena MA, Martínez- Pardo M. 2004. Niveles de
neurotransmisores dopaminérgicos y serotoninérgicos en líquido cefalorraquídeo en niños. An
Esp Pediatr 60:82.
6.- Femenino de 39 años de edad la cual inicia con un cuadro de exoftalmos axial de varias
semanas de evolución, con predominio en su ojo derecho. En la exploración se aprecia una
conjuntiva con síntomas discretos de hiperemia y edema, una queratitis de carácter punteado
en tercio inferior corneal y se sospecha una retracciónpalpebral al observar cómo el borde del
párpado superior se encuentra por encima del limbo, permitiéndonos visualizar la esclerótica.
La paciente no refiere diminución de visión ni alteraciones tipo visión doble y toma presión
intraocular con parámetros dentro de los normales. De los siguientes el diagnóstico más
probable es:
a) Tumor intraorbitario.
b) Enfermedad de Graves-Basedow.
c) Queratoconjuntivitis epidémica.
d) Tumor intraocular.
OFTALMOPATÍA TIROIDEA.
Es la causa más frecuente de exoftalmos tanto bilateral como unilateral en adultos. La forma
típica aparece en pacientes con enfermedad de Graves-Basedow, en los que podemos
encontrar exoftalmos y síndrome palpebro-retráctil. Este síndrome ocular puede aparecer en
pacientes eutiroideos o hipotiroideos, pudiendo constituir el signo más precoz de una
tirotoxicosis incipiente.
CLÍNICA.
Se distinguen dos formas clínicas:
a) Tirotóxica (hipersensibilidad a las catecolaminas y habitualmente hay hipertiroidismo):
exoftalmos moderado depresible.
Hay edema del contenido orbitario, pero no fibrosis ni oftalmoparesia.
b) Maligna (inflamación orbitaria autoinmune y puede haber normo o hipotirodismo): exoftalmos
irreductible severo con oftalmopejia progresiva y queratitis por exposición. Puede afectar al
nervio óptico por compresión y producir pérdida visual. Hay fibrosis de la grasa y vientres
musculares.
Manifestaciones oculares asociadas: retracción palpebral bilateral que permite ver la esclera
por encima del limbo, disminución de la motilidad palpebral, alteración de la pigmentación
palpebral, hiperemia conjuntival.
DIAGNÓSTICO.
Por los signos clínicos descritos y exploraciones complementarias, como la exoftalmometría
(medida de la protrusión ocular), la radiología (aumento de densidad de los tejidos blandos), el
engrosamiento del vientre de algunos músculos extraoculares (apreciados en la TC, la RM y la
ecografía orbitaria) y la analítica sistémica
AACE Thyroid Task Force. American Association of Clinical Endocrinologists medical guidelines
for clinical practice fo rthe evaluation and treatment of hyperthyroidism and hypothyroidism.
Endocr Pract. 2002;8(6).
Davies TF, Larsen PR. Thyrotoxicosis. In: Kronenberg HM, Melmed S, Polonsky KS, Larsen
PR, eds. Williams Textbook of Endocrinology. 11th ed. Philadelphia, Pa: Saunders Elsevier;
2008:chap 11.
7.- Masculino de 5 años es llevado al servicio de urgencias por dolor súbito hiperemia, y
aumento de volumen en región interna de órbita derecha. Exploración Física: Se palpa masa
bien delimitada dolorosa en el borde interno de la órbita.
La medida terapéutica inicial en este paciente es:
a) Sólo drenaje.
b) Dicloxacilina y drenaje.
c) Tobramicina tópico y drenaje.
d) Extirpación de saco lagrimal
El tratamiento debe iniciarse antes de que se identifique el microorganismo causal. Tan
pronto se obtienen cultivos de nariz, conjuntivas y sangre deben administrarse antibióticos.
La terapéutica antibiótica inicial debe cubrir estafilococos, H influenzae y anaerobios. La
mayor parte de los casos responde a los antibióticos. Aquellos en los cuales esto no es así,
pueden requerir drenaje.
Riordan P,Witcher j, Oftalmología general de Vaughan y Asbury, 17ª Ediciión, Pág. 262
8.- Paciente diagnosticado con esclerosis múltiple (considerada como un proceso inflamatorio
y desmielinizante del SNC). En el diagnóstico de esta enfermedad es muy útil la presencia de:
a) Más de 100 liníocitos por microlitro en el líquido cefalorraquídeo.
b) Más de 50 polimorfonucleares por microlitro en el líquido cefalorraquídeo.
c) Elevaciones del ácido úrico en plasma.
d) Bandas oligoclonales en el líquido cefalorraquídeo.
La esclerosis múltiple (EM) es una enfermedad desmielinizante del sistema tema nervioso
central, que habitualmente se caracteriza por ataques recurrentes de disfunción neurológica
focal y multifocal.
Los síntomas y signos clásicos de EM dependen de la localización del foco de desmielinización
y pueden incluir una variedad de disfunciones que no son específicas para la EM, tales como
alteración de la visión, ataxia y temblor intencional, debilidad o parálisis de una o más
extremidades, espasticidad y problemas vesicales. Hay,sin embargo, tres anormalidades,
altamente sugestivas de EM:
 Neuritis óptica, la cual es el síntoma inicial en cerca del 25% de los pacientes
 Oftalmoplejia internuclear que se asocia con nistagmus monoocular
 Signo de Lhermitte, que es una sensación "eléctrica" por detrás del cuello y de la
espalda que llega hasta las piernas (al flexionar el cuello).
En fibras nerviosas normales mielinizadas, la conducción ocurre de manera saltatoria; las
corrientes de acción se confinan a las secciones no mielinizadas del axón saltando de un
nódulo de Ranvier al siguiente. Este tipo de conducción es mucho más eficiente desde el punto
de vista energético que la transmisión a través de la superficie del axón entero, aumentando
así la velocidad de conducción con una pérdida mínima de energía.
La desmielinización de una porción de la fibra nerviosa normalmente mielinizada, puede
conducir a: bloqueo de la conducción en el sitio de la pérdida de mielina, disminución de la
velocidad de conducción nerviosa a través de la fibra alterada y fatiga subjetiva aumentada o
un mayor consumo de energía.
La disfunción neurológica observada en la EM es una reflexión de la alteración de la
conducción a través de segmentos parcial o completamente desmielinizados o de una fibra
nerviosa mielínica. Además, el hecho de que el tiempo de conducción a través de los
segmentos desmielinizados disminuye con aumento de la temperatura puede explicar por qué
los síntomas clínicos de EM se empeoran al aumentar la temperatura corporal.
Diagnóstico
El diagnóstico de EM se basa fundamentalmente en dos parámetros. Por un lado los hallazgos
en los estudios de imagen por Resonancia Magnética en los que se presentan (en las
imágenes dependientes de T2) áreas hiperintensas de localización fundamentalmente
periventricular en la substancia blanca subcortical en múltiples localizaciones así como también
en la médula espinal; este estudio permite visualizar no solamente la localización de las áreas
desmielinizantes sino también en forma seriada pueden realizarse evaluaciones para valorar la
evolución clínica y subclínica de la enfermedad. Los otros estudios básicos para el diagnóstico
son la determinación de bandas oligoclonales, la determinación de la proteína básica de la
mielina y la medición de las inmunoglobulinas en el líquido cefalorraquídeo. Estos dos factores,
en conjunto con la información clínica, permiten establecer con ciertas bases de seguridad el
diagnóstico de esta enfermedad.
Referencias bibliográficas:
1. Noseworthy JH, Lucchinetti C, Rodríguez M, Weinshenker BJ. Multiple sclerosis. N.
England J Med. 2000;343:938-52.
2. Omari KM, John GR, Sealfon SC, Raine CS. CXC chemokine receptors on human
oligodendrocytes: implications for multiple sclerosis. Brain 2005;128:1003-15.
3. Mi S, Miller RH, Lee X, et al. LINGO-1 negatively regulates myelination by
oligodendrocytes. Nat Neurosci. 2005;8:745-51.
4. John GR, Shankar SL, Shafit-Zagardo B, et al . Multiple sclerosis: re-expression of a
developmental pathway that restricts remyelination. Nat Med. 2002;8:1115-21.
5. Lucchinetti C. The spectrum of idiopathic inflammatory demyelinanting disease. In:
American Academy of Neurology. Syllabi on CD ROM. 2000.
6. Hartung HP, Grossman RI. ADEM: distinct disease or part of the MS spectrum?
Neurology 2001; 56:1257-60.
7. Capello E, Voskuhl RR, and McFarland HF, Raine CS. Multiple sclerosis: re-expression
of a developmental gene in chronic lesions correlates with remyelination. Ann Neurol.
1997; 41:797-805.
8. Atlanta. Georgia. AAN. 115 Anual Meeting. Natural Course of Multiple Sclerosis
Redefined: National Inst of Neurological Disorder and Stroke, 16, 1990.
9. Hemmer B, Archelos JJ, Hartung HP. New concepts in the inmunopathogenesis of MS.
Nat Rev Neurosci. 2002;3:291-301.
Kurtzke JF. Geography in multiple sclerosis. J Neurol. 1977;215:1-26.
9.- Una mujer de 66 años presenta deterioro al caminar. En la exploración se encuentra marcha
ligeramente espástica, mala posición y sensación de vibración en los dedos de los pies, reflejos
de estiramiento muscular en las rodillas +++ y reflejos aquíleos ausentes. De los siguientes
diagnósticos el más probable ES:
a) Hipovitaminosis B12
b) Esclerosis múltiple
c) Hidrocefalia con presión normal (NPH)
d) Infección con virus linfotrópico humano de células T tipo I (HTLV-I)
La hipovitaminosis B12 causa degeneración combinada de sistemas. Los pacientes presentan
alteración de la marcha caracterizada por espasticidad y disminución de la sensación vibratoria
y la posición. Puede haber neuropatía leve que produce depresión de los reflejos aquíleos.
Como es una causa tratable de marcha anormal, es importante identificar la deficiencia de vitan
B12. La esclerosis múltiple (MS) no se manifiesta en este grupo de edad. La ausencia de
reflejos aquíleos es un dato común de MS; más bien, la hiperactividad de los reflejos concuerda
con los datos de neurona motora superior encontrados a menudo. La hidrocefalia de presión
normal (NPH) es una causa de deterioro de la marcha. Sin embargo, hay donación cognitiva y
los pacientes pueden tener incontinencia urinaria. La alteración de la vibración y la sensación
de posición, así como la ausencia de reflejos aquíleos, no son característicos de NPH. La
infección virus linfotrópico de células T humano tipo I (HTLV-I) es causa de mielopatía y debe
sospecharse en pacientes que han recibido transfusiones de sangre, usuarios de drogas
intravenosas o personas que hayan habitado en áreas endémicas. La adrenomieloneuropatía
es un trastorno recesivo ligado a X que guarda relación con adrenoleucodistrofia, la cual
típicamente se presenta en varones jóvenes. El trastorno origina la acumulación de ácidos
grasos de cadena muy larga debido a una imposibilidad para catabolizar estos lípidos. Las
mujeres portadoras manifiestan una paraparesia espástica leve, pero no la gama de datos que
se encuentran en esta paciente.
BIBLIOGRAFíA:
Braunwald, Isselbachre, Petersdorf, Wilson, Martin, Fauci
“HARRISON: PRINCIPIOS DE MEDICINA INTERNA”
Undécima edición D.S.Mc Laren, M.Frigg
10.- Paciente de 79 años que consulta por una mácula pigmentada heterocroma, de contorno
irregular, de unos 2 x 3 cm de diámetro, localizada en mejilla derecha, que ha experimentado
un crecimiento muy lento en los últimos años. El diagnóstico más probable de este paciente es:
a) Melanoma lentiginoso acral.
b) Carcinoma Basocelular pigmentado.
c) Eritema fijo pigmentario.
d) Lentigo maligno.
El lentigo maligno puede definirse como una mácula pigmentada extensa localizada en una
zona de la piel expuesta al daño actínico, habitualmente desarrollada en ancianos y que
consiste en una proliferación de melanocitos atípicos situados sobre una epidermis atrófica.
La importancia del lentigo maligno reside en que puede ser el sustrato de un melanoma
maligno.
El lentigo maligno también ha sido denominado, lentigo de Hutchinson, lentigo senil, melanosis
precancerosa circunscrita, lentigo “malin des viellards ”, melanosis premaligna, melanosis
circunscrita preblastomatosa y melanocitoma no nevoide precanceroso.
El lentigo maligno puede ser considerado como la única variedad de displasia melanocítica
intraepidérmica capaz de evolucionar a un melanoma invasivo.
El porcentaje de melanomas que se suponen derivados de un lentigo maligno es pequeño, 2.5
a 5%. El lentigo maligno es fácilmente reconocible y habitualmente tratable en las fases más
precoces de su desarrollo.
El lentigo maligno fue descrito en 1892 por Hutchinson. Dubreuilh remarcó su naturaleza
precancerosa en 1894. Mishima determinó que el precursor histogénico del lentigo maligno es
diferente al precursor del nevus juncional descrito por Allen y Spitz. La histopatología
macroscópica y microscópica del lentigo maligno fue documentada por Whyte y Helwig (1) y
por Clark y Mihm.
Lentigo maligno 01. Dermatoscopia. Pigmentación asimétrica de las salidas foliculares (flechas
pequeñas), estructura romboidal (flecha grande), oclusión de las salidas foliculares (círculo) y
puntos azul-gris (asterisco) Lentigo Maligno
Cox NH, Aitchison TC, Sirel JM, Mackie RM. Comparison between lentigo maligna melanoma
and other hystogenetic types of malignant melanoma of the head and neck. Br J Cancer 1996;
73:940-4.
Cohen LM, McCall MW, Zax RH. Mohs micrographic surgery for lentigo maligna and lentigo
maligna melanoma. Dermatol Surg 1998; 24:673-77
11.- Femenino de 26 años su padecimiento actual inició hace 4 años, cuando sus ciclos
menstruales empezaron a ser irregulares. Su ritmo actual es de 40-90 x 3-4. FUR: hace 3
meses. Inició vida sexual a los 23 años y no ha podido embarazarse. En la exploración física
encontramos la piel ligeramente seca, hay salida de líquido blanquecino escaso a la expresión
del pezón izquierdo con presencia de giordano positivo derecho.
El estudio que es de mayor utilidad para aclarar el diagnóstico es:
a) prueba de embarazo
b) LH, FSH y estrógenos
c) tomografía de cráneo
d) prolactina
En toda paciente con galactorrea, trastornos menstruales, hirsutismo, disminución de la libido o
infertilidad, deben determinarse los niveles de PRL plasmática, entre el tercer y quinto día del
ciclo menstrual, si este es regular o en cualquier momento, si presenta amenorrea u
oligomenorrea.2 Si en la primera determinación se obtienen cifras elevadas debe repetirse y si
se confirma nuevamente, se establece el diagnóstico de hiperprolactinemia. Una historia clínica
com-pleta, con un interrogatorio y examen físico detallados, permitirá orientarnos hacia la
causa de la hiperprolactinemia. La primera causa que se debe descartar antes de emprender
otros estudios, es el embarazo. El uso de estrógenos, anticonceptivos orales y drogas que
aumentan la secreción de PRL debe precisarse en el interrogatorio, así como la presencia de
quemaduras u otras lesiones en tórax que se buscan en el examen físico.
Pocas mediciones hormonales tienen el significado clínico que se observa con la prolactina. La
técnica está bien estandarizada y presenta bastante confiabilidad; la muestra de sangre se
puede obtener a cualquier hora del día y debido a las variaciones fisiológicas (efecto del
ejercicio, alimentos, irritación de la pared costal) cuando se obtiene una cifra de prolactina por
arriba de lo normal, es necesario repetir el análisis.
Varios fármacos pueden producir una moderada elevación en los niveles de prolactina, sin
alcanzar los valores que se encuentran en prolactinoma; las drogas más comunes son
metoclopramida, fenotiazinas, risperidona, verapamil,metildopa, reserpina y los inhibidores de
MAO.
Exámenes: prolactina plasmática (prolactinemia), tirotropina (hormona estimulante de la
glándula tiroides o TSH), test de embarazo.
Prolactina menor de 100 ng/ml: baja sospecha de prolactinoma.
Prolactina entre 100 y 300 ng/ml: mayor posibilidad de prolactinoma.
Prolactina superior a 300 ng/ml: alta sospecha de prolactinoma. Es precisa resonancia
magnética nuclear (RNM) de hipófisis (detecta prolactinoma mayor de 3mm).
.
En el 50% de los casos de hiperprolactinemia se detecta tumor: microadenomas o
macroadenomas; la prolactina suele encontrarse entre 100 y 200ng/ml; los macroadenomas
tienen efecto de masa y causan hipopituitarismo; los microadenomas no presentan efecto de
masa y sólo producen hipogonadismo.
Referencias
1. Schlechte JA. Prolactinoma. N Engl J Med 2003;349:2035-2041.
2. Zárate A, Canales ES, Jacobs LS, Soria J, Daughaday WH. Restoration of ovarian function in
patients with the amenorrhea-galactorrhea syndrome after long-term therapy with L-Dopa. Fertil
Steril 1973;24:340.
3. Tyson JE, Carter JN, Andreassen B, Huth J, Smith B. Nursing mediated
prolactin and luteinizing hormone secretion during puerperal lactation. Fertil
Steril 1978;30:154.
4. Schlechte JA, Sherman BM, Chapler FK, VanGilder J. Long-term followup of women with
surgically treated prolactin-secreting pituitary tumors. J
Clin Endocrinol Metab 1986;62:1296-301.
5. Losa M, Mortini P, Barzaghi R, Gioia L, Giovanelli M. Surgical treatment of prolactin-secreting
pituitary adenomas: early results and long-term outcome. J Clin Endocrinol Metab
2002;87:3180-3186.
12.- Es una de las enfermedades tiroideas autoinmunes organo-específica más frecuentes. Es
más común en la mujer, posee una asociación directa con otras enfermedades autoinmunes y,
por lo general, se presenta con hipertiroidismo, bocio difuso, oftalmopatía, además, en algunos
casos coincide con mixedema pretibial:
a) Carcinoma basocelular
b) Enfermedad de Graves
c) Adenoma tóxico
d) Síndrome del eutiroideo enferm
Introducción
La enfermedad de Graves-Basedow (EGB) constituye la entidad más importante, por su
frecuencia, entre las enfermedades que producen hipertiroidismo. Se caracteriza por la
presentación de la tríada sintomática constituida por hipertiroidismo, bocio difuso y oftalmopatía
(50%). Es más frecuente en la 3ª y 4ª década de la vida y afecta con más frecuencia a mujeres
(7/1 en zonas no bociógenas y 3/1 a partir de los 45 años en zonas de bocio endémico) 1.
Etiopatogenia
Aunque no se conocen exactamente los factores que inician y mantienen la enfermedad, hay
elementos que permiten encuadrarla en el marco de la patología autoinmunitaria, como la
presencia de autoanticuerpos antitiroglobulina y antitiroperoxidasas (TPO), así como la
presencia de inmunoglobulinas antireceptor de la TSH (TRAb) que son estimuladoras de la
función y del crecimiento del tiroides.
Para el desarrollo de la enfermedad parece que es necesaria la intervención de factores
genéticos y ambientales (exceso de Yodo) 2.
Clínica
La enfermedad se caracteriza por la presencia de síntomas de hipertiroidismo (nerviosismo
99%, sudoración 91%, intolerancia al calor y palpitaciones 89%, pérdida de peso 85%,
aumento de apetito 69% y síntomas oculares 55%) y signos clínicos (aumento uniforme del
tamaño de la glándula 97%, oftalmopatía 60%, taquicardia en reposo 90%, temblor 70% y
retracción palpebral 38%) relacionados con el aumento de receptores adrenérgicos que
determinan las hormonas tiroideas 1,2.
La historia natural de la enfermedad se caracteriza por fases cíclicas de exacerbación y
remisión, de duración y presentación imprevistas; aunque, hoy en día, está artefactada por los
tratamientos utilizados. En aproximadamente el 25% de los pacientes, especialmente en
aquellos con una forma leve de enfermedad, el proceso se autolimita al año o más, regresando
espontáneamente a un estado eutiroideo 3.
En cuanto a la oftalmopatía, hay una forma no infiltrativa o pálpebro-retráctil (afecta más
frecuentemente al sexo femenino, afecta simétricamente a ambos ojos y que evoluciona
paralelamente al hipertiroidismo) y una forma infiltrativa (infiltración del tejido retroorbitario por
mucopolisacáridos y linfocitaria con protusión ocular asimétrica)1.
Otras manifestaciones menos frecuentes son la aparición de mixedema pretibial o dermopatía
infiltrativa y, la acropaquia tiroidea 1.
Referencias
1.- Foz M. Enfermedades del tiroides. En Rozman C (ed): Farreras-Rozman Medicina interna.
Editorial Doyma. Barcelona, 1992:1997.
2.- Wartofsky L. Diseases of the thyroid. En Braunwald E, Isselbacher KJ, Wilson JD, Martin JB,
Fauci AS, Kasper DL (eds): Harrison’s principles of internal medicine. Editorial McGraw-Hill.
EEUU, 1994: 1930.
3.- De Groot. Graves’ diseases and the manifestations of thyrotoxicosis. En De Groot LJ, Reed
Larsen P, Hennemann G.(eds): The thyroid and its diseases. 1996: 371.
4.- Haynes RC, Murad F. Drogas tiroideas y antitiroideas. En Goodman LS, Gilman A (eds): Las
Bases farmacológicas de la terapéutica. Editorial Panamericana. México, 1982: 1376.
5. - Feliciano DV. Everything you wanted to know about Graves’ disease. Am J Surg 1992,
(164): 404.
13.- Masculino de 42 años con antecedentes de litiasis renal cálcica bilateral recidivante.
Acude Con resultados de laboratorio los cuales demuestran una calcemia de 11.1 mg%,
fosfatemia de 2 mg, calciuria de 280 mg/24h y fosfaturia de 1600 mg/24h, con una reabsorción
tubular de fósforo (RTP) del 70%. La sospecha más probable del origen de ésta litiasis es:
a) Acidosis tubular
b) Intoxicación vitamina D
c) Hiperparatiroidismo primario
d) Sarcoidosis
Exploración y pruebas complementarias.
El examen clínico es muy pobre. Excepcionalmente se puede palpar una tumoración cervical
en el 4% de los casos. Son los síntomas lo que hacer evocar el diagnóstico y aconsejan
determinar la calcemia. Calcemia: su determinación es el elemento fundamental del diagnóstico
y se aconseja repetirla varias veces hasta comprobar su existencia, ya que puede ser
constante o intermitente por circunstancias como la deshidratación o determinados fármacos
pueden causar un aumento transitorio de la misma.
HIPERCALCEMIA
Es el aumento del calcio sérico por encima de 10,5 mg/dl. Cerca de la mitad de las
hipercalcemias son “falsas hipercalcemias”, por extracción sanguínea en condiciones no
ideales (postpandriales, torniquetes prolongados...) o por hiperalbuminemia que condiciona
“pseudohipercalcemia”, por lo que ante todo paciente con hipercalcemia debemos en primer
lugar calcular la calcemia según las proteínas totales (calciocorregido Ca2+) en base a la
siguiente fórmula:
Junto al calcio se ha determinar la albúmina pues si esta es menor de cuatro habrá que corregir
convenientemente la cifra del calcio. Como se ha expuesto, la etiología más frecuente de las
hipercalcemias es el hiperparatiroidismo primario. Los valores sumamente altos suelen
asociarse a los infrecuentes carcinomas paratiroideos, ello es debido a que el tumor produce
una proteína parecida a la hormona paratiroidea que ha sido detectada en el suero y que se
denomina proteína relacionada con la hormona paratiroidea. Se aconseja investigar esta
proteína en los pacientes con una aparición reciente de hipercalcemia y valores normales de
PTH, no obstante el hiperparatiroidismo normocalcémico es una posibilidad rara, si no existe
insuficiencia renal, ni deficiencia de vitamina D. A pesar de ello pueden darse situaciones
confusas de hiperparatiroidismo con calcemia en ayunas normal, en estos casos puede ser útil
comprobar la hipercalcemia postabsortiva, o realizar una prueba de provocación con
benzotiacidas.
La fosforemía generalmente esta disminuida. La hipofosfatemia es un dato diagnóstico menos
específico que la hipercalcemia.
En el hiperparatiroidismo es frecuente la hipercalciuria, pero menos que en otros estados de
hipercalcemia. La calciuria está elevada siendo superior a 200 mg/24 h, si bien sus valores
oscilan en función del aporte de calcio, así cuando es bajo, la calciuria se sitúa entre 50 y 150
mh/24 h; al aumentar el aporte, los valores de calciuria oscilan entre 100 y 300 mg/24h..La
determinación de la calciuria se realiza cuando las cifras de calcio y PTH en suero están
elevadas. Esta determinación es necesaria para descartar una HHF antes de la exploración
quirúrgica, así como para decidir cuando intervenir a un paciente con hiperparatiroidismo
primario asintomático
La fosfaturia está elevada, siendo superior a 800 mg/24 h. El aclaramiento de fósforo unido a la
creatinina está elevado, siendo superior a 12 ml/mn
Las fosfatasas alcalinas esta elevadas en las formas con signos óseos así como la citremia
(por encima de 26 mg/dl) y la hidroxipolinuria. Puede aparecer con frecuencia una acidosis
metabólica con hipercloremia.
Determinación de la PTH: es determinante. La tríada analítica clásica diagnóstica del
hiperparatiroidismo primario incluye: hipercalcemia, hiper-PTH e hipofosforemia.
También su pueden realizar pruebas de frenaje por perfusión cálcica y una prueba de
estimulación por hipocalcemia provocada por medio de EDTA sódico, esta prueba aporta pocos
datos útiles debido a su dificultad de interpretación.
ALTERACIONES DEL METABOLISMO DEL CALCIO
AUTORES:
Carlos López Vargas
Coral Suero Méndez
José Mª Benavente Ramos
14.- Se trata de paciente masculino de 14 años de edad higiénica dietética, convivencia con
aves, gatos e ingestión de berros. Inicia su padecimiento actual hace dos semanas con fiebre
de 39ºC sin predominio de horario, náusea y dolor en hipocondrio derecho. A la exploración
física con palidez generalizada y hepatomegalia 3-3-5. Resto de la exploración sin datos
patológicos. Laboratorio con BH, con Hb de 11.5, Hto de 40, leucocitos; 16,500, eosinófilos;
38%, linfocitos; 30%, formas inmaduras; 4, plaquetas; 270,000, Fosfatasa alcalina; 280 UI/L. El
diagnóstico más probable ES:
a) Larva migrans visceral
b) Toxoplasmosis generalizada
c) Fasciolosis
d) Gnatostomiasis
Definición
La Fasciolosis es una zoonosis parasitaria causada por la Fasciola hepática que ocasiona
patología y sintomatología hepato-biliar.
Clínica
Se considera las siguientes formas de presentación clínica:
a) Sintomática: Aguda o invasiva, crónica o de localización y extrahepática.
1. Aguda o invasiva: Hay tres elementos esenciales a identificar: hepatomegalia dolorosa,
fiebre y eosinofilia con cifras que superan frecuentemente el 30-40%.
2. Crónica o de localización: La sintomatología y signología corresponden a padecimiento
crónico hepato biliar incluyendo cólicos biliares y litiasis biliar.
3. Extrahepática: Incluye nódulos subcutáneos en el hipocondrio derecho, seno derecho,
escápula derecha con poco dolor local y signos inflamatorios. Eosinofilia alta.
b) Asintomática: En algunas personas los síntomas o signos suelen pasar desapercibidos.
Diagnóstico de Laboratorio
En la forma aguda, la búsqueda de huevos en las deposiciones es inútil, ya que las formas
juveniles están en el tejido hepático, por lo tanto las pruebas inmunobiológicas son importantes.
Son útiles la inmunoelectroforesis o inmunodifusión buscando el arco 2 de Caprón (4). El
inmunoblot o westernblot tiene buena sensibilidad y especificidad (5,6). Se han identificado
fracciones antigénicas en las cistenilproteasas de F. hepatica (7).
En las formas crónicas, la búsqueda de huevos en heces es lo indicado. Son útiles la
sedimentación rápida de Lumbreras (8). Recientemente se han preparado anticuerpos
monoclonales contra el parásito y ello ha permitido elaborar la técnica de ELISA para detectar
los coproantígenos (E/S) del parásito en heces. La ecografía de vías biliares (v.b.) puede
detectar al parásito moviéndose en las v.b. o vesícula. En formas extrahepáticas, la eosinofilia
alta es orientadora; F. hepatica en las biopsias confirma el diagnóstico.
15.- Ante un paciente con diagnóstico de trastorno obsesivo-compulsivo, usted decide indicar el
siguiente fármaco por ser el principal tratamiento de elección:
a) Antipsicóticos.
b) Antidepresivos inhibidores de la recaptación de noradrenalina.
c) Antidepresivos inhibidores de la recaptación de serotonina.
d) Benzodiacepinas.
El abordaje farmacológico del TOC se basa en los inhibidores selectivos de la recaptura de
serotonina (ISRS), medicamentos que han resultado ser efectivos y seguros. Como grupo, los
ISRS son igual de eficaces que la clorimipramina, pero producen menos efectos secundarios y,
por lo tanto, mejor tolerancia y mejor apego al tratamiento. La efectividad antiobsesiva parece
ser independiente de su actividad antidepresiva.
Entre los factores predictores de respuesta al tratameinto, se ha propuesto que los altos
puntajes en las compulsiones predicen una mala respuesta al tratamiento. Para los casos
refractarios y resistentes o cuando hay síntomas de comorbilidad, se han utilizado, con relativo
éxito, combinaciones con diferentes IRS, con benzodiacepinas, o bien, con potenciadores
como el litio, o antipsicóticos, como la risperidona y el haloperidol. El uso del carbonato de litio
es controvertido, aunque parece ser útil como potenciador a largo plazo entre 15 y 30% de los
pacientes. La combinación de ISRS con antipsicóticos comenzó a utilizarse en los pacientes
con síntomas psicóticos, aunque ahora se combinan también en los pacientes resistentes.
Tratamiento farmacológico del TOC
Cristina Lóyzaga*
Humberto Nicolini*
*División de Investigaciones Clínicas. Instituto Nacional de Psiquiatría.
Ramón de la Fuente. Calzada México-Xochimilco, 101, San Lorenzo
Huipulco, 14370, México D.F.
Primera versión: 14 de septiembre de 2000.
Aceptado: 26 de septiembre de 2000.
16.- Masculino de 44 años, con antecedentes de mialgias, artralgias de grandes
articulaciones, fatiga y pérdida ponderal desde 5 meses. Hace 2 semanas presentó febrícula y
10 días después orina oscura y tos seca. Dentro de sus antecedentes menciona infecciones
recurrentes de los oídos. A la exploración física cuenta con 37.2 °C, frecuencia cardiaca de 110
por minuto, frecuencia respiratoria de 18 por minuto, tensión arterial de 130/80 mm Hg, oídos
con cicatrices en ambos tímpanos, tórax con estertores en ambas bases, dolor costovertebral y
en extremidades hemorragia periungueal en los dedos. Laboratorio: leucocitosis de 13,200.
Hepatocitos: 35 %, VSG 55 mm/h, BUN 49 mg/dl, creatinina 4.0 mg/dl, serología con factor
reumatoide 1/40, VDRL negativo, anticuerpos citoplasmáticos antineutrófilos positivos,
sedimento urinario con cilindros hemáticos-gabinete: Rx con infiltrados alveolares bilaterales.
El diagnóstico más probable de este paciente es.
a) Poliarteritis.
b) Polimialgia reumática.
c) Vasculitis probablemente granulomatosa de Wegener.
d) Púrpura de Henoch-Schonlein.
La Púrpura de Schönlein-Henoch es un síndrome vasculítico sistémico, de pequeños vasos,
con afectación primaria de la piel, articulaciones, tuvo digestivo y riñones (1-11). Es la causa
más frecuente de púrpura no trombocitopénica y vasculitis en niños. (1) (9)
Se postula que la etiopatogenia puede ser debida a una alteración inmunológica, pues en esta
enfermedad se describen una gran variedad de anormalidades de la IgA, así como niveles
elevados de citosina, FNT alfa e IL-6. Se plantea una asociación entre esta respuesta
inmunitaria y una causa infecciosa, en la literatura se describen casos desencadenados por
Estreptococo sp, virus de la hepatitis A y C, Salmonella sp, Mycoplasma, virus de Epstein Barr,
varicela, parvovirus B19, adenovirus, parainfluenza, campylobacter y yersinia entre otros,
también se invocan determinados fármacos, alimentos, vacunas, exposición al frío y picaduras
de insectos. (1, 3, 9)
Parece ser que la afinidad de los gérmenes por el endotelio y los hematíes se debe a un
esfingofosfolípido neutro en la pared de estas células y su efecto citotóxico está mediado por
una proteína no estructural del germen, que induce la apoptosis mediante la ruta del TNF. Este
efecto puede explicar el daño selectivo hacia los capilares característicos de la PSH, y hacer
que las células sean más susceptibles al depósito de IgA que sufren. (9)
Los hallazgos histopatológicos son los típicos de la vasculitis leucocitoclástica ligados
patogénicamente al depósito de inmunocomplejos, activación del complemento, quimiotaxis de
neutrófilos, con daño secundario de la pared vascular debido a la liberación de enzimas por
parte de los neutrófilos activados. (1, 9, 12, 13)
El diagnóstico se basa fundamentalmente en las manifestaciones clínicas. (1, 2)
17.- Femenino de 18 meses, originário de zona rural del estado de Guerrero, es traído a
consulta por cojera desde que empezó a caminar (a los 16 meses). A la exploración presenta
marcha en Trendelenburg. La primera sospecha diagnóstica es:
a) Sinovitis transitoria de cadera.
b) Enfermedad de Perthes.
c) Luxación congénita de cadera.
d) Artritis séptica de cadera. .
La falta de relación normal en las estructuras que forman una articulación se conoce como
luxación. En el caso de la cadera, la cabeza del fémur no encaja debidamente en su cavidad
(acetábulo).
El desarrollo del acetábulo será normal siempre que la cabeza del fémur permanezca en
posición correcta y el esfuerzo muscular sea adecuado.
Siempre que esto no se cumpla, se producirá una alteración entre el cótilo y la cabeza femoral,
dando lugar a una alteración con el resultado de luxación de la cadera.
FACTORES DE RIESGO
El factor principal en cuanto a la estructura es la falta de estabilidad en la articulación debido a
la laxitud de la cápsula y del ligamento redondo.
Entre los factores genéticos más destacados estaría pertenecer a una familia con
antecedentes, factor que se observa en ciertas razas, (en relación con una laxitud ligamentosa
familiar).
La frecuencia es mayor en las niñas que en los niños, factor que se incrementa más cuando la
presentación del feto es de nalgas completas.
La posición del niño cuando ha nacido también es determinante, siempre que la posición sea
estable se reduce el riesgo de luxación.
DIAGNÓSTICO: EXPLORACIÓN FÍSICA
a) Maniobras de Ortolani y Barlow - click audible-. Ortolani: el click se produce cuando la fóvea
de la cabeza del fémur encuentra la cresta cotiloidea caminando sobre ella. La limitación a la
abducción desaparece, esto es signo de reducción.
Barlow: se realiza la aducción de la cadera, que tras una ligera presión longitudinal sobre el
fémur, produce una luxación.
La prueba de Ortolani traduce una luxación y la de Barlow una cadera luxable.
b) Asimetría de pliegues -no valorable en luxaciones bilaterales-.
c) Limitación a la abducción.
d) Dismetría
La marcha de Trendelenburg también es llamada marcha en sacudida del glúteo medio o de
los abductores. Observamos la marcha de Trendelenburg en pacientes con mala función de los
abductores. Si estos músculos se acortan, pierden su ventaja mecánica. El glúteo mediano,
que es el principal abductor de la cadera, se origina en la superficie lateral del ilíaco y se
inserta en la superficie superior del trocánter mayor. Si una enfermedad de la cadera provoca
que el trocánter mayor se localice más proximalmente de lo normal (se mueve más cerca de la
masa muscular del glúteo mediano), la contracción del músculo generará una fuerza abductora
menor.
Son causas de dicho acortamiento:
- Luxación congénita de cadera
- Coxa vara debida a:
* Causa idiopática
* Fractura del cuello femoral consolidada con mala alineación
* Enfermedad de Perthes
* Deslizamiento epifisario
* Raquitismo
García Portabella, M.
Luxación congénita de cadera antes de los tres meses de edad. 2001.
Disponible en:
http://www.vhebron.es/htr/ortopediatria.
• Garcia-Siso Pardo, J. M.
Displasia del desarrollo de la cadera.
(Parte I). Peditría Rural y Extrahospitalaria.
Vol. 32. Nº 304 Págs. 481-491. 2002.
• Graf, R.
Classification of hip joint dysplasia by means of
sonography.
Arch Orthop Trauma Surg 102:248-255, 1984.
• Edeiken, J.
Luxación congénita de cadera.
Diagnóstico Radiológico de las Enfermedades de los Huesos.
Ed. Médica Panamericana. Págs. 388-392.
18.-The following conditon is commonly seen in the magnesium-amonium-phosphate (struvite)
stones:
a) Recurrent P. mirabilis infection
b) Double J stent placement
c) Resorptive hypercalciuria
d) Renal azotemia
Esta bacteria de colonias redondeadas tiene la habilidad de producir grandes niveles de
ureasa. La ureasa hidroliza urea a amoníaco, (NH3) y eso hace a la orina más alcalina. Y al
subir la alcalinidad puede liderar la formación de cristales de estruvita, carbonato de calcio, y/o
apatita. Esta bacteria puede encontrarse en cálculos, y esas bacterias escondidas allí, pueden
reiniciar una infección post tratamientos antibióticos. Al desarrollarse los cálculos, después de
un tiempo pueden seguir creciendo más y causar obstrucción dando fallas renales.
Proteus también puede producir infecciones de heridas, septicemia y neumonías, sobre todo en
pacientes hospitalizados.
 Esipov, Sergei E. and J. A. Shapiro (1998). «Kinetic model of Proteus mirabilis swarm
colony development». Journal of Mathematical Biology 36 (3).
doi 10.1007/s002850050100.
 Frénod, Emmanuel (2006). «Existence result for a model of Proteus mirabilis swarm».
Differential and integral equations 19 (6): pp. 697-720.
http://arxiv.org/abs/math.FA/0702761.
19.- Femenino de 34 años, es atendida en sala de partos secundario a eutocia, durante la
reparación de la episiotomía media hay un marcado incremento en el sangrado transvaginal. La
causa más probable de la sintomatología de esta paciente es:
a) Retención de restos placentarios
b) Atonía uterina
c) Laceración cervical
d) Laceración vaginal
Atonía uterina
La atonía uterina o inercia, consiste en la falta de contracción del útero gestante luego del
alumbramiento. Este fenómeno ocurre en el 2 al 5 % de los partos por vía baja. La atonía
uterina es favorecida por numerosas situaciones como: la multiparidad, la sobre distensión
uterina, la macrosomía, el hidramnios, los intentos de versión, los antece3
0 M. Pesaresi, J. Palacios Jaraquemada Patología de Urgencia, Año 9, Nro. 3, Septiembre de
2001 dentes de hemorragia pre y posparto, la corioammnionitis y la interrupción en la
administración de oxitócicos, entre otros. No ha sido demostrada una relación directa entre las
atonías y el uso de los anestésicos administrados por vía peridural. Por el contrario, hay una
asociación demostrada con el uso de Halotano como anestésico general. El tratamiento clásico
de la atonía uterina está constituído por la reposición volumétrica (sangre, coloides y
cristaloides), la utilización de masajes y por la administración de drogas que promueven la
contracción del músculo uterino. Cuando estas medidas no son eficientes, se procede casi
invariablemente a la histerectomía de hemostasia. Es de notar, que la atonía uterina puede
aparecer en el primer embarazo y sin ningún antecedente previo. La atonía uterina constituye,
en casi todas las series mundiales, más del 50% de las hemorragias graves del posparto.
Patología de Urgencia, Año 9, Nro. 3, Septiembre de 2001
20.- Lactante de 10 meses. Acude al servicio de urgencias por presentar, vómito en 5 ocasiones,
fiebre 38.7 ºc, Antecedentes: Acude a guardería. Desde hace 2 días presentó aumento en el
número de evacuaciones en promedio de 4 a 5 veces al día, de consistencia líquidas,
abundantes, sin moco ni sangre, actualmente ha pesnetado 6 en las últimas 6 hrs. . Previamente
sano. Exploración física: Temp. 38. 5, Oc, alerta, sed aumentada bebe con avidez, ojos
hundidos, llanto sin lágrimas, mucosa oral seca, fontanela anterior hundida.
El tratamiento de primera elección para este paciente es:
a) Plan a de hidratacion oral.
b) Plan b de hidratacion oral.
c) Plan c de hidratacion oral.
d) Antibiotico y ayuno.
Las enfermedades diarreicas, de acuerdo con la evaluación del estado de hidratación, se
clasifican en: casos sin deshidratación, con deshidratación, con choque hipovolémico por
deshidratación.
7.2.3 Caso sin deshidratación, es aquel que presenta generalmente menos de cuatro
evacuaciones líquidas en 24 horas, ausencia de vómito, sin signos clínicos de deshidratación.
7.2.4 Caso con deshidratación, es aquel que presenta dos o más de las manifestaciones
clínicas siguientes:
7.2.4.1 Inquieto o irritable;
7.2.4.2 Ojos hundidos, llanto sin lágrimas;
7.2.4.3 Boca y lengua secas, saliva espesa;
7.2.4.4 Respiración rápida;
7.2.4.5 Sed aumentada, bebe con avidez;
7.2.4.6 Elasticidad de la piel, mayor o igual a dos segundos;
7.2.4.7 Pulso rápido;
7.2.4.8 Llenado capilar de tres a cinco segundos;
7.2.4.9 Fontanela anterior hundida (lactantes);
7.2.5 Caso con choque hipovolémico, es aquel que presenta dos o más de las manifestaciones
clínicas siguientes:
7.2.5.1 Inconsciente o hipotónico;
7.2.5.2 No puede beber;
7.2.5.3 Pulso débil o ausente;
7.2.5.4 Llenado capilar mayor de cinco segundos;
7.2.6 El manejo de los casos de enfermedades diarreicas se basa en tres planes generales
de tratamiento:
7.2.6.1 Plan A: Para pacientes con enfermedad diarreica sin deshidratación con atención en el
hogar:
7.2.6.1.1 Continuar con la alimentación habitual;
7.2.6.1.2 Aumentar la ingesta de los líquidos de uso regular en el hogar así como Vida Suero
Oral: de este último, en los niños menores de un año de edad, ofrecer media taza (75 ml) y en
los mayores de un año, una taza (150 ml) y administrarlo a cucharadas o mediante sorbos
pequeños, después de cada evacuación.
7.2.6.1.3 Capacitar a la madre para reconocer los signos de deshidratación y otros de alarma por
enfermedades diarreicas: (sed intensa, poca ingesta de líquidos y alimentos, numerosas heces
líquidas, fiebre, vómito y sangre en las evacuaciones), con el propósito de que acuda
nuevamente a solicitar atención médica en forma oportuna.
7.2.6.2 Plan B: Para pacientes con diarrea y deshidratación con atención en la unidad de salud:
7.2.6.2.1 Administrar Vida Suero Oral 100 ml por kilogramo de peso, en dosis fraccionadas cada
30 minutos durante cuatro horas;
7.2.6.2.2 Si el paciente presenta vómito, esperar 10 minutos e intentar otra vez la hidratación
oral, más lentamente;
7.2.6.2.3 Al mejorar el estado de hidratación, pasar al Plan A. En caso contrario, repetir el Plan B
por otras cuatro horas, de no existir mejoría pasar al Plan C;
7.2.6.2.4 Si los vómitos persisten, existe rechazo al Vida Suero Oral, o gasto fecal elevado (más
de 10 g/kg/hora o más de tres evacuaciones por hora) se hidratará con sonda nasogástrica, a
razón de 20 a 30 ml de Vida Suero Oral por kilogramo de peso, por hora.
7.2.6.3 Plan C: Para pacientes con choque hipovolémico por deshidratación:
7.2.6.3.1 Inicie inmediatamente administración de líquidos por vía intravenosa, con solución
Hartmann; si no se encuentra disponible, use solución salina isotónica al 0.9%, de acuerdo con
el siguiente esquema:
PRIMERA HORA
50 ml/kg
SEGUNDA HORA
25 ml/kg
TERCERA HORA
25 ml/kg
Evalúe al paciente continuamente. Si no mejora, aumente la velocidad de infusión.
Cuando pueda beber (usualmente en dos a tres horas), administre VSO, a dosis de 25
ml/kg/hora; mientras sigue líquidos IV.
Al completar la dosis IV, evalúe al paciente para seleccionar Plan A o B, y retirar venoclisis, o
repetir Plan C.
Si selecciona el Plan A, observe durante dos horas para asegurarse de que el responsable
encargado del paciente puede mantenerlo hidratado con VSO y además, alimentarlo en su
domicilio.
NORMA Oficial Mexicana NOM-031-SSA2-1999, Para la atención a la salud del niño.
Evaluación del estado de hidratación de un paciente con diarrea17
Signos Plan A Plan B Plan C
Definición Leve o inaparente Moderada o clínica Grave
Pérdida de agua
corporal
Menos de 50 ml/kg peso o
menos de 5% del peso
50 a 100 ml/kg peso ó
6 a 9% del peso
100 ml/kg peso o más
(10% o más del peso)
Condición
general
Bien, alerta Irritable Letárgico o inconsciente
Globo ocular Normales Algo hundidos Muy hundidos y secos
Lágrimas Presentes Ausentes Ausentes
Mucosa oral Húmeda Seca Muy seca
Sed Paciente bebe normalmente
Paciente bebe con
avidez, sediento
Paciente bebe mal o no
es capaz de hacerlo
Pliegue cutáneo
Vuelve a lo norma
rápidamente
Se retrae lentamente
< 2 s
Se retrae muy lentamente
> 2 s
Tratamiento Plan A Plan B Plan C
21.- Masculino de 17 años de edad, con antecedentes de cuadro gripal de tres días de evolución,
inicia hace 12 hrs. con dolor en la bolsa escrotal y aumento de volumen, a la exploración física
se encuentra edema. tensión y enrojecimiento del escroto y no se puede palpar el testículo
derecho. El diagnóstico más probable de este paciente es:
a) Torsión testicular
b) Orquiepididimitis
c) Hidrocele agudo
d) Hernia inguinal estrangulada.
El síndrome escrotal agudo es una urgencia urológica cuyo principal síntoma es el dolor intenso
del contenido escrotal, su importancia radica en el diagnóstico precoz de la torsión testicular,
cuadro de emergencia quirúrgica, por lo que se ha establecido un síndrome en el que se
engloban diversas afecciones intraescrotales cuyo denominador común es el dolor testicular de
aparición brusca que puede ir acompañado de otros síntomas dependiendo de su etiología,
como tumefacción escrotal, náuseas, vómitos, síndrome miccional... que nos orientarán sobre el
diagnóstico.
RECUERDO ANATOMICO
El escroto es la bolsa que aloja al testículo, éste es una glándula de forma tubular cuya capa
externa es la túnica albugínea. Del polo superior del testículo sale el epidídimo que tras
descender por la cara posterior de éste, se continúa con el cordón espermático. El cordón
espermático está formado por el conducto deferente, los vasos sanguíneos (arterias
espermáticas y deferencial y venas espermáticas), linfáticos y nervios que llegan a la glándula. El
cordón espermático está envuelto por el músculo cremaster.
La irrigación del testículo procede de la arteria espermática interna, rama de la aorta abdominal y
el drenaje venoso se realiza a través de la vena espermática interna que desemboca en la vena
cava inferior en el lado derecho y en la vena renal en el lado izquierdo.
Las funciones del testículo son endocrina, las células de Leydig sintetizan y liberan la
testosterona y exocrina que es la espermatogénesis.
ORQUITIS Y EPIDIDIMITIS
La inflamación del contenido escrotal plantea un problema de diagnóstico en el escroto agudo
principalmente con la torsión funicular. La infección aislada del testículo es rara, siendo más
frecuente la presentación en forma de orquiepididimitis. Predomina en hombres jóvenes
sexualmente activos y en ancianos con infección urinaria pero se puede producir a cualquier
edad.
La causa más frecuente de la inflamación del contenido escrotal es la infección. Así en los
jóvenes prepuberales los gérmenes más frecuentes son las enterobacterias aunque sobre
factores predisponentes como son las malformaciones congénitas (válvulas uretrales,
abombamiento ectópico de un uréter a vesícula seminal). En el adolescente es causa frecuente
la orquitis en el transcurso de una parotiditis. En los varones jóvenes menores de 40 años es la
causa infecciosa con mucho lo más frecuente, sobre todo enterobacterias (E. coli, Proteus
Mirabilis, Klebsiella pneumoniae) y gérmenes de transmisión sexual como Neisseria gonorrheae,
Mycoplasma, pero sobre todo la Chlamydia tracomatis, y cocos gram positivos como
estafilococos aureus. Por encima de los 40 años a la causa infecciosa se suele añadir una
obstrucción urinaria distal, tuberculosis, reflujo urinario posterior a la prostatectomía transuretral,
y la reacción granulomatosa por el tratamiento del cáncer vesical superficial con bacilo de
Calmette Guerin (BCG).
También puede aparecer orquiepididimitis en el curso de la varicela, fiebre tifoidea,
mononucleosis infecciosa, rickettsiosis, brucelosis filariasis, actinomicosis, sinusitis, osteomielitis,
endocarditis y ser secundaria a cuadros septicémicos por E. coli, Klebsiella, pseudomona,
estreptococo y estafilococo. Una causa no infecciosa es el tratamiento con amioradona.
Clínicamente aparece dolor escrotal intenso, irradiado a lo largo del cordón espermático por lo
que irradia a ingle e incluso a abdomen con tumefacción escrotal no distinguiéndose el epidídimo
del testículo, con induración de la piel escrotal que puede producir hidrocele reactivo con
síntomas de infección urinaria concomitantes y de secreción uretral, además de afectación del
estado general con fiebre que puede llegar a ser de hasta 40 °C, con escalofríos, acompañado
de náuseas y vómitos.
En la exploración física, el escroto se observa edematoso, tenso y enrojecido,
el cordón se palpa engrosado y doloroso, siendo muy dolorosa la palpación, y difícil de distinguir
el límite entre escroto y epidídimo en casos evolucionados. El signo de Prehn, al elevar el
testículo afecto produce mejoría del dolor, es positivo. La transiluminación del escroto es
negativa a no ser que exista un hidrocele reactivo, en tal caso será positiva.
En cuanto a los estudios complementarios habrá que realizar analítica de orina, cultivo de orina,
y de semen y de secreción uretral si la hubiere, cultivo de Lowenstein y tinción de Ziehl-Nielsen
tanto en orina como en semen e inmunofluorescencia para Chlamydia, además de antibiograma.
También es necesario realizar hemograma, hemocultivos seriados, una bioquímica sanguínea
que incluya urea, creatinina e iones.
En el sedimento de orina nos mostrará piuria en la mayoría de las ocasiones, discreta piuria sin
bacteriemia en las ETS y piuria franca con urocultivo positivo en caso de gérmenes gram
negativos, aunque también podremos encontrar a pacientes con sedimento urinario negativo. En
el hemograma es frecuente leucocitosis con desviación a la izquierda.
La ecografía nos mostrará engrosamiento y edema de las túnicas escrotales, pudiendo haber o
no hidrocele, con epidídimo engrosado (patrón heterogénico), y un testículo aumentado de
tamaño (patrón hipoecogénico global) o como una zona hipoecogénica perihiliar.
La eco-dopopler está indicada cuando hay dudas sobre el diagnóstico diferencial con la torsión
funicular, siendo en estos casos muy útil, y en la orquiepididimitis encontraremos una
hipervascularización.
La gammagrafía está indicada para el diagnóstico diferencial con procesos isquémicos y
neoplásicos, es un método de gran sensibilidad y especificidad,
pero que no se tiene siempre disponible en el área de urgencias.
Se observará un aumento de la captación a nivel testicular debido a la hiperemia. Si después de
todas las pruebas complementarias existen dudas sobre el diagnóstico, será necesaria la cirugía
para llegar al diagnóstico definitivo.
Referencias
1. Young Y., Miller R Incidence of malignant tumours in U.S. children. J Pediatr 1975; 86: 254-
258.
2. Reiter A., Schrappe M. Chemotherapy in 998 unselected childhood acute lymphoblastic
leukemia patients. Results and conclusions of the multicenter trial ALL-BFM 86. Blood 1994;
84: 3122-3133.
3. Schorin M., Blattner S., Gelber R., et al.
Treatment of childhood acute lymphoblastic leukemia: Results of Dana Farber Cancer
Institute/Children s Hospital acute lymphoblastic
leukemia consortium protocol 85-01. J Clin
Oncol 1994; 12: 740-747.
22.-Se trata de escolar de 10 años con los siguientes antecedentes: a los 6 años y en varios
análisis se detecta hematuria microscópica, con niveles IgA normales y normocalciuria. A la edad
de 9 años persistía la hematuria en los análisis e incluso habían observado algún episodio
recortado de hematuria macroscópica. Un año más tarde se detectó proteinuria moderada de
1250 mg/24 horas. En el momento de la consulta persisten las alteraciones en el sedimiento,
pero la proteinuria es de rango nefrótico, con creatinina sérica de 1,3 mg/dl. Existen
antecedentes familiares de nefropatía evolutiva con desarrollo de insuficiencia renal y de miopía
familiar por “lenticonus”. El diagnóstico más probable es del menor es:
a) Glomerulonefritis mesangial (enfermedad de Berger).
b) Enfermedad poliquística autosómica dominante.
c) Nefritis intersticial por hipersensibilidad.
d) Enfermedad de Alport
En medicina, la enfermedad de Alport es una enfermedad genética ligada al cromosoma X, en
la que una mutación en la síntesis del colágeno afecta los riñones, oídos, y ojos causando
sordera y trastornos de la vista, incluyendo distrofia de la córnea y cataratas. Fue inicialmente
identificado por el médico británico Cecil A. Alport en 1927, que describió una familia británica en
la que muchos miembros desarrollaban enfermedades renales. Él describió que los hombres
afectados en la familia morían a causa de enfermedades renales, mientras que las mujeres
estaban menos afectadas.
Se conoce ahora que muchos casos del síndrome de Alport son causados por una mutación en
el gen del colágeno COL4A5, entre otras. Este gen codifica la cadena alfa-5 del colágeno tipo IV
y está localizado en el cromosoma X. En ciertas familias se ven plaquetas de gran tamaño
circulantes en la sangre, trombocitopenia e inclusiones leucocitarias que se parecen a las
encontradas en la anomalía de May-Hegglin
El síndrome de Alport se caracteriza por tener afección renal, coclear y ocular. La principal señal
de este síndrome es la hematuria microscópica (microhematuria). Los hombres con el síndrome
Alport ligado al cromosoma X (XLAS) padecen microhematuria desde una edad muy temprana.
Alrededor del 90% de mujeres con XLAS también la tienen. Hay 2 métodos para el diagnóstico
clínico: secuenciación y análisis de deleción/duplicación. El análisis de secuenciación de
COL4A5 identifica cerca del 80% de las mutaciones de individuos afectados con antecedentes
familiares en herencia ligada al X. El análisis de deleción/duplicación del gen COL4A5 identifica
deleciones (típicamente multiexónicas) cercanas al 10% de individuos afectados con
antecedentes familiares ligada al X
Referencias
Behrman RE, Kliegman RM, Jenson HB. Nelson Textbook of Pediatrics. 17th ed. Philadelphia,
Pa: Saunders; 2003.
23.- Masculino de 2 de dos meses de edad, con sospecha de menigoencefalitis por clínica y un
líquido cefalorraquídeo con aumento de leucocitos, aumento de proteínas e hipoglucorraquia en
el que no se tiene aún un germen aislado. El tratamiento de elección es:
a) Dicloxacilina y amikacina
b) Ampicilina y cefotaxime
c) Penicilina sódica cristalina
d) Vancomicina
La meningoencefalitis es definida como la inflamación de las meninges y el encéfalo. Dentro de
las causas de la meningoencefalitis purulenta la más frecuente es la bacteriana.
La etiología varía dependiendo del grupo de edad, en menores de 2 meses el más común es
Streptococcus del grupo b, después de esta edad el más frecuente es el S. pneumoniae. El
cuadro clínico se caracteriza por 4 síndromes: infeccioso, de hipertensión endocraneana,
meníngeo y de daño neuronal. Las complicaciones pueden ser agudas, subagudas y crónicas. El
diagnóstico se hace con el estudio citoquímico y cultivo del LCR. Las medidas terapéuticas irán
encaminadas al manejo de los signos y síntomas componentes de los 4 síndromes. El
tratamiento específico se iniciará en forma empírica y se modificará de acuerdo a los resultados
del cultivo.
En la meningitis bacteriana el aspecto macroscópico LCR es turbio o incluso purulento debido al
mayor contenido de células y proteínas. En la mayoría de los casos de meningoencefalitis
bacteriana el número de leucocitos es superior a 500 x mm3; y el predominio de células
polimorfonucleares es mayor a 50%.
La presencia de hipoglucorraquia (< 50% de la glicemia central o glucorraquia < 40 mg/dL) e
hiperproteinorraquia moderada (entre 200 y 500 mg/dL) sugiere etiología bacteriana. Sin
embargo, en casos con cuadros neurológicos de evolución más prolongada, mayor de 10 días, el
diagnóstico diferencial debe establecerse con meningoencefalitis de etiología tuberculosa.
Enfermedades infecciosas y microbiología
Número number 1 enero-marzo
January-march 2002 volumen volume 22
Meningoencefalitis bacteriana
Fortino solórzano santos,* maría guadalupe miranda novales,**
Rita d díaz ramos*
24.- Un niño de 12 años en Yautepec Morelos, se encuentra dormido sobre una toalla a la orilla
de una alberca, súbitamente presenta dolor intenso en muslo derecho y se observa una
pequeña pápula eritematosa El niño refiere sensación de ardor intenso en el muslo, su madre
aplica una pomada con antihistamínico y le administra un antihistamínico oral.
Dos horas después el paciente inicia parestesias, nausea y vómito, comienza a presentar dolor
abdominal intenso, en episodios, síntomas que en una hora se hacen más intensos, la causa
más probable de este cuadro es:
a) Picadura de viuda negra.
b) Picadura de alacrán.
c) Picadura de cara de niño.
d) Picadura de nauyaca
El veneno que inocula es 15 veces más potente que el veneno de una serpiente de cascabel y
puede condicionar efectos sistémicos graves e incluso la muerte.
El primer síntoma generalmente es un dolor similar a una punción con un alfiler y la sensación se
experimenta cuando realmente se ha efectuado la picadura de la araña. Sin embargo, es posible
que algunas personas no lo sientan. Puede haber hinchazón y enrojecimiento leve y una lesión
en forma de diana.
De 15 minutos a una hora más tarde, un dolor muscular sordo se irradia desde el área de la
picadura a todo el cuerpo.
Síntomas
Náuseas
Ataque al estado general
Diaforesis
Contracturas musculares
Dolor muscular
Retención urinaria
Estreñimiento
Taquicardia
Insuficiencia cardiaca
Hipertensión arterial
Inquietud
Ansiedad
Sensación de muerte
Inminente
Current Pediatric Diagnosis and Treatment 17 Ed Mc Graw Hill. Pag 346 . 2005
25.- Preescolar de 3 años es llevado a consulta por preentar hiporexia. Antecedentes: Originario
de zona rural del estado de Guerrero, geofagia positiva, dolor abdominal, cólico desde hace
varios meses. Las evacuaciones en los últimos 5 días son semilíquidas acompañadas de moco y
pujo no sangre. E.F.: mala higiene personal, desnutrido, abdomen globoso blando y dolor a la
presión en colon descendente, peristalsis aumentada. La complicación más frecuente que se
puede presentar en este paciente es:
a) Apendicitis.
b) Obstruccion intestinal.
c) Prolapso rectal, trichurosis
d) Perforación intestinal.
Introducción.
La trichuriosis es una geohelmintiasis frecuente en zonas tropicales, rurales. Se estima que se
encuentran infectadas unos 100 millones de personas en Latinoamérica y Caribe (Hotez PJ, et
al., 2008). Predomina en niños en edad escolar, en quienes se asocia a colitis crónica y
síndrome disentérico, retardo en el crecimiento y disminución de peso; la deficiencia en las
funciones cognitivas y alteraciones conductuales se han relacionado con anemia ferropriva, altas
cargas parasitarias y desnutrición. Los casos de la parasitosis en adultos que viven en zonas
endémicas han aumentado, pero no se reportan usualmente. (Khuroo M, et al. 2010).
La ascariosis y la trichuriosis son las infecciones por geohelmintos más frecuentes en México.
NTDs en LAC: Prevalencia y distribución. Hotez PJ, et al, 2008.
LAC: Latin American and Caribbean Region
NTDs. Neglected Tropical Diseases.
Las lesiones intestinales y el cuadro clinico varían en relación directa al número de parásitos y
factores dependientes del hospedero (edad, estado nutricional, infecciones concomitantes).
En infecciones leves y moderadas el daño, apenas apreciable, consiste en compresión mecánica
de las células de la mucosa colónica y se asocia a dolor abdominal de tipo cólico y episodios
diarreicos.
En infecciones masivas la mucosa intestinal se encuentra edematosa y friable, con sangrado
fácil; es característica la degeneración y necrosis de las células cercanas a la cabeza del
parásito, con pequeñas hemorragias subepiteliales e inflamación con infiltración difusa de
linfocitos y eosinófilos.
Induce, al igual que los otros geohelmintos, una respuesta de tipo Th2 y respuesta reguladora
Th2/Treg (Jackson JA, et al. 2009).
Las manifestaciones clínicas varían de acuerdo a la masividad de la infección y la presencia de
otros parásitos (poliparasitismo) e incluyen dolor abdominal, cefalea, hiporexia, pérdida de peso,
diarrea crónica, disentería, pujo, tenesmo, prolapso rectal y signos y síntomas relacionados con
anemia hipocrómica microcítica; cada tricocéfalo expolia alrededor de 0.005 ml de sangre/día y
restos tisulares. Además, la irritación constante de las terminaciones nerviosas intramurales
redunda en hiperperistaltismo.
Complicaciones.
- Poliparasitismo
- Prolapso rectal
- Anemia
- Apendicitis
- Infección bacteriana 2aria
- Retraso pondoestatural y déficit cognitivo en escolares.
TRICHURIOSIS
Dra. Teresa Uribarren Berrueta
Departamento de Microbiología y Parasitología, Facultad de Medicina, UNAM October, 2010
Vínculos.
- Mohammad S. Khuroo, Mehnaaz S. Khuroo, and Naira S. Khuroo. Trichuris dysentery
syndrome: a common cause of chronic iron deficiency anemia in adults in an endemic area (with
videos). Gastrointestinal Endoscopy, Jan 2010; 71(1):200-204. doi:10.1016/j.gie.2009.08.002
- Geary TG, Woo K, McCarthy JS, Mackenzie CD, Horton J, Prichard RK, de Silva NR, (...),
Bundy DA. Unresolved issues in anthelmintic pharmacology for helminthiases of humans. Int J
Parasitol 2010;40(1):1-13. doi:10.1016/j.ijpara.2009.11.001 Geohelmintos y otros nematodos.
- Kyung-Sun Ok, et al. Trichuris trichiura Infection Diagnosed by Colonoscopy: Case Reports and
Review of Literature. Korean J Parasitol. Sept 2009;47(3):275-280
DOI: 10.3347/kjp.2009.47.3.275
- Hu Y, Xiao S-H, Aroian RV. The new anthelmintic tribendimidine is an L-type (Levamisole and
Pyrantel) nicotinic acetylcholine receptor agonist. PLoS Neglected Tropical Diseases 2009;3(8),
art. no. e499.
- Jackson JA, Friberg IM, Little S, Bradley JE. Review series on helminths, immune modulation
and the hygiene hypothesis: Immunity against helminths and immunological phenomena in
modern human populations: Coevolutionary legacies? Immunology 2009;126 (1):18-27.
doi:10.1111/j.1365-2567.2008.03010.x
26.- Acude al servicio paciente masculino de 66 años de edad el cual refiere disminución de
la visión en ambos ojos con varios meses de evolución, esta disminución se presenta a la
visión de lejos y más acentuada en la visión próxima o de lectura. Además ha notado mayor
pérdida visual con luz solar intensa y se deslumbra con mayor facilidad. No refiere alteraciones
en la percepción de los colores, aunque sí cree verlos más apagados, tampoco refiere
metamorfopsias. El diagnóstico más probable es:
a) Aumento de la presbicia.
b) Desarrollo de cataratas.
c) Desarrollo de neuropatía óptica anterior isquémica.
d) Desarrollo de glaucoma.
Cataratas.
Cualquier opacidad del cristalino, conlleve o no incapacidad funcional.
CATARATA CONGÉNITA.
Por alteraciones a partir de la 4ª ó 5ª semana de embarazo. Aparecen al nacer o en los
primeros tres meses de vida. Provocan más del 10% de las cegueras en edad escolar. Pueden
ser:
• Hereditarias (10-25%). Bien aisladas o asociadas a malformaciones oculares o sistémicas.
Comportamiento familiar y generalmente bilaterales.
• Embriopatías. Por infecciones intrauterinas, sobre todo durante el primer trimestre de
embarazo, como rubéola, toxoplasmosis y citomegalovirus.
• Metabólicas.
- Galactosemia. Por déficit del enzima galactosa 1-P uridil transferasa, que se transforma en
galactitol. Este se deposita en el cristalino, provocando una retención de agua que lo opacifica.
A medida que el niño va tomando leche van apareciendo cataratas, hepatoesplenomegalia y
retraso mental. Reversible, si se diagnostica en las primeras fases de la enfermedad .
- Hipoparatiroidismo o pseudohipoparatiroidismo: cuando el Ca2+ es menor de 9,5 mg/100 ml,
se puede originar catarata.
• Tóxicas. Por clorpromacina, corticoides, hipervitaminosis D.
• Carenciales. Por déficit de vitamina A, triptófano, ácido fólico o vitamina B12.
• Cromosomopatías. Síndrome de Down (50% tienen cataratas),
Turner.
CATARATAS SECUNDARIAS.
Aparecen en el curso de procesos oculares o sistémicos.
Oculares: queratitis, uveítis, tumores coroideos, traumatismos, desprendimiento de retina y
glaucoma, entre otras.
Sistémicas:
• Metabólicas. En diabetes, hipertiroidismo, enfermedad de Wilson, distrofia miotónica de
Steinert •
Sindermatóticas. En patología dermatológica: poiquilodermia, esclerodermia y eccema atópico.
• Tóxicas. Por metales como talio, plata, mercurio, hierro, cobre, o fármacos como corticoides,
mióticos, antimitóticos, ergotamina.
CATARATA SENIL.
Forma más frecuente de catarata y causa más frecuente de pérdida visual reversible en países
desarrollados. Son bilaterales, aunque de desarrollo asimétrico.
CLÍNICA.
Se produce una disminución progresiva de la agudeza visual, sin dolor ni inflamación. Mejora
en ambientes poco iluminados o tras instilar un midriático, y empeora en ambientes muy
iluminados. En algunos pacientes los síntomas comienzan con una mejoría de la presbicia, por
un aumento en el índice de refracción del cristalino. También pueden aparecer fotofobia, visión
de halos coloreados y diplopía monocular (debidos a diferencias de refracción entre zonas de
la lente).
Puede ser nuclear, cortical (por hiperhidratación) o subcapsular posterior. Se denomina
incipiente cuando la opacidad y el trastorno visual son mínimos, madura cuando la opacidad es
total y el déficit visual severo, e hipermadura, cuando se produce disolución y licuefacción de
las fibras, perdiendo su estructura y apareciendo de color gris homogéneo. La cápsula puede
presentar pliegues y el núcleo caer hacia abajo.
MUY IMPORTANTE
En un paciente de edad avanzada que progresivamente desarrolla disminución de la agudeza
visual que empeora con luz intensa y mejora de la presbicia, debemos sospechar catarata
senil.
COMPLICACIONES.
Si la catarata no se opera, pueden aparecer las siguientes complicaciones:
• Iridociclitis. Por salida de las proteínas del cristalino a través de la cápsula.
• Glaucoma secundario agudo o facomórfico. El cristalino capta agua y aumenta de volumen,
provocando un aplanamiento de la cámara anterior y un cierre angular con aumento de PIO.
• Glaucoma facolítico. Proteínas cristalinianas que pasan a cámara anterior y provocan una
obstrucción de la malla trabecular.
• Luxación del cristalino.
DIAGNÓSTICO.
El diagnóstico general de cataratas se hace provocando midriasis farmacológica y observando
a simple vista, apareciendo una leucocoria en los estadios avanzados; o bien observando
mediante oftalmoscopia directa a unos 30 cm, apreciándose manchas oscuras sobre el reflejo
rojo del fondo del ojo, o impidiendo ver este reflejo cuando son maduras; o bien con lámpara de
hendidura, que permite ubicar la opacidad dentro del cristalino.
TRATAMIENTO.
El tratamiento es exclusivamente quirúrgico. La técnica más utilizada se denomina
facoemulsificación
27.- Paciente femenino de 45 años que presenta palpitaciones, debilidad, palidez de
tegumentos, taquicardia, glositis, parestesias dístales y después ascendentes; y que cursa con
ataxia. Bh con Hb de 8; VCM 110 fL; leucocitos y plaquetas discretamente disminuidos. Lo
primero que pensaría es:
a) Infección crónica.
b) Leucemia.
c) Hemorragia.
d) Deficiencia de cobalamina.
Farreras. Medicina interna. Cap 206. Decimoquinta edición 2004, Pp. 1652-1653. Dado que la
cobalamina en el trofismo adecuado de la piel y las mucosas y el mantenimiento de la
mielinizaciòn adecuada, pueden aparecer alteraciones digestivas y neurológicas, como glositis
(de Hunter) y trastornos gastrointestinales inespecíficos (flatulencia, digestiones pesadas),
aunque algunos pacientes presentan diarrea que sólo cede con el tratamiento con cobalamina.
Las manifestaciones neurológicas se deben a degeneración axonal y desmielinizaciòn de los
cordones medulares posteriores (degeneración combinada subaguda).
28.- Femenino de 62 años con diagnóstico de miastenia gravis. De los siguientes
medicamentos el que está dirigido al manejo de esta patología es:
a) Neostigmina
b) Quinidina
c) Sumatriptán
d) Succinilcolina
Manejo farmacológico: Existen diferentes pautas terapéuticas dirigidas a contrarrestar los
síntomas de la enfermedad o el mecanismo inmunológico. Los fármacos utilizados son:
- Inhibidores de la Acetilcolinesterasa (Neostigmina, Piridostigmina). Dirigidos al manejo
sintomático de la MG, mejorando la fuerza motora pero no la progresión de la enfermedad. Su
mecanismo de acción es la inhibición reversible de la acetilcolinesterasa, lo cual genera un
aumento de ACh en la placa motora. La dosis a utilizar es variable y debe modificarse en
distintas etapas de la enfermedad, incluso siendo frecuente no lograr un efecto uniforme en los
diferentes grupos musculares en un mismo paciente. El objetivo por lo tanto será utilizar la
dosis mínima con la que se genere la mejor respuesta clínica. El efecto se obtiene de 30
minutos a 2 horas de la administración y tiene una duración de hasta 6 horas. Las dosis
recomendadas de Piridostigmina son de 15-60 mg cada 4-6 horas vía oral y de Neostigmina
0,5-2 mg/kg cada 4-6 horas intramuscular. Las reacciones adversas asociadas son: dolor
abdominal, hipersalivación, aumento de las secreciones respiratorias y bradicardia y se
relacionan con el efecto colinérgico generado, por lo que es necesario administrar
concomitantemente atropina2,3.
Ponsetia JM: Miastenia Gravis. Manual Terapéutico. Barcelona; Springer Verlag Ibérica, 1995
2.- Ponsetia JM, Espina E, Armengola M: Diagnóstico y Tratamiento de la Miastenia grave. Med
Clin (Barc) 2000; 115: 264-70. 3.- Drachman DB: Myasthenia gravis. N Engl J Med 1994;
330: 1797-810 4.- Andrews PI: Autoimmune myasthenia gravis in childhood. Semin Neurol
2004; 24: 101-10 5. - Anlar B: Juvenile myasthenia: diagnosis and treatment. Paediatr Drugs
2000; 2: 161-9. 6. - Gajdos P: Myasthenic syndrome. Diagnosis trends. Rev Prat 2000; 50:
419-23 7. -
29.- Femenino de 34 años la cual labora en lavandería, hace varios años presenta
enrojecimiento de los pliegues proximales de las uñas de varios dedos de las manos, que
ocasionalmente le supuran. El diagnóstico más probable de la paciente es:
a) Dermatitis de contacto.
b) Paroniquia candidiásica crónica.
c) Liquen plano.
d) Dermatoficia.
Infección por Candida del lecho ungueal que se presenta con mayor frecuencia como una
onicolisis asociada con paroniquia, aunque también se observa la destrucción completa del
lecho ungueal y la erosión de la zona distal y lateral de las uñas de los dedos, sin distrofia
ungueal total. La perionixis candidiásica se trata de la inflamación con eritema, edema, dolor y
salida ocasional de pus blanquecino escaso y espeso del reborde ungueal de uno o varios
dedos de la mano. Con frecuencia, se acompaña de onicopatía (discoloración, onicolisis,
distrofia total, etc.).
Predisponentes: Todas las labores manuales que generen humedad. (muy frecuente en amas
de casas, trabajadores de restaurantes, lavanderas, etc.)
BIBLIOGRAFÍA:
1. Crespo Erchiga V. Protocolo diagnóstico de contaminantes. En "Micología para
dermatólogos" Ed.
Janssen, Madrid, 1994, pp:49-70.
2. Crespo V, De Luís B, Delgado V, Crespo A y Vera. Espectro etiológico de las onicomicosis
en nuestro medio. CO7. II Congreso Nacional de Micología. Santiago de Compostela. Junio,
1994.
3. Crespo Erchiga V, Delgado Florencio V y Martínez García S. Micología dermatológica. Ed.
M.R.A. Barcelona, 2006.
4. Daniel III CR. The Diagnosis of Nail Fungal Infection. Editorial Arch Dermatol 1991;127:1566-
1567.
5. Delgado Florencio V. Protocolo de identificación de dermatofitos. En "Micología para
dermatólogos".
Ed. Janssen, Madrid, 1994, pp:27-41.
6. Delgado Florencio V. Estrategia en el diagnóstico y tratamiento de las micosis superficiales.
Ed.
Aula Médica, Madrid, 1994.
7. Delgado V, Abad Romero-Balmas J, Armijo Moreno M y Dulanto F. Scopulariopsis
brevicaulis como agente de onicomicosis. Actas Deermo-Sif. 1976; 9-10:693-700.
8. English MP. Nails and fungi. Br J Dermatol 1976; 94:697-701.
9. Fevilhade de Chauvin M. Onicomicosis. Dermatología práctica. 1994; 9:1-2.
30.- Femenino que desde hace varias semanas le han aparecido de forma eruptiva unas
máculas y placas eritematosas en el tronco, refiere que hubo una lesión más grande que
precedió a las demás. Las lesiones presentan una descamación fina en la periferia y son dis-
cretamente pruriginosas. No existe afectación palmoplantar. La serología luética es negativa.
EL tipo de pitiriasis más probable ES:
a) P. liquenoide crónica.
b) P. rubra pilaris.
c) P. rosada.
d) P. versicolor
La pitiriasis rosada es una enfermedad exantemática autolimitada de causa desconocida que
cursa con lesiones maculopapulosas de aspecto asalmonado en áreas proximales de las
extremidades y en tronco, donde se distribuyen de forma paralela a las líneas de la hendidura
(Stulberg DL, 2004).
Existen algunas características que sugieren una etiología vírica (pródromos, predominio
estacional, evolución variable, ausencia de recaídas, hallazgos histológicos...) que algunos
autores relacionan con el herpes virus humano 7, aunque otros lo desmienten (Chuh A, 2004).
Afecta con más frecuencia a niños mayores y jóvenes adultos (10-35 años) y es ligeramente
más frecuente en mujeres (Chuang TY, 1982).
El cuadro típico se inicia con una lesión redondeada u oval, de 4-8 cm. única, generalmente
situada en el tronco, de color asalmonado, discretamente descamativa y en ocasiones
ligeramente pruriginosa, llamada “madre”. Puede acompañarse de síntomas generales leves:
astenia, anorexia, odinofagia, cefalea y artralgias.
En 1-2 semanas esta lesión se torna mas escamosa y clara en el centro al tiempo que
aparecen otras de menor tamaño que se diseminan por las extremidades (áreas proximales) y
por el tronco distribuyéndose simétricamente en forma arbórea a lo largo de las hendiduras
costales. En la espalda es más evidente (“signo del árbol de Navidad”) (Stulberg DL, 2004)
La evolución de las lesiones es similar a las de la lesión “madre” desapareciendo en 5-6
semanas, aunque en algunos casos puede permanecer más tiempo y en otros verse áreas
hiper o hipo pigmentadas durante algún tiempo. La tasa de recurrencia a los 5 años es inferior
al 2% (Chuang TY, 1982).
Pitiriasis Rosada
Chuh A, Chan H, Zawar V. Pityriasis rosea--evidence for and against an infectious
aetiology. Epidemiol Infect. 2004 Jun;132(3):381-90.
Chuh AA, Chan HH. Prospective case-control study of chlamydia, legionella and
mycoplasma infections in patients with pityriasis rosea.
Chuh AA, Dofitas BL, Comisel GG, Reveiz L, Sharma V, Garner SE, Chu F.
Interventions for pityriasis rosea. Cochrane Database Syst Rev. 2007 Apr
18;(2):CD005068.
Chuh AA. Quality of life in children with pityriasis rosea: a prospective case control
study. Pediatr Dermatol. 2003 Nov-Dec;20(6):474-8.
Hsu S, Le EH, Khoshevis MR. Differential diagnosis of annular lesions. Am Fam
Physician. 2001 Jul 15;64(2):289-96] Eur J Dermatol. 2002 Mar-Apr;12(2):170-3.
Miranda SB, Lupi O, Lucas E. Vesicular pityriasis rosea: response to erythromycin treatment. J
Eur Acad Dermatol Venereol. 2004 Sep;18(5):622-5
31.- Femenino de 39 años, acude a consulta refiriendo mareo a la bipedestación.
Antecedentes: DM2 de 10 años de evolución bajo tratamiento médico. Exploración física: TA
105/60mmHg sentada y de pie 95/60mmHg, FC 76 x´, FR 16 x´. Usted solicita ES. Reportando:
Na 164 mEq/l, osmolaridad urinaria 210 mOsm/l, osmolaridad sérica 330 mOsm/l, sodio urinario
45 mOsm /l.
El diagnóstico de primera elección en este paciente es:
a) Diabetes juvenil
b) Diabetes insípida nefrogénica.
c) Diabetes insípida central.
d) Diabetes periférica
Diabetes insípida nefrogénica: Se caracteriza por la falta de respuesta tubular a la AVP, en
presencia de concentraciones normales de la misma.
Causas: alteraciones del receptor V2, uso de litio, hipercalcemia, hipopotasemia, enfermedades
renales que interfieren en la concentración de orina.
Consecuencias: Poliuria hipoosmótica (>40ml/Kg./d) (<200 mOsm/Kg.), tendencia a la
deshidratación hipertónica (285-290mOsm/Kg.), con sodio de 140-145mEq/l, exceso de sed en
personas con conciencia conservada, la ingesta de sodio es igual a la secreción
Hernando L, Nefrología Clínica, Ed. Panamericana, 3ªEdición, 2008, Pág. 42-43
32.- Mujer de 37 años. Acude a consulta por presentar cefalea, cansancio e irregularidades
menstruales con ritmo de 36 a 50 x 2-3 días. No se ha podido embarazar después de 18 meses
de actividad sexual regular. No tiene antecedentes importantes. EF: Campos visuales
normales, tiroides aumentada de tamaño una vez y aumentada de consistencia, no tiene
galactorrea. Resto normal. Laboratorio: química sanguínea, Bh y electrolitos normales.
Prolactina 47 ng/dL (< 25), perfil tiroideo: TSH 18 mUI/ml, T4t: 50 nmol/L (57.9 a 154.4), T4L:
7.7 pmol/L (9 a 24), T3T: 1.06 nmol/L (1.2 a 2.9), T3L: 1.96 pmol/L (3 a 6.31)
La etiología más frecuente de este problema es:
a) Tiroiditis autoinmune crónica
b) Microadenoma hipofisario
c) Resistencia a la insulina
d) Enfermedad de Graves
Inicialmente el hipotiroidismo se diagnosticaba mediante la cuantificación por técnicas de Radio
Inmuno Análisis (RIA) de las hormonas circulantes triyodotironina y tiroxina; el proceso era
lento y sometido a muchos factores de error que hacían su sensibilidad y especificidad poco
confiables. Posteriormente, se desarrollaron técnicas para la medición de la TSH hipofisiaria
igualmente mediante el RIA lo que mejoró en forma importante la sensibilidad para el
diagnóstico de esta enfermedad; sin embargo, los niveles de detección de la prueba se
encontraban en el orden de 1 µIU/ml lo que hacía que la prueba no fuera sensible para valores
menores de 1 µIU/ml. Debido a esto se crearon técnicas de segunda generación mediante la
cuantificación de TSH por anticuerpos monoclonales y RIA, el IRMA (Immuno Radiometric with
Monoclonal Antibodies) que permitió detectar valores de TSH en rangos de 0.1 µIU/ml;
posibilitando desde entonces diagnosticar pacientes con hipertiroidismo primario; pero con la
limitante de que para esta técnica era imposible detectar valores de TSH menores de 0.1
µIU/ml por lo que se creo la medición de TSH mediante quimioluminiscencia o métodos
enzimáticos, es decir las técnicas de tercera generación, las cuales pueden detectar valores
de TSH de 0.01 µIU/ml; con lo que se logra el espectro ideal para una prueba de laboratorio
que tiene la capacidad de diagnosticar tanto la hipofunción como la hiperfunción(20).
Además el avance no sólo fue en la medición de TSH sino también en las hormonas tiroideas
que han evolucionado simultáneamente con la TSH y ya se miden incluso las fracciones libres
de hormonas y las fracciones totales, lo que ha facilitado el manejo de estos pacientes. Gracias
a esta evolución en técnicas de laboratorio, el diagnóstico de hipotiroidismo primario es
bastante sencillo. Niveles de TSH superiores al valor máximo de la técnica serían diagnósticos
de la disfunción; pero no es tan fácil. Cuando tenemos un paciente con toda la sintomatología
del hipotiroidismo y la TSH se encuentra elevada el diagnóstico es obvio; pero podemos tener
pacientes con síntomas muy inespecíficos como depresión y con examen físico normal a
quienes se les encuentran valores de TSH por encima del límite superior y con hormonas
tiroideas normales. Se trata de un hipotiroidismo o es un valor ligeramente elevado ocasional
de una persona sana(21). Igualmente tenemos otra circunstancia que ha sido descrita con
mayor frecuencia: pacientes con valores de TSH en el límite superior normal y con dislipidemia
a quienes se les da tratamiento con hormonas tiroideas y su dislipidemia se corrige
manteniendo valores de TSH en rangos normales. Todas las circunstancias anteriores han
hecho que aparezca en el hipotiroidismo primario la expresión de hipotiroidismo subclinico, que
ha sido objeto de reuniones y congresos dedicados exclusivamente a este tema. La sociedad
Europea de Tiroides hace algunas recomendaciones para el manejo de esta situación que se
consideran útiles como guía (Tabla ).
Tabla. Enfoque del paciente con disfunción tiroidea de acuerdo a los niveles de hormona
estimulante de la tiroides (TSH).
Si TSH < 0.4m U/L
Si TSH 0.4
a 2.0 mU/L Si TSH 2.01 a 5.0mU/L
Si TSH > 5.0
mU/L
Medir T3 y T4 totales o
libres para diagnostico de
hipertiroidismo.
Normal,
Repetir
cada cinco
años
Medir T4 libre y anticuerpos
antitiroideos
Dar tratamiento
para
hipotiroidismo
1. Si AAT (-) y T4 libre es normal
repetir screening cada año. Si TSH
es > 4.0mU/l en dos ocasiones dar
tratamiento
2. Si AAT (+) y/o T4 libre esta baja
o normal baja tratar si TSH es
mayor de 3.0 mU/l y observar a los
otros
Tomado de Koutras DA. Subclinical hypothyroidism. En G. Hennemann, E.P. Krenning,
Thyroid International Merck KGaA, Darmstadt 1999 (3), 6-9
HIPOTIROIDISMO
PRIMARIO:
congénito: agenesia
autoinmune
postablativo
tiroiditis subaguda
deficiencia de yodo
SECUNDARIO
tumor hipofisiario
iatrogénico
TERCIARIO
33.- Se trata de masculino de 11 años de edad sin antecedentes de importancia para el
padecimiento actual. Inicia hace 14 días con rinorrea hialina, tos en accesos, adinofagia,
cefalea y febrícula. Es atendida por médico quién administra manejo sintomático con discreta
mejoría. Hace 5 días inicia con dolor dental,la rinorrea se torna mucopurulenta con incremento
de eventos de tos de predominio nocturno. A la exploración presenta faringe hiperémica con
descarga posterior, halitosis, narinas con moco purulento.
El diagnóstico más probable es:
a) Faringoamigdalitis bacteriana
b) Sinusitis aguda
c) Difteria
d) Faringoamigdalitis viral
SINUSITIS.
Las inflamaciones de los senos paranasales constituyen una afección frecuente, aunque
generalmente banal, y con frecuencia son una extensión de la patología de las fosas nasales.
Aproximadamente un 5 % de la población padece en algún momento una sinusitis crónica.
El seno más afecto es el maxilar, seguido del etmoides, frontal y esfenoidal: Durante el
periodo pediátrico los senos más afectados son los etmoidales.
CLASIFICACIÓN:
- Sinusitis aguda: Duración menor a 4 semanas
- Sinusitis subaguda: Duración: 4 a 12 semanas
Examen final 20460
Examen final 20460
Examen final 20460
Examen final 20460
Examen final 20460
Examen final 20460
Examen final 20460
Examen final 20460
Examen final 20460
Examen final 20460
Examen final 20460
Examen final 20460
Examen final 20460
Examen final 20460
Examen final 20460
Examen final 20460
Examen final 20460
Examen final 20460
Examen final 20460
Examen final 20460
Examen final 20460
Examen final 20460
Examen final 20460
Examen final 20460
Examen final 20460
Examen final 20460
Examen final 20460
Examen final 20460
Examen final 20460
Examen final 20460
Examen final 20460
Examen final 20460
Examen final 20460
Examen final 20460
Examen final 20460
Examen final 20460
Examen final 20460
Examen final 20460
Examen final 20460
Examen final 20460
Examen final 20460
Examen final 20460
Examen final 20460
Examen final 20460
Examen final 20460
Examen final 20460
Examen final 20460
Examen final 20460
Examen final 20460
Examen final 20460
Examen final 20460
Examen final 20460
Examen final 20460
Examen final 20460
Examen final 20460
Examen final 20460
Examen final 20460
Examen final 20460
Examen final 20460
Examen final 20460
Examen final 20460
Examen final 20460
Examen final 20460
Examen final 20460
Examen final 20460
Examen final 20460
Examen final 20460
Examen final 20460
Examen final 20460
Examen final 20460
Examen final 20460
Examen final 20460
Examen final 20460
Examen final 20460
Examen final 20460
Examen final 20460
Examen final 20460
Examen final 20460
Examen final 20460
Examen final 20460

Más contenido relacionado

La actualidad más candente

Abdomen agudo perforativo
Abdomen agudo perforativoAbdomen agudo perforativo
Abdomen agudo perforativolainskaster
 
Hernias pared abdominal 2017 (Hernia Inguinal/Hernia femoral)
Hernias pared abdominal 2017 (Hernia Inguinal/Hernia femoral)Hernias pared abdominal 2017 (Hernia Inguinal/Hernia femoral)
Hernias pared abdominal 2017 (Hernia Inguinal/Hernia femoral)Andrés Zúñiga Zapata
 
ENAM 2021 - 20 DE MARZO.pdf
ENAM 2021 - 20 DE MARZO.pdfENAM 2021 - 20 DE MARZO.pdf
ENAM 2021 - 20 DE MARZO.pdfEdward leyva
 
Abdomen agudo perforado
Abdomen agudo perforadoAbdomen agudo perforado
Abdomen agudo perforadojesus maza
 
Enterocolitis necrotizante
Enterocolitis necrotizanteEnterocolitis necrotizante
Enterocolitis necrotizanteFela Berecochea
 
Gangrena de fournier 2015
Gangrena de fournier 2015Gangrena de fournier 2015
Gangrena de fournier 2015Sofia Thomas
 
Lavado peritoneal diagnóstico
Lavado peritoneal diagnósticoLavado peritoneal diagnóstico
Lavado peritoneal diagnósticoAna Angel
 
Abdomen Agudo
Abdomen AgudoAbdomen Agudo
Abdomen AgudoFAMEN
 
CLASE DE ABDOMEN AGUDO HEMORRÁGICO
CLASE DE ABDOMEN AGUDO HEMORRÁGICOCLASE DE ABDOMEN AGUDO HEMORRÁGICO
CLASE DE ABDOMEN AGUDO HEMORRÁGICOLUIS del Rio Diez
 
Apendicitis aguda
Apendicitis agudaApendicitis aguda
Apendicitis agudaariverarodr
 

La actualidad más candente (20)

Abdomen agudo perforativo
Abdomen agudo perforativoAbdomen agudo perforativo
Abdomen agudo perforativo
 
Hernias pared abdominal 2017 (Hernia Inguinal/Hernia femoral)
Hernias pared abdominal 2017 (Hernia Inguinal/Hernia femoral)Hernias pared abdominal 2017 (Hernia Inguinal/Hernia femoral)
Hernias pared abdominal 2017 (Hernia Inguinal/Hernia femoral)
 
Colangitis caso clinico
Colangitis   caso clinicoColangitis   caso clinico
Colangitis caso clinico
 
Patologia del canal inguinal
Patologia del canal inguinalPatologia del canal inguinal
Patologia del canal inguinal
 
ENAM 2021 - 20 DE MARZO.pdf
ENAM 2021 - 20 DE MARZO.pdfENAM 2021 - 20 DE MARZO.pdf
ENAM 2021 - 20 DE MARZO.pdf
 
Abdomen agudo perforado
Abdomen agudo perforadoAbdomen agudo perforado
Abdomen agudo perforado
 
Enterocolitis necrotizante
Enterocolitis necrotizanteEnterocolitis necrotizante
Enterocolitis necrotizante
 
Preguntas pancreatitis enarm
Preguntas pancreatitis enarmPreguntas pancreatitis enarm
Preguntas pancreatitis enarm
 
Gangrena de fournier 2015
Gangrena de fournier 2015Gangrena de fournier 2015
Gangrena de fournier 2015
 
Lavado peritoneal diagnóstico
Lavado peritoneal diagnósticoLavado peritoneal diagnóstico
Lavado peritoneal diagnóstico
 
Invaginación Intestinal
Invaginación IntestinalInvaginación Intestinal
Invaginación Intestinal
 
Abdomen Agudo
Abdomen AgudoAbdomen Agudo
Abdomen Agudo
 
Preguntas enarm c.c.l.
Preguntas enarm c.c.l.Preguntas enarm c.c.l.
Preguntas enarm c.c.l.
 
Apendice
ApendiceApendice
Apendice
 
Abdomen agudo vascular
Abdomen agudo vascularAbdomen agudo vascular
Abdomen agudo vascular
 
Desgarro de Mallory Weiss
Desgarro de Mallory WeissDesgarro de Mallory Weiss
Desgarro de Mallory Weiss
 
Sesión Académica del CRAIC "Enfermedad de Kawasaki"
Sesión Académica del CRAIC "Enfermedad de Kawasaki"Sesión Académica del CRAIC "Enfermedad de Kawasaki"
Sesión Académica del CRAIC "Enfermedad de Kawasaki"
 
Absceso Hepatico
Absceso HepaticoAbsceso Hepatico
Absceso Hepatico
 
CLASE DE ABDOMEN AGUDO HEMORRÁGICO
CLASE DE ABDOMEN AGUDO HEMORRÁGICOCLASE DE ABDOMEN AGUDO HEMORRÁGICO
CLASE DE ABDOMEN AGUDO HEMORRÁGICO
 
Apendicitis aguda
Apendicitis agudaApendicitis aguda
Apendicitis aguda
 

Similar a Examen final 20460

Similar a Examen final 20460 (20)

Estenosis del piloro
Estenosis del piloroEstenosis del piloro
Estenosis del piloro
 
Linfadenitis mesenterica
Linfadenitis mesentericaLinfadenitis mesenterica
Linfadenitis mesenterica
 
DIARREA AGUDA INFECCIOSA EN PEDIATRIA
DIARREA AGUDA INFECCIOSA EN PEDIATRIADIARREA AGUDA INFECCIOSA EN PEDIATRIA
DIARREA AGUDA INFECCIOSA EN PEDIATRIA
 
Caso clínico
Caso clínicoCaso clínico
Caso clínico
 
Enterocolitis necrozante
Enterocolitis necrozanteEnterocolitis necrozante
Enterocolitis necrozante
 
E coli
E coliE coli
E coli
 
Geohelmintiasis.pptx
Geohelmintiasis.pptxGeohelmintiasis.pptx
Geohelmintiasis.pptx
 
gastroenteritis.pptx
gastroenteritis.pptxgastroenteritis.pptx
gastroenteritis.pptx
 
Diarrea Infecciosa 1
Diarrea Infecciosa 1Diarrea Infecciosa 1
Diarrea Infecciosa 1
 
Adenitis mesentérica. Presentación de un caso
Adenitis mesentérica. Presentación de un casoAdenitis mesentérica. Presentación de un caso
Adenitis mesentérica. Presentación de un caso
 
Caso completo
Caso completoCaso completo
Caso completo
 
Uncinariasis
UncinariasisUncinariasis
Uncinariasis
 
CPHAP 043 Enterocolitis Necrosante
CPHAP 043 Enterocolitis NecrosanteCPHAP 043 Enterocolitis Necrosante
CPHAP 043 Enterocolitis Necrosante
 
Gastroenteritis por e. coli equpo 1 grupo 704
Gastroenteritis por e. coli  equpo 1  grupo 704Gastroenteritis por e. coli  equpo 1  grupo 704
Gastroenteritis por e. coli equpo 1 grupo 704
 
Diarrea infecciosa-1
Diarrea infecciosa-1Diarrea infecciosa-1
Diarrea infecciosa-1
 
07_Estenosis pilorica.pdf
07_Estenosis pilorica.pdf07_Estenosis pilorica.pdf
07_Estenosis pilorica.pdf
 
Tratamiento ambulatorio de la Diarrea
Tratamiento ambulatorio de la Diarrea Tratamiento ambulatorio de la Diarrea
Tratamiento ambulatorio de la Diarrea
 
Sepsis neonatal
Sepsis neonatalSepsis neonatal
Sepsis neonatal
 
Sindrome diarreico y disenterico
Sindrome diarreico y disentericoSindrome diarreico y disenterico
Sindrome diarreico y disenterico
 
EPI.pptx
EPI.pptxEPI.pptx
EPI.pptx
 

Último

ANALGESIA Y SEDACION EN EL SERVICIO DE UNIDAD DE CUIDADOS INTENSIVOS ADULTOS
ANALGESIA Y SEDACION EN EL SERVICIO DE UNIDAD DE CUIDADOS INTENSIVOS ADULTOSANALGESIA Y SEDACION EN EL SERVICIO DE UNIDAD DE CUIDADOS INTENSIVOS ADULTOS
ANALGESIA Y SEDACION EN EL SERVICIO DE UNIDAD DE CUIDADOS INTENSIVOS ADULTOSXIMENAJULIETHCEDIELC
 
INFECCION DE TRACTO URINARIO (ITU) EN GESTANTES
INFECCION DE TRACTO URINARIO (ITU) EN GESTANTESINFECCION DE TRACTO URINARIO (ITU) EN GESTANTES
INFECCION DE TRACTO URINARIO (ITU) EN GESTANTESangelojosue
 
21542401-Historia-Natural-Del-Infarto-Agudo-de-Miocardio.pdf
21542401-Historia-Natural-Del-Infarto-Agudo-de-Miocardio.pdf21542401-Historia-Natural-Del-Infarto-Agudo-de-Miocardio.pdf
21542401-Historia-Natural-Del-Infarto-Agudo-de-Miocardio.pdfHANNIBALRAMOS
 
EJERCICIOS DE BUERGUER ALLEN FISIOTERAPIApptx
EJERCICIOS DE BUERGUER ALLEN FISIOTERAPIApptxEJERCICIOS DE BUERGUER ALLEN FISIOTERAPIApptx
EJERCICIOS DE BUERGUER ALLEN FISIOTERAPIApptxMaria969948
 
Emergencia Neumológica: Crisis asmática.pptx
Emergencia Neumológica: Crisis asmática.pptxEmergencia Neumológica: Crisis asmática.pptx
Emergencia Neumológica: Crisis asmática.pptxMediNeumo
 
CASO NEONATAL ictericia Rev MH 04.2024.pdf
CASO NEONATAL ictericia Rev MH 04.2024.pdfCASO NEONATAL ictericia Rev MH 04.2024.pdf
CASO NEONATAL ictericia Rev MH 04.2024.pdfMAHINOJOSA45
 
Dengue 2024 actualización en el tratamiento autorización de los síntomas trab...
Dengue 2024 actualización en el tratamiento autorización de los síntomas trab...Dengue 2024 actualización en el tratamiento autorización de los síntomas trab...
Dengue 2024 actualización en el tratamiento autorización de los síntomas trab...jchahua
 
Acceso venoso periferico, caracteristicas y funciones
Acceso venoso periferico, caracteristicas y funcionesAcceso venoso periferico, caracteristicas y funciones
Acceso venoso periferico, caracteristicas y funcionesDamaryHernandez5
 
Laboratorios y Estudios de Imagen _20240418_065616_0000.pdf
Laboratorios y Estudios de Imagen _20240418_065616_0000.pdfLaboratorios y Estudios de Imagen _20240418_065616_0000.pdf
Laboratorios y Estudios de Imagen _20240418_065616_0000.pdfHecmilyMendez
 
tecnicas practivas DIGITOPUNTURA SHIATZU.ppt
tecnicas practivas DIGITOPUNTURA SHIATZU.ppttecnicas practivas DIGITOPUNTURA SHIATZU.ppt
tecnicas practivas DIGITOPUNTURA SHIATZU.pptLEONCIOVASQUEZMARIN2
 
ICTERICIA INFANTIL Y NEONATAL 2024 v2.0.pdf
ICTERICIA INFANTIL Y NEONATAL 2024 v2.0.pdfICTERICIA INFANTIL Y NEONATAL 2024 v2.0.pdf
ICTERICIA INFANTIL Y NEONATAL 2024 v2.0.pdfMAHINOJOSA45
 
Asfixia por confinamiento en medicina legal.pptx
Asfixia por confinamiento en medicina legal.pptxAsfixia por confinamiento en medicina legal.pptx
Asfixia por confinamiento en medicina legal.pptxanalaurafrancomolina
 
Infecciones de la piel y partes blandas(Impétigo, celulitis, erisipela, absce...
Infecciones de la piel y partes blandas(Impétigo, celulitis, erisipela, absce...Infecciones de la piel y partes blandas(Impétigo, celulitis, erisipela, absce...
Infecciones de la piel y partes blandas(Impétigo, celulitis, erisipela, absce... Estefa RM9
 
GENERALIDADES SOBRE LA CESAREA, RESIDENCIA DE GINECOLOGIA Y OBSTETRICIA
GENERALIDADES SOBRE LA CESAREA, RESIDENCIA DE GINECOLOGIA Y OBSTETRICIAGENERALIDADES SOBRE LA CESAREA, RESIDENCIA DE GINECOLOGIA Y OBSTETRICIA
GENERALIDADES SOBRE LA CESAREA, RESIDENCIA DE GINECOLOGIA Y OBSTETRICIAYinetCastilloPea
 
PROCESO DE EXTRACCION: MACERACION DE PLANTAS.pptx
PROCESO DE EXTRACCION: MACERACION DE PLANTAS.pptxPROCESO DE EXTRACCION: MACERACION DE PLANTAS.pptx
PROCESO DE EXTRACCION: MACERACION DE PLANTAS.pptxJOSEANGELVILLALONGAG
 
Como se produjo la Penicilina de forma massiva en la II Guerra Mundial.pdf
Como se produjo la Penicilina de forma massiva en la II Guerra Mundial.pdfComo se produjo la Penicilina de forma massiva en la II Guerra Mundial.pdf
Como se produjo la Penicilina de forma massiva en la II Guerra Mundial.pdfKEVINYOICIAQUINOSORI
 
Trabajo de parto y mecanismos de trabajo de parto.pdf
Trabajo de parto y mecanismos de trabajo de parto.pdfTrabajo de parto y mecanismos de trabajo de parto.pdf
Trabajo de parto y mecanismos de trabajo de parto.pdfLizbehPrez1
 
Alcohol etílico bioquimica, fisiopatologia
Alcohol etílico bioquimica, fisiopatologiaAlcohol etílico bioquimica, fisiopatologia
Alcohol etílico bioquimica, fisiopatologiassuser76dfc8
 
"La auto-regulación como concepto esencial para la seguridad de la praxis clí...
"La auto-regulación como concepto esencial para la seguridad de la praxis clí..."La auto-regulación como concepto esencial para la seguridad de la praxis clí...
"La auto-regulación como concepto esencial para la seguridad de la praxis clí...Badalona Serveis Assistencials
 

Último (20)

ANALGESIA Y SEDACION EN EL SERVICIO DE UNIDAD DE CUIDADOS INTENSIVOS ADULTOS
ANALGESIA Y SEDACION EN EL SERVICIO DE UNIDAD DE CUIDADOS INTENSIVOS ADULTOSANALGESIA Y SEDACION EN EL SERVICIO DE UNIDAD DE CUIDADOS INTENSIVOS ADULTOS
ANALGESIA Y SEDACION EN EL SERVICIO DE UNIDAD DE CUIDADOS INTENSIVOS ADULTOS
 
INFECCION DE TRACTO URINARIO (ITU) EN GESTANTES
INFECCION DE TRACTO URINARIO (ITU) EN GESTANTESINFECCION DE TRACTO URINARIO (ITU) EN GESTANTES
INFECCION DE TRACTO URINARIO (ITU) EN GESTANTES
 
21542401-Historia-Natural-Del-Infarto-Agudo-de-Miocardio.pdf
21542401-Historia-Natural-Del-Infarto-Agudo-de-Miocardio.pdf21542401-Historia-Natural-Del-Infarto-Agudo-de-Miocardio.pdf
21542401-Historia-Natural-Del-Infarto-Agudo-de-Miocardio.pdf
 
EJERCICIOS DE BUERGUER ALLEN FISIOTERAPIApptx
EJERCICIOS DE BUERGUER ALLEN FISIOTERAPIApptxEJERCICIOS DE BUERGUER ALLEN FISIOTERAPIApptx
EJERCICIOS DE BUERGUER ALLEN FISIOTERAPIApptx
 
Emergencia Neumológica: Crisis asmática.pptx
Emergencia Neumológica: Crisis asmática.pptxEmergencia Neumológica: Crisis asmática.pptx
Emergencia Neumológica: Crisis asmática.pptx
 
CASO NEONATAL ictericia Rev MH 04.2024.pdf
CASO NEONATAL ictericia Rev MH 04.2024.pdfCASO NEONATAL ictericia Rev MH 04.2024.pdf
CASO NEONATAL ictericia Rev MH 04.2024.pdf
 
Situaciones difíciles. La familia reconstituida
Situaciones difíciles. La familia reconstituidaSituaciones difíciles. La familia reconstituida
Situaciones difíciles. La familia reconstituida
 
Dengue 2024 actualización en el tratamiento autorización de los síntomas trab...
Dengue 2024 actualización en el tratamiento autorización de los síntomas trab...Dengue 2024 actualización en el tratamiento autorización de los síntomas trab...
Dengue 2024 actualización en el tratamiento autorización de los síntomas trab...
 
Acceso venoso periferico, caracteristicas y funciones
Acceso venoso periferico, caracteristicas y funcionesAcceso venoso periferico, caracteristicas y funciones
Acceso venoso periferico, caracteristicas y funciones
 
Laboratorios y Estudios de Imagen _20240418_065616_0000.pdf
Laboratorios y Estudios de Imagen _20240418_065616_0000.pdfLaboratorios y Estudios de Imagen _20240418_065616_0000.pdf
Laboratorios y Estudios de Imagen _20240418_065616_0000.pdf
 
tecnicas practivas DIGITOPUNTURA SHIATZU.ppt
tecnicas practivas DIGITOPUNTURA SHIATZU.ppttecnicas practivas DIGITOPUNTURA SHIATZU.ppt
tecnicas practivas DIGITOPUNTURA SHIATZU.ppt
 
ICTERICIA INFANTIL Y NEONATAL 2024 v2.0.pdf
ICTERICIA INFANTIL Y NEONATAL 2024 v2.0.pdfICTERICIA INFANTIL Y NEONATAL 2024 v2.0.pdf
ICTERICIA INFANTIL Y NEONATAL 2024 v2.0.pdf
 
Asfixia por confinamiento en medicina legal.pptx
Asfixia por confinamiento en medicina legal.pptxAsfixia por confinamiento en medicina legal.pptx
Asfixia por confinamiento en medicina legal.pptx
 
Infecciones de la piel y partes blandas(Impétigo, celulitis, erisipela, absce...
Infecciones de la piel y partes blandas(Impétigo, celulitis, erisipela, absce...Infecciones de la piel y partes blandas(Impétigo, celulitis, erisipela, absce...
Infecciones de la piel y partes blandas(Impétigo, celulitis, erisipela, absce...
 
GENERALIDADES SOBRE LA CESAREA, RESIDENCIA DE GINECOLOGIA Y OBSTETRICIA
GENERALIDADES SOBRE LA CESAREA, RESIDENCIA DE GINECOLOGIA Y OBSTETRICIAGENERALIDADES SOBRE LA CESAREA, RESIDENCIA DE GINECOLOGIA Y OBSTETRICIA
GENERALIDADES SOBRE LA CESAREA, RESIDENCIA DE GINECOLOGIA Y OBSTETRICIA
 
PROCESO DE EXTRACCION: MACERACION DE PLANTAS.pptx
PROCESO DE EXTRACCION: MACERACION DE PLANTAS.pptxPROCESO DE EXTRACCION: MACERACION DE PLANTAS.pptx
PROCESO DE EXTRACCION: MACERACION DE PLANTAS.pptx
 
Como se produjo la Penicilina de forma massiva en la II Guerra Mundial.pdf
Como se produjo la Penicilina de forma massiva en la II Guerra Mundial.pdfComo se produjo la Penicilina de forma massiva en la II Guerra Mundial.pdf
Como se produjo la Penicilina de forma massiva en la II Guerra Mundial.pdf
 
Trabajo de parto y mecanismos de trabajo de parto.pdf
Trabajo de parto y mecanismos de trabajo de parto.pdfTrabajo de parto y mecanismos de trabajo de parto.pdf
Trabajo de parto y mecanismos de trabajo de parto.pdf
 
Alcohol etílico bioquimica, fisiopatologia
Alcohol etílico bioquimica, fisiopatologiaAlcohol etílico bioquimica, fisiopatologia
Alcohol etílico bioquimica, fisiopatologia
 
"La auto-regulación como concepto esencial para la seguridad de la praxis clí...
"La auto-regulación como concepto esencial para la seguridad de la praxis clí..."La auto-regulación como concepto esencial para la seguridad de la praxis clí...
"La auto-regulación como concepto esencial para la seguridad de la praxis clí...
 

Examen final 20460

  • 1. Universidad La Salle. Facultad Mexicana de Medicina. Curso de Extensión Universitaria para la Preparación del Examen Nacional para Aspirantes a Residencias Médicas. Examen Final 2016 1.- Se trata de masculino de 9 meses, ingresa al servicio de urgencias por presentar en 24 hrs., 7 evacuaciones diarreicas, sin sangre, fiebre no cuantificada y vómito 4 veces. La madre refiere que en las últimas horas el menor está hiporéxico pero bebe con avidez. E.F. : Inquieto, fontanela anterior deprimida, ojos hundidos, llanto sin lágrimas, lienzo húmedo negativo, pulsos normales, llenado capilar de 3 segundos. El agente causal más probable en éste paciente es: a) Parasito. b) Virus. c) Bacteria. d) Toxina. Causada por un agente infeccioso Es el tipo más frecuente. La etiología del 70% de los casos de diarrea infecciosa es la viral.4 Todos los años, las causas principales de gastroenteritis infantil son los Rotavirus, causantes de 600.000-800.000 muertes en todo el mundo. El virus infecta los enterocitos del intestino, disminuye la actividad de las enzimas que digieren los azúcares, y disminuye la reabsorción del ión Na+ y del agua en el intestino. Además producen activación del sistema nervioso entérico y la secreción de iones Cl-. Todo ello produce un exceso de fluidos en la luz intestinal, que tiene como consecuencia una diarrea acuosa. Otros agentes etiológicos virales son los Norovirus, que ejercen una acción directa sobre la actividad de las enzimas de los enterocitos. Gastroenteritis Virales Agentes etiológicos Reovirus (Rotavirus) Calicivirus (Agente Norwalk) Adenovirus (entéricos) Astrovirus Otros: Torovirus, Picornavirus, Picornabirnavirus Causas de Diarrea Infantil Aguda9 Acuosa Con sangre (Disentérica) Secretora Osmótica Invasiva No invasiva
  • 2. Rotavirus Vibrio cholerae ECET Vibrios no cólera Shigella Virus G. lamblia Cryptosporidium Laxantes Desnutrición Shigella ECEI Salmonella no tifoidea Campylobacter jejun Yersinia E. coli O157:H7 ECEP Clostridium difficile 2.- Se trata de femenino de 6 años ingresa al servicio de urgencias, refiere la madre que la menor ha presentado tos, ardor retroesternal y silbido del pecho. . Tiene tos irritativa, no productiva, hipertermia no cuantificada. Antecedentes: Originaria de Veracruz, nivel socioeconómico bajo, habita en un rancho. Malos hábitos de higiene. Exploración física: febril 39 °c, con dificultad para respirar, aleteo nasal, tiraje intercostal y subcostal. Laboratorio: eosinofilia marcada. Radiografía de tórax muestra infiltrados redondos u ovalados de unos milímetros hasta varios cm. de diámetro, bilateral. El tratamiento de elección en éste síndrome es: a) TMP/SMZ 800mg / 160MG c/12 hrs b) Amikacina 30 mg. por kg. de peso para 24 hrs. c) Albendazol 400 mg dosis unica. d) Clindamicina 300 mg v.0. cada 8 hrs
  • 3. Tratamiento. Mebendazol (100 mg/12 h/3 dias o 500 mg a dosis unica). Alternativas: pamoato de pyrantel, 10 mg/kg (maximo 1 g), dosis unica; albendazol, 400 mg, dosis unica (ninos entre 2-5 anos: 200 mg); piperazina, 75 mg/kg (maximo 3,5 g en adultos o 2,5 g en Niños entre 2-12 años). La ivermectina tambien es eficaz. (200 μg/kg dosis unica). Parasitosis intestinales J. Gascón Brustenga y J. Muñoz Gutiérrez 3.- Se trata de recién nacido pretermino de 30 semanas de gestación, peso de 1200 gr. Cursó con enfermedad de membranas hialinas por lo que se manejo con una dosis de surfactante y manejo con ventilación convencional por 3 días. Posteriormente inició con alimentación enteral por sonda orogástrica con leche humana, sin tolerancia a la misma (residuo gástrico y distensión abdominal), sangre oculta en heces e ictericia. El diagnóstico más probable es: a) Alergia a las proteínas de la leche b) Enterocolitis necrotizante c) Atresia intestinal d) Gastroenteritis infecciosa
  • 4. La enterocolitis necrotizante (ECN) es una enfermedad grave que afecta a recién nacidos, en especial prematuros, con una incidencia y morbimortalidad elevados. Constituye la urgencia gastrointestinal más frecuente en las UCI neonatales. Se presenta como un síndrome gastrointestinal y sistémico que comprende síntomas variados y variables, como distensión e hipersensibilidad abdominal, sangre en heces, intolerancia a la alimentación, apnea, letargia, y en casos avanzados acidosis, sepsis, CID y shock. El síndrome clínico ha sido clasificado en estadios por Bell y col. (1978) y modificado por Walsh y Klegman (1986) para incluir hallazgos sistémicos, intestinales y radiológicos.  A. Estadio I : sospecha de enterocolitis necrotizante  Los hallazgos sistémicos son inespecíficos.  Los hallazgos intestinales incluye el residuo gástrico y heces guayaco –positivas.  Los hallazgos radiológicos son normales e inespecíficos.  B. Estadio II A: enterocolitis necrotizante leve  Los hallazgos sistémicos son similares al estadio I.  Los hallazgos intestinales incluyen distensión abdominal prominente con hipersensibilidad a la palpación o sin ella, ruidos hidroaéreos ausentes, sangre macroscópica en materia fecal.  Los hallazgos radiológicos, íleo con asas dilatadas con áreas focales de neumatosis intestinal.  C. Estadio II B: enterocolitis necrotizante moderada  Los hallazgos sistémicos incluyen acidosis leve y trombocitopenia  Los hallazgos intestinales incluyen edema de la pared abdominal e hipersensibilidad a la palpación con una masa palpable o sin ella.  Los hallazgos radiológicos incluyen neumatosis extensa y ascitis temprana.  Puede haber gas en la vena porta intrahepática.  D. Estadio IIIA: enterocolitis necrotizante avanzada:  Los hallazgos sistémicos incluyen acidosis respiratoria y metabólica, ventilación asistida por apnea, hipotensión arterial, oliguria, neutropenia y coagulación intravascular diseminada.  Los hallazgos intestinales incluyen edema que disemina, eritema e induración del abdomen.  Los hallazgos radiológicos incluyen ascitis prominente y asa centinela persistente sin perforación.  E. Estadio IIIB: enterocolitis necrotizante avanzada:  Los hallazgos sistémicos revelan signos vitales e índices de laboratorio en deterioro, síndrome de shock y desequilibrio electrolítico.  Los hallazgos intestinales y radiológicos muestran evidencias de perforación. Hallazgos radiológicos en la radiografía de abdomen  Distensión abdominal generalizada  Íleo paralítico  Neumatosis intestinal quistoide (patognomónico) La neumatosis intestinal quistoide se la puede visualizar de varias maneras, cúmulos de gas lineales, curvilíneos, esponjosos y espumosos.
  • 5. Este último debe distinguirse de materia fecal o meconio mezclado con aire. Cualquier lactante con sospecha de enterocolitis necrotizante en el que se encuentren radiográficamente colecciones de aire lineales, curvilíneas esponjosas o espumosas debe considerarse que tiene neumatosis intestinal quistoide hasta que se demuestre lo contrario. La neumatosis intestinal quistoide suele verse con mayor frecuencia en el colon, pero puede ocurrir desde estomago hasta recto. 1. Hartmann G. E., Drugas G. T., Shochat S. J. Post-necrotizing enterocolitis strictures presenting with sepsis of perforation: risk of clinical observation. J. Pediatr. Surg. 1988; 23: 562-6. 2. Kosloske A. M., Burstein J., Bartow S. A. Intestinal obstruction due to colonic stricture following neonatal necrotizing enterocolitis. Ann Surg. 1980 Aug;192 (2): 202-7. 3. Schwartz M. Z., Hayden C. K., Richardson C. J., Tyson K. R., Lobe T. E. A prospective evaluation of intestinal stenosis following necrotizing enterocolitis. J. Pediatr. Surg. 1982 Dec; 17 (6): 764-70. 4. Bell M. J., Ternberg J. L., Askin F. B. Intestinal stricture in necroting enterocolitis. J. Pediatr. Surg. 1976; 11: 319-27. 5. Pokorny W. J., Harr V. L., McGill, C. W., et al; Intestinal stenosis resulting from necrotizing enterocolitis. Am J. Surg 1981 42: 721-724. 6. Schimpl G., Hollwarth M. E., Fotter R., Becker H. Late intestinal strictures following successful treatment of necrotizing enterocolitis. Acta Paediatr. Suppl. 1994; 396: 80-3. 7. Bütter A., Flageole H., Laberge J. M. The Changing face of Surgical Indication for Necrotizing Enterocolitis J. Pediatr. Surg. 2002; 37: 469- 499. Gobet R. , Sacher P. , Schwobel M. G. Surgical procedures in colonic strictures after necrotizing enterocolitis. Acta Paediatr. Suppl. 1994;396:77-9. 4.- Ingresa al servicio de urgencias paciente masculino de 2 años que presenta letargo después de haber estado jugando sin vigilancia. El niño hasta entonces estaba sano, con un desarrollo normal. En la historia familiar aparece como dato importante que uno de sus hermanos tiene epilepsia y toma fenobarbital. La exploración revela un niño letárgico sin fiebre que abre brevemente los ojos ante estímulos dolorosos. No hay signos de traumatismo ni anomalías focales. Las pupilas son pequeñas pero reactivas. La etiología más probable relacionada al letargo del niño es: a) Convulsiones no observadas. b) Intoxicación. c) Hemorragia intracraneal. d) Traumatismo craneoencefálico no observado. . La causa más común de inicio agudo de somnolencia en un niño que empieza a caminar, es la intoxicación. La presencia de agentes tóxicos potenciales en la casa es importante en la historia clínica. La miosis también sugiere intoxicación y la falta de signos de traumatismo o anomalías focales hace que una hemorragia sea poco probable. LECTURA RECOMENDADA PROGRAMA DE ACTUALIZACION CONTINUA EN PEDIATRIA INTOXICACIONES EN PEDIATRÍA Dr. Miguel Angel Montoya Cabrera
  • 6. Jefe del Departamento de Admisión Continua y Toxicología, Hospital de Pediatría, Centro Médico Nacional Siglo XXI, Instituto Mexicano del Seguro Social. Miembro de la Academia Nacional de Medicina, de la Academia Mexicana de Cirugía y de la Academia Mexicana de Pediatría 5.- Femenino de 8 meses refiere la madre que ha presentado cuadros frecuentes de vómitos, movimientos incordinados y crisis convulsivas de difícil control, se le ha puesto el pelo más claro que al resto de sus hermanos, piel seborreica, microcefalia, maxilar prominente, dientes espaciados y retraso en el crecimiento. El diagnóstico más probable de éste paciente es: a) Citrulinemia b) Homocistinuria c) Fenilcetonuria d) Enfermedad de Jarabe de Maple. La fenilcetonuria, es un tipo de hiperfenilalaninemia, también conocida como PKU, es una alteración del metabolismo en el que el organismo no puede metabolizar el aminoácido tirosina a partir de fenilalanina en el hígado. Esta enfermedad es genética y es provocada por la carencia de enzima fenilalanina hidroxilasa . La fenilcetonuria tiene como rasgo principal la herencia genética autosómica recesiva, es decir, los padres son portadores de los genes defectuosos y al ser traspasados de ambos progenitores, la enfermedad se expresa en los descendientes. La causa de la enfermedad es la carencia de la enzima fenilalanina hidroxilasa (FAOH) o de la dihidropterina reductasa (DHPR) (también llamada tirosina hidroxilasa). Ambas enzimas son responsables de la hidroxilación del aminoácido fenilalanina en la reacción que produce tirosina. Por ello, el defecto o falta de alguna de ellas determina un incremento de la concentración sanguínea de fenilalanina al impedirse su transformación en tirosina. También se aumenta la transaminación de la fenilalanina como vía metabólica alternativa, y asimismo se acumulan los metabolitos fenilpiruvato, fenilactato y fenilacetato. El defecto en la síntesis de FAOH se debe a una anomalía génica localizada en el cromosoma 12, y el de la DHPR en el cromosoma 4. Existen también formas de la enfermedad con déficits parciales. El fenilpiruvato es un neurotori que afecta gravemente al cerebro durante el crecimiento y el desarrollo. Los efectos de su acumulación causan oligofrenia fenilpirúvica, caracterizada por un cociente intelectual inferior a 20. Los primeros meses de vida, los niños que padecen esta enfermedad parecen estar sanos. Entre los tres y los seis meses pierden el interés por el entorno, y al año se evidencia un retraso importante en su desarrollo. Los síntomas suelen ser retraso psicomotor, cuadros psicóticos de tipo autista, convulsiones, síndrome de West, convulsiones generalizadas y un eczema facial muy rebelde. Por lo general su desarrollo físico es bueno, tienden a tener el cabello más claro que sus hermanos, piel clara, y presentan un olor característico a paja mojada. Cuadro clínico La enfermedad se manifiesta, por primera vez, algunas semanas después del nacimiento, iniciándose con una elevación en el plasma de la fenilalanina hasta un nivel 30 veces superior al normal y por la excreción de ácido fenilpirúvico por la orina. Transcurridos 6 meses se hace patente el retraso del desarrollo mental. La mayor parte de los pacientes son deficientes graves o profundos y en ocasiones se alcanza la deficiencia media.
  • 7. El portador de esta anomalía, que nace tras un embarazo normal y sin complicaciones, se desarrolla durante los primeros meses casi siempre sin mostrar anormalidad ninguna. Sin embargo Partington encontró, casi en la mitad de los lactantes, la existencia de vómitos en los primeros meses de vida y en un tercio de ellos una irritabilidad inacostumbrada. En una proporción similar de casos, a los padres ya les había llamado la atención un desagradable olor del cuerpo del niño. Una parte de ellos mostró dermatosis eczematiformes durante el primer trimestre, 7 de 36 ya había tenido en el primer año de vida ataques convulsivos. A los 9 meses llama la atención el retraso en el desarrollo psicomotor. Datos físicos El desarrollo corporal cursa casi con normalidad. No obstante puede comprobarse cierta tendencia al enanismo, aunque también se han descrito casos con tallas superior a la frecuente. La dentición suele retrasarse hasta después del undécimo mes. La gran mayoría de los enfermos muestran una piel clara, ojos azules, y color claro del pelo. Alrededor del 10% poseen cabellos oscuros. La pobreza de pigmentos llama más la atención en los pueblos de cabellos oscuros. La piel de los portadores además de ser clara es muy suave aterciopelada y muy sensible. En algunos enfermos se han observado eflorescencias papulosas en las caras de extensión de las extremidades y en la faz. En ciertos pacientes se puede encontrar también una tendencia a la acrocianosis. Datos conductuales Características clínicas raras  Cifosis.  Pies planos.  Espina bifida.  Sindactilia en los dedos de los pies.  Bloqueo cardiaco intraventricular.  Hipogenitalismo.  Dermografismo.  Sensibilidad a la luz.  Hipersegmentación de las células neutrófilas de la sangre.  Disminución de la tolerancia a la galactosa.  Metabolismo basal ligeramente elevado. Las etapas del desarrollo habitual, la edad en la que el niño se sienta y habla, a veces, se alcanzan a la edad normal, pero, de ordinario, se retrasa. En la edad límite en que debe esperarse que el niño normalmente realice estos actos, el 35% no puede andar y el 63% no puede hablar. Estos niños, en general, tienen un peso y talla promedio por debajo del correspondiente a su edad. En la mitad de los casos tiene microcefalia y prominencia del maxilar. Sus movimientos son lentos y patosos y a menudo suelen adoptar la posición de sastre. Las anomalías del tono muscular que contribuyen a estos cambios son de origen neurológico.2 de cada 3 pacientes tienen hiperreflexia tediciosa e hipercinesia sobreañadida estos últimos son voluntarios y muy variados Referencias 1. Scriver CR, Kaufman S. 2001. Hyperphenylalaninemia: phenylalanine hydroxylase deficiency. In Scriver CR, Beaudet AL, Valle D, Sly WS (eds). The Metabolic and Molecular bases of Inherited Disease. McGraw-Hill., pp
  • 8. 1667-1724. 2. Martínez-Pardo M, Marchante C. et al: 1998. ”Protocolo de diagnóstico, tratamiento y seguimiento de las hiperfenilalaninemias”. An. Esp. Pediatría suplemento 114: 3-8. 10 3. Belanger-Quintana A, Morais A, Mena MA, Martínez- Pardo M. 2004. Niveles de neurotransmisores dopaminérgicos y serotoninérgicos en líquido cefalorraquídeo en niños. An Esp Pediatr 60:82. 6.- Femenino de 39 años de edad la cual inicia con un cuadro de exoftalmos axial de varias semanas de evolución, con predominio en su ojo derecho. En la exploración se aprecia una conjuntiva con síntomas discretos de hiperemia y edema, una queratitis de carácter punteado en tercio inferior corneal y se sospecha una retracciónpalpebral al observar cómo el borde del párpado superior se encuentra por encima del limbo, permitiéndonos visualizar la esclerótica. La paciente no refiere diminución de visión ni alteraciones tipo visión doble y toma presión intraocular con parámetros dentro de los normales. De los siguientes el diagnóstico más probable es: a) Tumor intraorbitario. b) Enfermedad de Graves-Basedow. c) Queratoconjuntivitis epidémica. d) Tumor intraocular. OFTALMOPATÍA TIROIDEA. Es la causa más frecuente de exoftalmos tanto bilateral como unilateral en adultos. La forma típica aparece en pacientes con enfermedad de Graves-Basedow, en los que podemos encontrar exoftalmos y síndrome palpebro-retráctil. Este síndrome ocular puede aparecer en pacientes eutiroideos o hipotiroideos, pudiendo constituir el signo más precoz de una tirotoxicosis incipiente. CLÍNICA. Se distinguen dos formas clínicas: a) Tirotóxica (hipersensibilidad a las catecolaminas y habitualmente hay hipertiroidismo): exoftalmos moderado depresible. Hay edema del contenido orbitario, pero no fibrosis ni oftalmoparesia. b) Maligna (inflamación orbitaria autoinmune y puede haber normo o hipotirodismo): exoftalmos irreductible severo con oftalmopejia progresiva y queratitis por exposición. Puede afectar al nervio óptico por compresión y producir pérdida visual. Hay fibrosis de la grasa y vientres musculares. Manifestaciones oculares asociadas: retracción palpebral bilateral que permite ver la esclera por encima del limbo, disminución de la motilidad palpebral, alteración de la pigmentación palpebral, hiperemia conjuntival. DIAGNÓSTICO. Por los signos clínicos descritos y exploraciones complementarias, como la exoftalmometría (medida de la protrusión ocular), la radiología (aumento de densidad de los tejidos blandos), el engrosamiento del vientre de algunos músculos extraoculares (apreciados en la TC, la RM y la ecografía orbitaria) y la analítica sistémica
  • 9. AACE Thyroid Task Force. American Association of Clinical Endocrinologists medical guidelines for clinical practice fo rthe evaluation and treatment of hyperthyroidism and hypothyroidism. Endocr Pract. 2002;8(6). Davies TF, Larsen PR. Thyrotoxicosis. In: Kronenberg HM, Melmed S, Polonsky KS, Larsen PR, eds. Williams Textbook of Endocrinology. 11th ed. Philadelphia, Pa: Saunders Elsevier; 2008:chap 11. 7.- Masculino de 5 años es llevado al servicio de urgencias por dolor súbito hiperemia, y aumento de volumen en región interna de órbita derecha. Exploración Física: Se palpa masa bien delimitada dolorosa en el borde interno de la órbita. La medida terapéutica inicial en este paciente es: a) Sólo drenaje. b) Dicloxacilina y drenaje. c) Tobramicina tópico y drenaje. d) Extirpación de saco lagrimal El tratamiento debe iniciarse antes de que se identifique el microorganismo causal. Tan pronto se obtienen cultivos de nariz, conjuntivas y sangre deben administrarse antibióticos. La terapéutica antibiótica inicial debe cubrir estafilococos, H influenzae y anaerobios. La mayor parte de los casos responde a los antibióticos. Aquellos en los cuales esto no es así, pueden requerir drenaje. Riordan P,Witcher j, Oftalmología general de Vaughan y Asbury, 17ª Ediciión, Pág. 262 8.- Paciente diagnosticado con esclerosis múltiple (considerada como un proceso inflamatorio y desmielinizante del SNC). En el diagnóstico de esta enfermedad es muy útil la presencia de: a) Más de 100 liníocitos por microlitro en el líquido cefalorraquídeo. b) Más de 50 polimorfonucleares por microlitro en el líquido cefalorraquídeo. c) Elevaciones del ácido úrico en plasma. d) Bandas oligoclonales en el líquido cefalorraquídeo. La esclerosis múltiple (EM) es una enfermedad desmielinizante del sistema tema nervioso central, que habitualmente se caracteriza por ataques recurrentes de disfunción neurológica focal y multifocal. Los síntomas y signos clásicos de EM dependen de la localización del foco de desmielinización y pueden incluir una variedad de disfunciones que no son específicas para la EM, tales como alteración de la visión, ataxia y temblor intencional, debilidad o parálisis de una o más extremidades, espasticidad y problemas vesicales. Hay,sin embargo, tres anormalidades, altamente sugestivas de EM:  Neuritis óptica, la cual es el síntoma inicial en cerca del 25% de los pacientes
  • 10.  Oftalmoplejia internuclear que se asocia con nistagmus monoocular  Signo de Lhermitte, que es una sensación "eléctrica" por detrás del cuello y de la espalda que llega hasta las piernas (al flexionar el cuello). En fibras nerviosas normales mielinizadas, la conducción ocurre de manera saltatoria; las corrientes de acción se confinan a las secciones no mielinizadas del axón saltando de un nódulo de Ranvier al siguiente. Este tipo de conducción es mucho más eficiente desde el punto de vista energético que la transmisión a través de la superficie del axón entero, aumentando así la velocidad de conducción con una pérdida mínima de energía. La desmielinización de una porción de la fibra nerviosa normalmente mielinizada, puede conducir a: bloqueo de la conducción en el sitio de la pérdida de mielina, disminución de la velocidad de conducción nerviosa a través de la fibra alterada y fatiga subjetiva aumentada o un mayor consumo de energía. La disfunción neurológica observada en la EM es una reflexión de la alteración de la conducción a través de segmentos parcial o completamente desmielinizados o de una fibra nerviosa mielínica. Además, el hecho de que el tiempo de conducción a través de los segmentos desmielinizados disminuye con aumento de la temperatura puede explicar por qué los síntomas clínicos de EM se empeoran al aumentar la temperatura corporal. Diagnóstico El diagnóstico de EM se basa fundamentalmente en dos parámetros. Por un lado los hallazgos en los estudios de imagen por Resonancia Magnética en los que se presentan (en las imágenes dependientes de T2) áreas hiperintensas de localización fundamentalmente periventricular en la substancia blanca subcortical en múltiples localizaciones así como también en la médula espinal; este estudio permite visualizar no solamente la localización de las áreas desmielinizantes sino también en forma seriada pueden realizarse evaluaciones para valorar la evolución clínica y subclínica de la enfermedad. Los otros estudios básicos para el diagnóstico son la determinación de bandas oligoclonales, la determinación de la proteína básica de la mielina y la medición de las inmunoglobulinas en el líquido cefalorraquídeo. Estos dos factores, en conjunto con la información clínica, permiten establecer con ciertas bases de seguridad el diagnóstico de esta enfermedad. Referencias bibliográficas: 1. Noseworthy JH, Lucchinetti C, Rodríguez M, Weinshenker BJ. Multiple sclerosis. N. England J Med. 2000;343:938-52. 2. Omari KM, John GR, Sealfon SC, Raine CS. CXC chemokine receptors on human oligodendrocytes: implications for multiple sclerosis. Brain 2005;128:1003-15. 3. Mi S, Miller RH, Lee X, et al. LINGO-1 negatively regulates myelination by oligodendrocytes. Nat Neurosci. 2005;8:745-51. 4. John GR, Shankar SL, Shafit-Zagardo B, et al . Multiple sclerosis: re-expression of a developmental pathway that restricts remyelination. Nat Med. 2002;8:1115-21. 5. Lucchinetti C. The spectrum of idiopathic inflammatory demyelinanting disease. In: American Academy of Neurology. Syllabi on CD ROM. 2000. 6. Hartung HP, Grossman RI. ADEM: distinct disease or part of the MS spectrum? Neurology 2001; 56:1257-60. 7. Capello E, Voskuhl RR, and McFarland HF, Raine CS. Multiple sclerosis: re-expression of a developmental gene in chronic lesions correlates with remyelination. Ann Neurol. 1997; 41:797-805. 8. Atlanta. Georgia. AAN. 115 Anual Meeting. Natural Course of Multiple Sclerosis Redefined: National Inst of Neurological Disorder and Stroke, 16, 1990. 9. Hemmer B, Archelos JJ, Hartung HP. New concepts in the inmunopathogenesis of MS. Nat Rev Neurosci. 2002;3:291-301.
  • 11. Kurtzke JF. Geography in multiple sclerosis. J Neurol. 1977;215:1-26. 9.- Una mujer de 66 años presenta deterioro al caminar. En la exploración se encuentra marcha ligeramente espástica, mala posición y sensación de vibración en los dedos de los pies, reflejos de estiramiento muscular en las rodillas +++ y reflejos aquíleos ausentes. De los siguientes diagnósticos el más probable ES: a) Hipovitaminosis B12 b) Esclerosis múltiple c) Hidrocefalia con presión normal (NPH) d) Infección con virus linfotrópico humano de células T tipo I (HTLV-I) La hipovitaminosis B12 causa degeneración combinada de sistemas. Los pacientes presentan alteración de la marcha caracterizada por espasticidad y disminución de la sensación vibratoria y la posición. Puede haber neuropatía leve que produce depresión de los reflejos aquíleos. Como es una causa tratable de marcha anormal, es importante identificar la deficiencia de vitan B12. La esclerosis múltiple (MS) no se manifiesta en este grupo de edad. La ausencia de reflejos aquíleos es un dato común de MS; más bien, la hiperactividad de los reflejos concuerda con los datos de neurona motora superior encontrados a menudo. La hidrocefalia de presión normal (NPH) es una causa de deterioro de la marcha. Sin embargo, hay donación cognitiva y los pacientes pueden tener incontinencia urinaria. La alteración de la vibración y la sensación de posición, así como la ausencia de reflejos aquíleos, no son característicos de NPH. La infección virus linfotrópico de células T humano tipo I (HTLV-I) es causa de mielopatía y debe sospecharse en pacientes que han recibido transfusiones de sangre, usuarios de drogas intravenosas o personas que hayan habitado en áreas endémicas. La adrenomieloneuropatía es un trastorno recesivo ligado a X que guarda relación con adrenoleucodistrofia, la cual típicamente se presenta en varones jóvenes. El trastorno origina la acumulación de ácidos grasos de cadena muy larga debido a una imposibilidad para catabolizar estos lípidos. Las mujeres portadoras manifiestan una paraparesia espástica leve, pero no la gama de datos que se encuentran en esta paciente. BIBLIOGRAFíA: Braunwald, Isselbachre, Petersdorf, Wilson, Martin, Fauci “HARRISON: PRINCIPIOS DE MEDICINA INTERNA” Undécima edición D.S.Mc Laren, M.Frigg 10.- Paciente de 79 años que consulta por una mácula pigmentada heterocroma, de contorno irregular, de unos 2 x 3 cm de diámetro, localizada en mejilla derecha, que ha experimentado un crecimiento muy lento en los últimos años. El diagnóstico más probable de este paciente es: a) Melanoma lentiginoso acral. b) Carcinoma Basocelular pigmentado. c) Eritema fijo pigmentario. d) Lentigo maligno.
  • 12. El lentigo maligno puede definirse como una mácula pigmentada extensa localizada en una zona de la piel expuesta al daño actínico, habitualmente desarrollada en ancianos y que consiste en una proliferación de melanocitos atípicos situados sobre una epidermis atrófica. La importancia del lentigo maligno reside en que puede ser el sustrato de un melanoma maligno. El lentigo maligno también ha sido denominado, lentigo de Hutchinson, lentigo senil, melanosis precancerosa circunscrita, lentigo “malin des viellards ”, melanosis premaligna, melanosis circunscrita preblastomatosa y melanocitoma no nevoide precanceroso. El lentigo maligno puede ser considerado como la única variedad de displasia melanocítica intraepidérmica capaz de evolucionar a un melanoma invasivo. El porcentaje de melanomas que se suponen derivados de un lentigo maligno es pequeño, 2.5 a 5%. El lentigo maligno es fácilmente reconocible y habitualmente tratable en las fases más precoces de su desarrollo. El lentigo maligno fue descrito en 1892 por Hutchinson. Dubreuilh remarcó su naturaleza precancerosa en 1894. Mishima determinó que el precursor histogénico del lentigo maligno es diferente al precursor del nevus juncional descrito por Allen y Spitz. La histopatología macroscópica y microscópica del lentigo maligno fue documentada por Whyte y Helwig (1) y por Clark y Mihm. Lentigo maligno 01. Dermatoscopia. Pigmentación asimétrica de las salidas foliculares (flechas pequeñas), estructura romboidal (flecha grande), oclusión de las salidas foliculares (círculo) y puntos azul-gris (asterisco) Lentigo Maligno Cox NH, Aitchison TC, Sirel JM, Mackie RM. Comparison between lentigo maligna melanoma and other hystogenetic types of malignant melanoma of the head and neck. Br J Cancer 1996; 73:940-4. Cohen LM, McCall MW, Zax RH. Mohs micrographic surgery for lentigo maligna and lentigo maligna melanoma. Dermatol Surg 1998; 24:673-77 11.- Femenino de 26 años su padecimiento actual inició hace 4 años, cuando sus ciclos menstruales empezaron a ser irregulares. Su ritmo actual es de 40-90 x 3-4. FUR: hace 3 meses. Inició vida sexual a los 23 años y no ha podido embarazarse. En la exploración física encontramos la piel ligeramente seca, hay salida de líquido blanquecino escaso a la expresión del pezón izquierdo con presencia de giordano positivo derecho. El estudio que es de mayor utilidad para aclarar el diagnóstico es: a) prueba de embarazo b) LH, FSH y estrógenos c) tomografía de cráneo d) prolactina
  • 13. En toda paciente con galactorrea, trastornos menstruales, hirsutismo, disminución de la libido o infertilidad, deben determinarse los niveles de PRL plasmática, entre el tercer y quinto día del ciclo menstrual, si este es regular o en cualquier momento, si presenta amenorrea u oligomenorrea.2 Si en la primera determinación se obtienen cifras elevadas debe repetirse y si se confirma nuevamente, se establece el diagnóstico de hiperprolactinemia. Una historia clínica com-pleta, con un interrogatorio y examen físico detallados, permitirá orientarnos hacia la causa de la hiperprolactinemia. La primera causa que se debe descartar antes de emprender otros estudios, es el embarazo. El uso de estrógenos, anticonceptivos orales y drogas que aumentan la secreción de PRL debe precisarse en el interrogatorio, así como la presencia de quemaduras u otras lesiones en tórax que se buscan en el examen físico. Pocas mediciones hormonales tienen el significado clínico que se observa con la prolactina. La técnica está bien estandarizada y presenta bastante confiabilidad; la muestra de sangre se puede obtener a cualquier hora del día y debido a las variaciones fisiológicas (efecto del ejercicio, alimentos, irritación de la pared costal) cuando se obtiene una cifra de prolactina por arriba de lo normal, es necesario repetir el análisis. Varios fármacos pueden producir una moderada elevación en los niveles de prolactina, sin alcanzar los valores que se encuentran en prolactinoma; las drogas más comunes son metoclopramida, fenotiazinas, risperidona, verapamil,metildopa, reserpina y los inhibidores de MAO. Exámenes: prolactina plasmática (prolactinemia), tirotropina (hormona estimulante de la glándula tiroides o TSH), test de embarazo. Prolactina menor de 100 ng/ml: baja sospecha de prolactinoma. Prolactina entre 100 y 300 ng/ml: mayor posibilidad de prolactinoma. Prolactina superior a 300 ng/ml: alta sospecha de prolactinoma. Es precisa resonancia magnética nuclear (RNM) de hipófisis (detecta prolactinoma mayor de 3mm). . En el 50% de los casos de hiperprolactinemia se detecta tumor: microadenomas o macroadenomas; la prolactina suele encontrarse entre 100 y 200ng/ml; los macroadenomas tienen efecto de masa y causan hipopituitarismo; los microadenomas no presentan efecto de masa y sólo producen hipogonadismo. Referencias 1. Schlechte JA. Prolactinoma. N Engl J Med 2003;349:2035-2041. 2. Zárate A, Canales ES, Jacobs LS, Soria J, Daughaday WH. Restoration of ovarian function in patients with the amenorrhea-galactorrhea syndrome after long-term therapy with L-Dopa. Fertil Steril 1973;24:340. 3. Tyson JE, Carter JN, Andreassen B, Huth J, Smith B. Nursing mediated prolactin and luteinizing hormone secretion during puerperal lactation. Fertil Steril 1978;30:154. 4. Schlechte JA, Sherman BM, Chapler FK, VanGilder J. Long-term followup of women with surgically treated prolactin-secreting pituitary tumors. J Clin Endocrinol Metab 1986;62:1296-301. 5. Losa M, Mortini P, Barzaghi R, Gioia L, Giovanelli M. Surgical treatment of prolactin-secreting pituitary adenomas: early results and long-term outcome. J Clin Endocrinol Metab 2002;87:3180-3186.
  • 14. 12.- Es una de las enfermedades tiroideas autoinmunes organo-específica más frecuentes. Es más común en la mujer, posee una asociación directa con otras enfermedades autoinmunes y, por lo general, se presenta con hipertiroidismo, bocio difuso, oftalmopatía, además, en algunos casos coincide con mixedema pretibial: a) Carcinoma basocelular b) Enfermedad de Graves c) Adenoma tóxico d) Síndrome del eutiroideo enferm Introducción La enfermedad de Graves-Basedow (EGB) constituye la entidad más importante, por su frecuencia, entre las enfermedades que producen hipertiroidismo. Se caracteriza por la presentación de la tríada sintomática constituida por hipertiroidismo, bocio difuso y oftalmopatía (50%). Es más frecuente en la 3ª y 4ª década de la vida y afecta con más frecuencia a mujeres (7/1 en zonas no bociógenas y 3/1 a partir de los 45 años en zonas de bocio endémico) 1. Etiopatogenia Aunque no se conocen exactamente los factores que inician y mantienen la enfermedad, hay elementos que permiten encuadrarla en el marco de la patología autoinmunitaria, como la presencia de autoanticuerpos antitiroglobulina y antitiroperoxidasas (TPO), así como la presencia de inmunoglobulinas antireceptor de la TSH (TRAb) que son estimuladoras de la función y del crecimiento del tiroides. Para el desarrollo de la enfermedad parece que es necesaria la intervención de factores genéticos y ambientales (exceso de Yodo) 2. Clínica La enfermedad se caracteriza por la presencia de síntomas de hipertiroidismo (nerviosismo 99%, sudoración 91%, intolerancia al calor y palpitaciones 89%, pérdida de peso 85%, aumento de apetito 69% y síntomas oculares 55%) y signos clínicos (aumento uniforme del tamaño de la glándula 97%, oftalmopatía 60%, taquicardia en reposo 90%, temblor 70% y retracción palpebral 38%) relacionados con el aumento de receptores adrenérgicos que determinan las hormonas tiroideas 1,2. La historia natural de la enfermedad se caracteriza por fases cíclicas de exacerbación y remisión, de duración y presentación imprevistas; aunque, hoy en día, está artefactada por los tratamientos utilizados. En aproximadamente el 25% de los pacientes, especialmente en aquellos con una forma leve de enfermedad, el proceso se autolimita al año o más, regresando espontáneamente a un estado eutiroideo 3. En cuanto a la oftalmopatía, hay una forma no infiltrativa o pálpebro-retráctil (afecta más frecuentemente al sexo femenino, afecta simétricamente a ambos ojos y que evoluciona paralelamente al hipertiroidismo) y una forma infiltrativa (infiltración del tejido retroorbitario por mucopolisacáridos y linfocitaria con protusión ocular asimétrica)1. Otras manifestaciones menos frecuentes son la aparición de mixedema pretibial o dermopatía infiltrativa y, la acropaquia tiroidea 1. Referencias 1.- Foz M. Enfermedades del tiroides. En Rozman C (ed): Farreras-Rozman Medicina interna. Editorial Doyma. Barcelona, 1992:1997.
  • 15. 2.- Wartofsky L. Diseases of the thyroid. En Braunwald E, Isselbacher KJ, Wilson JD, Martin JB, Fauci AS, Kasper DL (eds): Harrison’s principles of internal medicine. Editorial McGraw-Hill. EEUU, 1994: 1930. 3.- De Groot. Graves’ diseases and the manifestations of thyrotoxicosis. En De Groot LJ, Reed Larsen P, Hennemann G.(eds): The thyroid and its diseases. 1996: 371. 4.- Haynes RC, Murad F. Drogas tiroideas y antitiroideas. En Goodman LS, Gilman A (eds): Las Bases farmacológicas de la terapéutica. Editorial Panamericana. México, 1982: 1376. 5. - Feliciano DV. Everything you wanted to know about Graves’ disease. Am J Surg 1992, (164): 404. 13.- Masculino de 42 años con antecedentes de litiasis renal cálcica bilateral recidivante. Acude Con resultados de laboratorio los cuales demuestran una calcemia de 11.1 mg%, fosfatemia de 2 mg, calciuria de 280 mg/24h y fosfaturia de 1600 mg/24h, con una reabsorción tubular de fósforo (RTP) del 70%. La sospecha más probable del origen de ésta litiasis es: a) Acidosis tubular b) Intoxicación vitamina D c) Hiperparatiroidismo primario d) Sarcoidosis Exploración y pruebas complementarias. El examen clínico es muy pobre. Excepcionalmente se puede palpar una tumoración cervical en el 4% de los casos. Son los síntomas lo que hacer evocar el diagnóstico y aconsejan determinar la calcemia. Calcemia: su determinación es el elemento fundamental del diagnóstico y se aconseja repetirla varias veces hasta comprobar su existencia, ya que puede ser constante o intermitente por circunstancias como la deshidratación o determinados fármacos pueden causar un aumento transitorio de la misma. HIPERCALCEMIA Es el aumento del calcio sérico por encima de 10,5 mg/dl. Cerca de la mitad de las hipercalcemias son “falsas hipercalcemias”, por extracción sanguínea en condiciones no ideales (postpandriales, torniquetes prolongados...) o por hiperalbuminemia que condiciona “pseudohipercalcemia”, por lo que ante todo paciente con hipercalcemia debemos en primer lugar calcular la calcemia según las proteínas totales (calciocorregido Ca2+) en base a la siguiente fórmula:
  • 16. Junto al calcio se ha determinar la albúmina pues si esta es menor de cuatro habrá que corregir convenientemente la cifra del calcio. Como se ha expuesto, la etiología más frecuente de las hipercalcemias es el hiperparatiroidismo primario. Los valores sumamente altos suelen asociarse a los infrecuentes carcinomas paratiroideos, ello es debido a que el tumor produce una proteína parecida a la hormona paratiroidea que ha sido detectada en el suero y que se denomina proteína relacionada con la hormona paratiroidea. Se aconseja investigar esta proteína en los pacientes con una aparición reciente de hipercalcemia y valores normales de PTH, no obstante el hiperparatiroidismo normocalcémico es una posibilidad rara, si no existe insuficiencia renal, ni deficiencia de vitamina D. A pesar de ello pueden darse situaciones confusas de hiperparatiroidismo con calcemia en ayunas normal, en estos casos puede ser útil comprobar la hipercalcemia postabsortiva, o realizar una prueba de provocación con benzotiacidas. La fosforemía generalmente esta disminuida. La hipofosfatemia es un dato diagnóstico menos específico que la hipercalcemia. En el hiperparatiroidismo es frecuente la hipercalciuria, pero menos que en otros estados de hipercalcemia. La calciuria está elevada siendo superior a 200 mg/24 h, si bien sus valores oscilan en función del aporte de calcio, así cuando es bajo, la calciuria se sitúa entre 50 y 150 mh/24 h; al aumentar el aporte, los valores de calciuria oscilan entre 100 y 300 mg/24h..La determinación de la calciuria se realiza cuando las cifras de calcio y PTH en suero están elevadas. Esta determinación es necesaria para descartar una HHF antes de la exploración quirúrgica, así como para decidir cuando intervenir a un paciente con hiperparatiroidismo primario asintomático La fosfaturia está elevada, siendo superior a 800 mg/24 h. El aclaramiento de fósforo unido a la creatinina está elevado, siendo superior a 12 ml/mn Las fosfatasas alcalinas esta elevadas en las formas con signos óseos así como la citremia (por encima de 26 mg/dl) y la hidroxipolinuria. Puede aparecer con frecuencia una acidosis metabólica con hipercloremia. Determinación de la PTH: es determinante. La tríada analítica clásica diagnóstica del hiperparatiroidismo primario incluye: hipercalcemia, hiper-PTH e hipofosforemia. También su pueden realizar pruebas de frenaje por perfusión cálcica y una prueba de estimulación por hipocalcemia provocada por medio de EDTA sódico, esta prueba aporta pocos datos útiles debido a su dificultad de interpretación.
  • 17. ALTERACIONES DEL METABOLISMO DEL CALCIO AUTORES: Carlos López Vargas Coral Suero Méndez José Mª Benavente Ramos 14.- Se trata de paciente masculino de 14 años de edad higiénica dietética, convivencia con aves, gatos e ingestión de berros. Inicia su padecimiento actual hace dos semanas con fiebre de 39ºC sin predominio de horario, náusea y dolor en hipocondrio derecho. A la exploración física con palidez generalizada y hepatomegalia 3-3-5. Resto de la exploración sin datos patológicos. Laboratorio con BH, con Hb de 11.5, Hto de 40, leucocitos; 16,500, eosinófilos; 38%, linfocitos; 30%, formas inmaduras; 4, plaquetas; 270,000, Fosfatasa alcalina; 280 UI/L. El diagnóstico más probable ES: a) Larva migrans visceral b) Toxoplasmosis generalizada c) Fasciolosis d) Gnatostomiasis Definición La Fasciolosis es una zoonosis parasitaria causada por la Fasciola hepática que ocasiona patología y sintomatología hepato-biliar. Clínica Se considera las siguientes formas de presentación clínica: a) Sintomática: Aguda o invasiva, crónica o de localización y extrahepática. 1. Aguda o invasiva: Hay tres elementos esenciales a identificar: hepatomegalia dolorosa, fiebre y eosinofilia con cifras que superan frecuentemente el 30-40%. 2. Crónica o de localización: La sintomatología y signología corresponden a padecimiento crónico hepato biliar incluyendo cólicos biliares y litiasis biliar. 3. Extrahepática: Incluye nódulos subcutáneos en el hipocondrio derecho, seno derecho, escápula derecha con poco dolor local y signos inflamatorios. Eosinofilia alta. b) Asintomática: En algunas personas los síntomas o signos suelen pasar desapercibidos. Diagnóstico de Laboratorio
  • 18. En la forma aguda, la búsqueda de huevos en las deposiciones es inútil, ya que las formas juveniles están en el tejido hepático, por lo tanto las pruebas inmunobiológicas son importantes. Son útiles la inmunoelectroforesis o inmunodifusión buscando el arco 2 de Caprón (4). El inmunoblot o westernblot tiene buena sensibilidad y especificidad (5,6). Se han identificado fracciones antigénicas en las cistenilproteasas de F. hepatica (7). En las formas crónicas, la búsqueda de huevos en heces es lo indicado. Son útiles la sedimentación rápida de Lumbreras (8). Recientemente se han preparado anticuerpos monoclonales contra el parásito y ello ha permitido elaborar la técnica de ELISA para detectar los coproantígenos (E/S) del parásito en heces. La ecografía de vías biliares (v.b.) puede detectar al parásito moviéndose en las v.b. o vesícula. En formas extrahepáticas, la eosinofilia alta es orientadora; F. hepatica en las biopsias confirma el diagnóstico. 15.- Ante un paciente con diagnóstico de trastorno obsesivo-compulsivo, usted decide indicar el siguiente fármaco por ser el principal tratamiento de elección: a) Antipsicóticos. b) Antidepresivos inhibidores de la recaptación de noradrenalina. c) Antidepresivos inhibidores de la recaptación de serotonina. d) Benzodiacepinas. El abordaje farmacológico del TOC se basa en los inhibidores selectivos de la recaptura de serotonina (ISRS), medicamentos que han resultado ser efectivos y seguros. Como grupo, los ISRS son igual de eficaces que la clorimipramina, pero producen menos efectos secundarios y, por lo tanto, mejor tolerancia y mejor apego al tratamiento. La efectividad antiobsesiva parece ser independiente de su actividad antidepresiva. Entre los factores predictores de respuesta al tratameinto, se ha propuesto que los altos puntajes en las compulsiones predicen una mala respuesta al tratamiento. Para los casos refractarios y resistentes o cuando hay síntomas de comorbilidad, se han utilizado, con relativo éxito, combinaciones con diferentes IRS, con benzodiacepinas, o bien, con potenciadores como el litio, o antipsicóticos, como la risperidona y el haloperidol. El uso del carbonato de litio es controvertido, aunque parece ser útil como potenciador a largo plazo entre 15 y 30% de los pacientes. La combinación de ISRS con antipsicóticos comenzó a utilizarse en los pacientes con síntomas psicóticos, aunque ahora se combinan también en los pacientes resistentes. Tratamiento farmacológico del TOC Cristina Lóyzaga* Humberto Nicolini* *División de Investigaciones Clínicas. Instituto Nacional de Psiquiatría. Ramón de la Fuente. Calzada México-Xochimilco, 101, San Lorenzo Huipulco, 14370, México D.F. Primera versión: 14 de septiembre de 2000. Aceptado: 26 de septiembre de 2000. 16.- Masculino de 44 años, con antecedentes de mialgias, artralgias de grandes articulaciones, fatiga y pérdida ponderal desde 5 meses. Hace 2 semanas presentó febrícula y 10 días después orina oscura y tos seca. Dentro de sus antecedentes menciona infecciones recurrentes de los oídos. A la exploración física cuenta con 37.2 °C, frecuencia cardiaca de 110 por minuto, frecuencia respiratoria de 18 por minuto, tensión arterial de 130/80 mm Hg, oídos con cicatrices en ambos tímpanos, tórax con estertores en ambas bases, dolor costovertebral y en extremidades hemorragia periungueal en los dedos. Laboratorio: leucocitosis de 13,200. Hepatocitos: 35 %, VSG 55 mm/h, BUN 49 mg/dl, creatinina 4.0 mg/dl, serología con factor reumatoide 1/40, VDRL negativo, anticuerpos citoplasmáticos antineutrófilos positivos, sedimento urinario con cilindros hemáticos-gabinete: Rx con infiltrados alveolares bilaterales.
  • 19. El diagnóstico más probable de este paciente es. a) Poliarteritis. b) Polimialgia reumática. c) Vasculitis probablemente granulomatosa de Wegener. d) Púrpura de Henoch-Schonlein. La Púrpura de Schönlein-Henoch es un síndrome vasculítico sistémico, de pequeños vasos, con afectación primaria de la piel, articulaciones, tuvo digestivo y riñones (1-11). Es la causa más frecuente de púrpura no trombocitopénica y vasculitis en niños. (1) (9) Se postula que la etiopatogenia puede ser debida a una alteración inmunológica, pues en esta enfermedad se describen una gran variedad de anormalidades de la IgA, así como niveles elevados de citosina, FNT alfa e IL-6. Se plantea una asociación entre esta respuesta inmunitaria y una causa infecciosa, en la literatura se describen casos desencadenados por Estreptococo sp, virus de la hepatitis A y C, Salmonella sp, Mycoplasma, virus de Epstein Barr, varicela, parvovirus B19, adenovirus, parainfluenza, campylobacter y yersinia entre otros, también se invocan determinados fármacos, alimentos, vacunas, exposición al frío y picaduras de insectos. (1, 3, 9) Parece ser que la afinidad de los gérmenes por el endotelio y los hematíes se debe a un esfingofosfolípido neutro en la pared de estas células y su efecto citotóxico está mediado por una proteína no estructural del germen, que induce la apoptosis mediante la ruta del TNF. Este efecto puede explicar el daño selectivo hacia los capilares característicos de la PSH, y hacer que las células sean más susceptibles al depósito de IgA que sufren. (9) Los hallazgos histopatológicos son los típicos de la vasculitis leucocitoclástica ligados patogénicamente al depósito de inmunocomplejos, activación del complemento, quimiotaxis de neutrófilos, con daño secundario de la pared vascular debido a la liberación de enzimas por parte de los neutrófilos activados. (1, 9, 12, 13) El diagnóstico se basa fundamentalmente en las manifestaciones clínicas. (1, 2) 17.- Femenino de 18 meses, originário de zona rural del estado de Guerrero, es traído a consulta por cojera desde que empezó a caminar (a los 16 meses). A la exploración presenta marcha en Trendelenburg. La primera sospecha diagnóstica es: a) Sinovitis transitoria de cadera. b) Enfermedad de Perthes. c) Luxación congénita de cadera. d) Artritis séptica de cadera. . La falta de relación normal en las estructuras que forman una articulación se conoce como luxación. En el caso de la cadera, la cabeza del fémur no encaja debidamente en su cavidad (acetábulo). El desarrollo del acetábulo será normal siempre que la cabeza del fémur permanezca en posición correcta y el esfuerzo muscular sea adecuado. Siempre que esto no se cumpla, se producirá una alteración entre el cótilo y la cabeza femoral, dando lugar a una alteración con el resultado de luxación de la cadera. FACTORES DE RIESGO El factor principal en cuanto a la estructura es la falta de estabilidad en la articulación debido a la laxitud de la cápsula y del ligamento redondo. Entre los factores genéticos más destacados estaría pertenecer a una familia con antecedentes, factor que se observa en ciertas razas, (en relación con una laxitud ligamentosa familiar).
  • 20. La frecuencia es mayor en las niñas que en los niños, factor que se incrementa más cuando la presentación del feto es de nalgas completas. La posición del niño cuando ha nacido también es determinante, siempre que la posición sea estable se reduce el riesgo de luxación. DIAGNÓSTICO: EXPLORACIÓN FÍSICA a) Maniobras de Ortolani y Barlow - click audible-. Ortolani: el click se produce cuando la fóvea de la cabeza del fémur encuentra la cresta cotiloidea caminando sobre ella. La limitación a la abducción desaparece, esto es signo de reducción. Barlow: se realiza la aducción de la cadera, que tras una ligera presión longitudinal sobre el fémur, produce una luxación. La prueba de Ortolani traduce una luxación y la de Barlow una cadera luxable. b) Asimetría de pliegues -no valorable en luxaciones bilaterales-. c) Limitación a la abducción. d) Dismetría La marcha de Trendelenburg también es llamada marcha en sacudida del glúteo medio o de los abductores. Observamos la marcha de Trendelenburg en pacientes con mala función de los abductores. Si estos músculos se acortan, pierden su ventaja mecánica. El glúteo mediano, que es el principal abductor de la cadera, se origina en la superficie lateral del ilíaco y se inserta en la superficie superior del trocánter mayor. Si una enfermedad de la cadera provoca que el trocánter mayor se localice más proximalmente de lo normal (se mueve más cerca de la masa muscular del glúteo mediano), la contracción del músculo generará una fuerza abductora menor. Son causas de dicho acortamiento: - Luxación congénita de cadera - Coxa vara debida a: * Causa idiopática * Fractura del cuello femoral consolidada con mala alineación * Enfermedad de Perthes * Deslizamiento epifisario * Raquitismo García Portabella, M. Luxación congénita de cadera antes de los tres meses de edad. 2001. Disponible en: http://www.vhebron.es/htr/ortopediatria. • Garcia-Siso Pardo, J. M. Displasia del desarrollo de la cadera. (Parte I). Peditría Rural y Extrahospitalaria. Vol. 32. Nº 304 Págs. 481-491. 2002. • Graf, R. Classification of hip joint dysplasia by means of sonography. Arch Orthop Trauma Surg 102:248-255, 1984. • Edeiken, J. Luxación congénita de cadera. Diagnóstico Radiológico de las Enfermedades de los Huesos. Ed. Médica Panamericana. Págs. 388-392. 18.-The following conditon is commonly seen in the magnesium-amonium-phosphate (struvite) stones: a) Recurrent P. mirabilis infection b) Double J stent placement c) Resorptive hypercalciuria d) Renal azotemia
  • 21. Esta bacteria de colonias redondeadas tiene la habilidad de producir grandes niveles de ureasa. La ureasa hidroliza urea a amoníaco, (NH3) y eso hace a la orina más alcalina. Y al subir la alcalinidad puede liderar la formación de cristales de estruvita, carbonato de calcio, y/o apatita. Esta bacteria puede encontrarse en cálculos, y esas bacterias escondidas allí, pueden reiniciar una infección post tratamientos antibióticos. Al desarrollarse los cálculos, después de un tiempo pueden seguir creciendo más y causar obstrucción dando fallas renales. Proteus también puede producir infecciones de heridas, septicemia y neumonías, sobre todo en pacientes hospitalizados.  Esipov, Sergei E. and J. A. Shapiro (1998). «Kinetic model of Proteus mirabilis swarm colony development». Journal of Mathematical Biology 36 (3). doi 10.1007/s002850050100.  Frénod, Emmanuel (2006). «Existence result for a model of Proteus mirabilis swarm». Differential and integral equations 19 (6): pp. 697-720. http://arxiv.org/abs/math.FA/0702761. 19.- Femenino de 34 años, es atendida en sala de partos secundario a eutocia, durante la reparación de la episiotomía media hay un marcado incremento en el sangrado transvaginal. La causa más probable de la sintomatología de esta paciente es: a) Retención de restos placentarios b) Atonía uterina c) Laceración cervical d) Laceración vaginal Atonía uterina La atonía uterina o inercia, consiste en la falta de contracción del útero gestante luego del alumbramiento. Este fenómeno ocurre en el 2 al 5 % de los partos por vía baja. La atonía uterina es favorecida por numerosas situaciones como: la multiparidad, la sobre distensión uterina, la macrosomía, el hidramnios, los intentos de versión, los antece3 0 M. Pesaresi, J. Palacios Jaraquemada Patología de Urgencia, Año 9, Nro. 3, Septiembre de 2001 dentes de hemorragia pre y posparto, la corioammnionitis y la interrupción en la administración de oxitócicos, entre otros. No ha sido demostrada una relación directa entre las atonías y el uso de los anestésicos administrados por vía peridural. Por el contrario, hay una asociación demostrada con el uso de Halotano como anestésico general. El tratamiento clásico de la atonía uterina está constituído por la reposición volumétrica (sangre, coloides y cristaloides), la utilización de masajes y por la administración de drogas que promueven la contracción del músculo uterino. Cuando estas medidas no son eficientes, se procede casi invariablemente a la histerectomía de hemostasia. Es de notar, que la atonía uterina puede aparecer en el primer embarazo y sin ningún antecedente previo. La atonía uterina constituye, en casi todas las series mundiales, más del 50% de las hemorragias graves del posparto. Patología de Urgencia, Año 9, Nro. 3, Septiembre de 2001 20.- Lactante de 10 meses. Acude al servicio de urgencias por presentar, vómito en 5 ocasiones, fiebre 38.7 ºc, Antecedentes: Acude a guardería. Desde hace 2 días presentó aumento en el número de evacuaciones en promedio de 4 a 5 veces al día, de consistencia líquidas, abundantes, sin moco ni sangre, actualmente ha pesnetado 6 en las últimas 6 hrs. . Previamente sano. Exploración física: Temp. 38. 5, Oc, alerta, sed aumentada bebe con avidez, ojos hundidos, llanto sin lágrimas, mucosa oral seca, fontanela anterior hundida.
  • 22. El tratamiento de primera elección para este paciente es: a) Plan a de hidratacion oral. b) Plan b de hidratacion oral. c) Plan c de hidratacion oral. d) Antibiotico y ayuno. Las enfermedades diarreicas, de acuerdo con la evaluación del estado de hidratación, se clasifican en: casos sin deshidratación, con deshidratación, con choque hipovolémico por deshidratación. 7.2.3 Caso sin deshidratación, es aquel que presenta generalmente menos de cuatro evacuaciones líquidas en 24 horas, ausencia de vómito, sin signos clínicos de deshidratación. 7.2.4 Caso con deshidratación, es aquel que presenta dos o más de las manifestaciones clínicas siguientes: 7.2.4.1 Inquieto o irritable; 7.2.4.2 Ojos hundidos, llanto sin lágrimas; 7.2.4.3 Boca y lengua secas, saliva espesa; 7.2.4.4 Respiración rápida; 7.2.4.5 Sed aumentada, bebe con avidez; 7.2.4.6 Elasticidad de la piel, mayor o igual a dos segundos; 7.2.4.7 Pulso rápido; 7.2.4.8 Llenado capilar de tres a cinco segundos; 7.2.4.9 Fontanela anterior hundida (lactantes); 7.2.5 Caso con choque hipovolémico, es aquel que presenta dos o más de las manifestaciones clínicas siguientes: 7.2.5.1 Inconsciente o hipotónico; 7.2.5.2 No puede beber; 7.2.5.3 Pulso débil o ausente; 7.2.5.4 Llenado capilar mayor de cinco segundos; 7.2.6 El manejo de los casos de enfermedades diarreicas se basa en tres planes generales de tratamiento: 7.2.6.1 Plan A: Para pacientes con enfermedad diarreica sin deshidratación con atención en el hogar: 7.2.6.1.1 Continuar con la alimentación habitual; 7.2.6.1.2 Aumentar la ingesta de los líquidos de uso regular en el hogar así como Vida Suero Oral: de este último, en los niños menores de un año de edad, ofrecer media taza (75 ml) y en los mayores de un año, una taza (150 ml) y administrarlo a cucharadas o mediante sorbos pequeños, después de cada evacuación. 7.2.6.1.3 Capacitar a la madre para reconocer los signos de deshidratación y otros de alarma por enfermedades diarreicas: (sed intensa, poca ingesta de líquidos y alimentos, numerosas heces líquidas, fiebre, vómito y sangre en las evacuaciones), con el propósito de que acuda nuevamente a solicitar atención médica en forma oportuna. 7.2.6.2 Plan B: Para pacientes con diarrea y deshidratación con atención en la unidad de salud: 7.2.6.2.1 Administrar Vida Suero Oral 100 ml por kilogramo de peso, en dosis fraccionadas cada 30 minutos durante cuatro horas; 7.2.6.2.2 Si el paciente presenta vómito, esperar 10 minutos e intentar otra vez la hidratación oral, más lentamente; 7.2.6.2.3 Al mejorar el estado de hidratación, pasar al Plan A. En caso contrario, repetir el Plan B por otras cuatro horas, de no existir mejoría pasar al Plan C; 7.2.6.2.4 Si los vómitos persisten, existe rechazo al Vida Suero Oral, o gasto fecal elevado (más de 10 g/kg/hora o más de tres evacuaciones por hora) se hidratará con sonda nasogástrica, a razón de 20 a 30 ml de Vida Suero Oral por kilogramo de peso, por hora. 7.2.6.3 Plan C: Para pacientes con choque hipovolémico por deshidratación: 7.2.6.3.1 Inicie inmediatamente administración de líquidos por vía intravenosa, con solución Hartmann; si no se encuentra disponible, use solución salina isotónica al 0.9%, de acuerdo con el siguiente esquema:
  • 23. PRIMERA HORA 50 ml/kg SEGUNDA HORA 25 ml/kg TERCERA HORA 25 ml/kg Evalúe al paciente continuamente. Si no mejora, aumente la velocidad de infusión. Cuando pueda beber (usualmente en dos a tres horas), administre VSO, a dosis de 25 ml/kg/hora; mientras sigue líquidos IV. Al completar la dosis IV, evalúe al paciente para seleccionar Plan A o B, y retirar venoclisis, o repetir Plan C. Si selecciona el Plan A, observe durante dos horas para asegurarse de que el responsable encargado del paciente puede mantenerlo hidratado con VSO y además, alimentarlo en su domicilio. NORMA Oficial Mexicana NOM-031-SSA2-1999, Para la atención a la salud del niño. Evaluación del estado de hidratación de un paciente con diarrea17 Signos Plan A Plan B Plan C Definición Leve o inaparente Moderada o clínica Grave Pérdida de agua corporal Menos de 50 ml/kg peso o menos de 5% del peso 50 a 100 ml/kg peso ó 6 a 9% del peso 100 ml/kg peso o más (10% o más del peso) Condición general Bien, alerta Irritable Letárgico o inconsciente Globo ocular Normales Algo hundidos Muy hundidos y secos Lágrimas Presentes Ausentes Ausentes Mucosa oral Húmeda Seca Muy seca Sed Paciente bebe normalmente Paciente bebe con avidez, sediento Paciente bebe mal o no es capaz de hacerlo Pliegue cutáneo Vuelve a lo norma rápidamente Se retrae lentamente < 2 s Se retrae muy lentamente > 2 s Tratamiento Plan A Plan B Plan C 21.- Masculino de 17 años de edad, con antecedentes de cuadro gripal de tres días de evolución, inicia hace 12 hrs. con dolor en la bolsa escrotal y aumento de volumen, a la exploración física se encuentra edema. tensión y enrojecimiento del escroto y no se puede palpar el testículo derecho. El diagnóstico más probable de este paciente es: a) Torsión testicular b) Orquiepididimitis c) Hidrocele agudo d) Hernia inguinal estrangulada. El síndrome escrotal agudo es una urgencia urológica cuyo principal síntoma es el dolor intenso del contenido escrotal, su importancia radica en el diagnóstico precoz de la torsión testicular, cuadro de emergencia quirúrgica, por lo que se ha establecido un síndrome en el que se engloban diversas afecciones intraescrotales cuyo denominador común es el dolor testicular de aparición brusca que puede ir acompañado de otros síntomas dependiendo de su etiología, como tumefacción escrotal, náuseas, vómitos, síndrome miccional... que nos orientarán sobre el diagnóstico. RECUERDO ANATOMICO
  • 24. El escroto es la bolsa que aloja al testículo, éste es una glándula de forma tubular cuya capa externa es la túnica albugínea. Del polo superior del testículo sale el epidídimo que tras descender por la cara posterior de éste, se continúa con el cordón espermático. El cordón espermático está formado por el conducto deferente, los vasos sanguíneos (arterias espermáticas y deferencial y venas espermáticas), linfáticos y nervios que llegan a la glándula. El cordón espermático está envuelto por el músculo cremaster. La irrigación del testículo procede de la arteria espermática interna, rama de la aorta abdominal y el drenaje venoso se realiza a través de la vena espermática interna que desemboca en la vena cava inferior en el lado derecho y en la vena renal en el lado izquierdo. Las funciones del testículo son endocrina, las células de Leydig sintetizan y liberan la testosterona y exocrina que es la espermatogénesis. ORQUITIS Y EPIDIDIMITIS La inflamación del contenido escrotal plantea un problema de diagnóstico en el escroto agudo principalmente con la torsión funicular. La infección aislada del testículo es rara, siendo más frecuente la presentación en forma de orquiepididimitis. Predomina en hombres jóvenes sexualmente activos y en ancianos con infección urinaria pero se puede producir a cualquier edad. La causa más frecuente de la inflamación del contenido escrotal es la infección. Así en los jóvenes prepuberales los gérmenes más frecuentes son las enterobacterias aunque sobre factores predisponentes como son las malformaciones congénitas (válvulas uretrales, abombamiento ectópico de un uréter a vesícula seminal). En el adolescente es causa frecuente la orquitis en el transcurso de una parotiditis. En los varones jóvenes menores de 40 años es la causa infecciosa con mucho lo más frecuente, sobre todo enterobacterias (E. coli, Proteus Mirabilis, Klebsiella pneumoniae) y gérmenes de transmisión sexual como Neisseria gonorrheae, Mycoplasma, pero sobre todo la Chlamydia tracomatis, y cocos gram positivos como estafilococos aureus. Por encima de los 40 años a la causa infecciosa se suele añadir una obstrucción urinaria distal, tuberculosis, reflujo urinario posterior a la prostatectomía transuretral, y la reacción granulomatosa por el tratamiento del cáncer vesical superficial con bacilo de Calmette Guerin (BCG). También puede aparecer orquiepididimitis en el curso de la varicela, fiebre tifoidea, mononucleosis infecciosa, rickettsiosis, brucelosis filariasis, actinomicosis, sinusitis, osteomielitis, endocarditis y ser secundaria a cuadros septicémicos por E. coli, Klebsiella, pseudomona, estreptococo y estafilococo. Una causa no infecciosa es el tratamiento con amioradona. Clínicamente aparece dolor escrotal intenso, irradiado a lo largo del cordón espermático por lo que irradia a ingle e incluso a abdomen con tumefacción escrotal no distinguiéndose el epidídimo del testículo, con induración de la piel escrotal que puede producir hidrocele reactivo con síntomas de infección urinaria concomitantes y de secreción uretral, además de afectación del estado general con fiebre que puede llegar a ser de hasta 40 °C, con escalofríos, acompañado de náuseas y vómitos. En la exploración física, el escroto se observa edematoso, tenso y enrojecido, el cordón se palpa engrosado y doloroso, siendo muy dolorosa la palpación, y difícil de distinguir el límite entre escroto y epidídimo en casos evolucionados. El signo de Prehn, al elevar el testículo afecto produce mejoría del dolor, es positivo. La transiluminación del escroto es negativa a no ser que exista un hidrocele reactivo, en tal caso será positiva. En cuanto a los estudios complementarios habrá que realizar analítica de orina, cultivo de orina, y de semen y de secreción uretral si la hubiere, cultivo de Lowenstein y tinción de Ziehl-Nielsen tanto en orina como en semen e inmunofluorescencia para Chlamydia, además de antibiograma. También es necesario realizar hemograma, hemocultivos seriados, una bioquímica sanguínea que incluya urea, creatinina e iones.
  • 25. En el sedimento de orina nos mostrará piuria en la mayoría de las ocasiones, discreta piuria sin bacteriemia en las ETS y piuria franca con urocultivo positivo en caso de gérmenes gram negativos, aunque también podremos encontrar a pacientes con sedimento urinario negativo. En el hemograma es frecuente leucocitosis con desviación a la izquierda. La ecografía nos mostrará engrosamiento y edema de las túnicas escrotales, pudiendo haber o no hidrocele, con epidídimo engrosado (patrón heterogénico), y un testículo aumentado de tamaño (patrón hipoecogénico global) o como una zona hipoecogénica perihiliar. La eco-dopopler está indicada cuando hay dudas sobre el diagnóstico diferencial con la torsión funicular, siendo en estos casos muy útil, y en la orquiepididimitis encontraremos una hipervascularización. La gammagrafía está indicada para el diagnóstico diferencial con procesos isquémicos y neoplásicos, es un método de gran sensibilidad y especificidad, pero que no se tiene siempre disponible en el área de urgencias. Se observará un aumento de la captación a nivel testicular debido a la hiperemia. Si después de todas las pruebas complementarias existen dudas sobre el diagnóstico, será necesaria la cirugía para llegar al diagnóstico definitivo. Referencias 1. Young Y., Miller R Incidence of malignant tumours in U.S. children. J Pediatr 1975; 86: 254- 258. 2. Reiter A., Schrappe M. Chemotherapy in 998 unselected childhood acute lymphoblastic leukemia patients. Results and conclusions of the multicenter trial ALL-BFM 86. Blood 1994; 84: 3122-3133. 3. Schorin M., Blattner S., Gelber R., et al. Treatment of childhood acute lymphoblastic leukemia: Results of Dana Farber Cancer Institute/Children s Hospital acute lymphoblastic leukemia consortium protocol 85-01. J Clin Oncol 1994; 12: 740-747. 22.-Se trata de escolar de 10 años con los siguientes antecedentes: a los 6 años y en varios análisis se detecta hematuria microscópica, con niveles IgA normales y normocalciuria. A la edad de 9 años persistía la hematuria en los análisis e incluso habían observado algún episodio recortado de hematuria macroscópica. Un año más tarde se detectó proteinuria moderada de 1250 mg/24 horas. En el momento de la consulta persisten las alteraciones en el sedimiento, pero la proteinuria es de rango nefrótico, con creatinina sérica de 1,3 mg/dl. Existen antecedentes familiares de nefropatía evolutiva con desarrollo de insuficiencia renal y de miopía familiar por “lenticonus”. El diagnóstico más probable es del menor es: a) Glomerulonefritis mesangial (enfermedad de Berger). b) Enfermedad poliquística autosómica dominante. c) Nefritis intersticial por hipersensibilidad. d) Enfermedad de Alport En medicina, la enfermedad de Alport es una enfermedad genética ligada al cromosoma X, en la que una mutación en la síntesis del colágeno afecta los riñones, oídos, y ojos causando sordera y trastornos de la vista, incluyendo distrofia de la córnea y cataratas. Fue inicialmente identificado por el médico británico Cecil A. Alport en 1927, que describió una familia británica en la que muchos miembros desarrollaban enfermedades renales. Él describió que los hombres afectados en la familia morían a causa de enfermedades renales, mientras que las mujeres estaban menos afectadas.
  • 26. Se conoce ahora que muchos casos del síndrome de Alport son causados por una mutación en el gen del colágeno COL4A5, entre otras. Este gen codifica la cadena alfa-5 del colágeno tipo IV y está localizado en el cromosoma X. En ciertas familias se ven plaquetas de gran tamaño circulantes en la sangre, trombocitopenia e inclusiones leucocitarias que se parecen a las encontradas en la anomalía de May-Hegglin El síndrome de Alport se caracteriza por tener afección renal, coclear y ocular. La principal señal de este síndrome es la hematuria microscópica (microhematuria). Los hombres con el síndrome Alport ligado al cromosoma X (XLAS) padecen microhematuria desde una edad muy temprana. Alrededor del 90% de mujeres con XLAS también la tienen. Hay 2 métodos para el diagnóstico clínico: secuenciación y análisis de deleción/duplicación. El análisis de secuenciación de COL4A5 identifica cerca del 80% de las mutaciones de individuos afectados con antecedentes familiares en herencia ligada al X. El análisis de deleción/duplicación del gen COL4A5 identifica deleciones (típicamente multiexónicas) cercanas al 10% de individuos afectados con antecedentes familiares ligada al X Referencias Behrman RE, Kliegman RM, Jenson HB. Nelson Textbook of Pediatrics. 17th ed. Philadelphia, Pa: Saunders; 2003. 23.- Masculino de 2 de dos meses de edad, con sospecha de menigoencefalitis por clínica y un líquido cefalorraquídeo con aumento de leucocitos, aumento de proteínas e hipoglucorraquia en el que no se tiene aún un germen aislado. El tratamiento de elección es: a) Dicloxacilina y amikacina b) Ampicilina y cefotaxime c) Penicilina sódica cristalina d) Vancomicina La meningoencefalitis es definida como la inflamación de las meninges y el encéfalo. Dentro de las causas de la meningoencefalitis purulenta la más frecuente es la bacteriana. La etiología varía dependiendo del grupo de edad, en menores de 2 meses el más común es Streptococcus del grupo b, después de esta edad el más frecuente es el S. pneumoniae. El cuadro clínico se caracteriza por 4 síndromes: infeccioso, de hipertensión endocraneana, meníngeo y de daño neuronal. Las complicaciones pueden ser agudas, subagudas y crónicas. El diagnóstico se hace con el estudio citoquímico y cultivo del LCR. Las medidas terapéuticas irán encaminadas al manejo de los signos y síntomas componentes de los 4 síndromes. El tratamiento específico se iniciará en forma empírica y se modificará de acuerdo a los resultados del cultivo. En la meningitis bacteriana el aspecto macroscópico LCR es turbio o incluso purulento debido al mayor contenido de células y proteínas. En la mayoría de los casos de meningoencefalitis bacteriana el número de leucocitos es superior a 500 x mm3; y el predominio de células polimorfonucleares es mayor a 50%. La presencia de hipoglucorraquia (< 50% de la glicemia central o glucorraquia < 40 mg/dL) e hiperproteinorraquia moderada (entre 200 y 500 mg/dL) sugiere etiología bacteriana. Sin embargo, en casos con cuadros neurológicos de evolución más prolongada, mayor de 10 días, el diagnóstico diferencial debe establecerse con meningoencefalitis de etiología tuberculosa.
  • 27. Enfermedades infecciosas y microbiología Número number 1 enero-marzo January-march 2002 volumen volume 22 Meningoencefalitis bacteriana Fortino solórzano santos,* maría guadalupe miranda novales,** Rita d díaz ramos* 24.- Un niño de 12 años en Yautepec Morelos, se encuentra dormido sobre una toalla a la orilla de una alberca, súbitamente presenta dolor intenso en muslo derecho y se observa una pequeña pápula eritematosa El niño refiere sensación de ardor intenso en el muslo, su madre aplica una pomada con antihistamínico y le administra un antihistamínico oral. Dos horas después el paciente inicia parestesias, nausea y vómito, comienza a presentar dolor abdominal intenso, en episodios, síntomas que en una hora se hacen más intensos, la causa más probable de este cuadro es: a) Picadura de viuda negra. b) Picadura de alacrán. c) Picadura de cara de niño. d) Picadura de nauyaca El veneno que inocula es 15 veces más potente que el veneno de una serpiente de cascabel y puede condicionar efectos sistémicos graves e incluso la muerte. El primer síntoma generalmente es un dolor similar a una punción con un alfiler y la sensación se experimenta cuando realmente se ha efectuado la picadura de la araña. Sin embargo, es posible que algunas personas no lo sientan. Puede haber hinchazón y enrojecimiento leve y una lesión en forma de diana. De 15 minutos a una hora más tarde, un dolor muscular sordo se irradia desde el área de la picadura a todo el cuerpo. Síntomas
  • 28. Náuseas Ataque al estado general Diaforesis Contracturas musculares Dolor muscular Retención urinaria Estreñimiento Taquicardia Insuficiencia cardiaca Hipertensión arterial Inquietud Ansiedad Sensación de muerte Inminente Current Pediatric Diagnosis and Treatment 17 Ed Mc Graw Hill. Pag 346 . 2005 25.- Preescolar de 3 años es llevado a consulta por preentar hiporexia. Antecedentes: Originario de zona rural del estado de Guerrero, geofagia positiva, dolor abdominal, cólico desde hace varios meses. Las evacuaciones en los últimos 5 días son semilíquidas acompañadas de moco y pujo no sangre. E.F.: mala higiene personal, desnutrido, abdomen globoso blando y dolor a la presión en colon descendente, peristalsis aumentada. La complicación más frecuente que se puede presentar en este paciente es: a) Apendicitis. b) Obstruccion intestinal. c) Prolapso rectal, trichurosis d) Perforación intestinal. Introducción. La trichuriosis es una geohelmintiasis frecuente en zonas tropicales, rurales. Se estima que se encuentran infectadas unos 100 millones de personas en Latinoamérica y Caribe (Hotez PJ, et al., 2008). Predomina en niños en edad escolar, en quienes se asocia a colitis crónica y síndrome disentérico, retardo en el crecimiento y disminución de peso; la deficiencia en las funciones cognitivas y alteraciones conductuales se han relacionado con anemia ferropriva, altas cargas parasitarias y desnutrición. Los casos de la parasitosis en adultos que viven en zonas endémicas han aumentado, pero no se reportan usualmente. (Khuroo M, et al. 2010). La ascariosis y la trichuriosis son las infecciones por geohelmintos más frecuentes en México. NTDs en LAC: Prevalencia y distribución. Hotez PJ, et al, 2008. LAC: Latin American and Caribbean Region NTDs. Neglected Tropical Diseases. Las lesiones intestinales y el cuadro clinico varían en relación directa al número de parásitos y factores dependientes del hospedero (edad, estado nutricional, infecciones concomitantes). En infecciones leves y moderadas el daño, apenas apreciable, consiste en compresión mecánica de las células de la mucosa colónica y se asocia a dolor abdominal de tipo cólico y episodios diarreicos. En infecciones masivas la mucosa intestinal se encuentra edematosa y friable, con sangrado fácil; es característica la degeneración y necrosis de las células cercanas a la cabeza del parásito, con pequeñas hemorragias subepiteliales e inflamación con infiltración difusa de linfocitos y eosinófilos.
  • 29. Induce, al igual que los otros geohelmintos, una respuesta de tipo Th2 y respuesta reguladora Th2/Treg (Jackson JA, et al. 2009). Las manifestaciones clínicas varían de acuerdo a la masividad de la infección y la presencia de otros parásitos (poliparasitismo) e incluyen dolor abdominal, cefalea, hiporexia, pérdida de peso, diarrea crónica, disentería, pujo, tenesmo, prolapso rectal y signos y síntomas relacionados con anemia hipocrómica microcítica; cada tricocéfalo expolia alrededor de 0.005 ml de sangre/día y restos tisulares. Además, la irritación constante de las terminaciones nerviosas intramurales redunda en hiperperistaltismo. Complicaciones. - Poliparasitismo - Prolapso rectal - Anemia - Apendicitis - Infección bacteriana 2aria - Retraso pondoestatural y déficit cognitivo en escolares. TRICHURIOSIS Dra. Teresa Uribarren Berrueta Departamento de Microbiología y Parasitología, Facultad de Medicina, UNAM October, 2010 Vínculos. - Mohammad S. Khuroo, Mehnaaz S. Khuroo, and Naira S. Khuroo. Trichuris dysentery syndrome: a common cause of chronic iron deficiency anemia in adults in an endemic area (with videos). Gastrointestinal Endoscopy, Jan 2010; 71(1):200-204. doi:10.1016/j.gie.2009.08.002 - Geary TG, Woo K, McCarthy JS, Mackenzie CD, Horton J, Prichard RK, de Silva NR, (...), Bundy DA. Unresolved issues in anthelmintic pharmacology for helminthiases of humans. Int J Parasitol 2010;40(1):1-13. doi:10.1016/j.ijpara.2009.11.001 Geohelmintos y otros nematodos. - Kyung-Sun Ok, et al. Trichuris trichiura Infection Diagnosed by Colonoscopy: Case Reports and Review of Literature. Korean J Parasitol. Sept 2009;47(3):275-280 DOI: 10.3347/kjp.2009.47.3.275 - Hu Y, Xiao S-H, Aroian RV. The new anthelmintic tribendimidine is an L-type (Levamisole and Pyrantel) nicotinic acetylcholine receptor agonist. PLoS Neglected Tropical Diseases 2009;3(8), art. no. e499. - Jackson JA, Friberg IM, Little S, Bradley JE. Review series on helminths, immune modulation and the hygiene hypothesis: Immunity against helminths and immunological phenomena in modern human populations: Coevolutionary legacies? Immunology 2009;126 (1):18-27. doi:10.1111/j.1365-2567.2008.03010.x
  • 30. 26.- Acude al servicio paciente masculino de 66 años de edad el cual refiere disminución de la visión en ambos ojos con varios meses de evolución, esta disminución se presenta a la visión de lejos y más acentuada en la visión próxima o de lectura. Además ha notado mayor pérdida visual con luz solar intensa y se deslumbra con mayor facilidad. No refiere alteraciones en la percepción de los colores, aunque sí cree verlos más apagados, tampoco refiere metamorfopsias. El diagnóstico más probable es: a) Aumento de la presbicia. b) Desarrollo de cataratas. c) Desarrollo de neuropatía óptica anterior isquémica. d) Desarrollo de glaucoma. Cataratas. Cualquier opacidad del cristalino, conlleve o no incapacidad funcional. CATARATA CONGÉNITA. Por alteraciones a partir de la 4ª ó 5ª semana de embarazo. Aparecen al nacer o en los primeros tres meses de vida. Provocan más del 10% de las cegueras en edad escolar. Pueden ser: • Hereditarias (10-25%). Bien aisladas o asociadas a malformaciones oculares o sistémicas. Comportamiento familiar y generalmente bilaterales. • Embriopatías. Por infecciones intrauterinas, sobre todo durante el primer trimestre de embarazo, como rubéola, toxoplasmosis y citomegalovirus. • Metabólicas. - Galactosemia. Por déficit del enzima galactosa 1-P uridil transferasa, que se transforma en galactitol. Este se deposita en el cristalino, provocando una retención de agua que lo opacifica. A medida que el niño va tomando leche van apareciendo cataratas, hepatoesplenomegalia y retraso mental. Reversible, si se diagnostica en las primeras fases de la enfermedad . - Hipoparatiroidismo o pseudohipoparatiroidismo: cuando el Ca2+ es menor de 9,5 mg/100 ml, se puede originar catarata. • Tóxicas. Por clorpromacina, corticoides, hipervitaminosis D. • Carenciales. Por déficit de vitamina A, triptófano, ácido fólico o vitamina B12. • Cromosomopatías. Síndrome de Down (50% tienen cataratas), Turner. CATARATAS SECUNDARIAS. Aparecen en el curso de procesos oculares o sistémicos. Oculares: queratitis, uveítis, tumores coroideos, traumatismos, desprendimiento de retina y glaucoma, entre otras. Sistémicas: • Metabólicas. En diabetes, hipertiroidismo, enfermedad de Wilson, distrofia miotónica de Steinert • Sindermatóticas. En patología dermatológica: poiquilodermia, esclerodermia y eccema atópico. • Tóxicas. Por metales como talio, plata, mercurio, hierro, cobre, o fármacos como corticoides, mióticos, antimitóticos, ergotamina. CATARATA SENIL. Forma más frecuente de catarata y causa más frecuente de pérdida visual reversible en países desarrollados. Son bilaterales, aunque de desarrollo asimétrico. CLÍNICA. Se produce una disminución progresiva de la agudeza visual, sin dolor ni inflamación. Mejora en ambientes poco iluminados o tras instilar un midriático, y empeora en ambientes muy iluminados. En algunos pacientes los síntomas comienzan con una mejoría de la presbicia, por un aumento en el índice de refracción del cristalino. También pueden aparecer fotofobia, visión de halos coloreados y diplopía monocular (debidos a diferencias de refracción entre zonas de la lente). Puede ser nuclear, cortical (por hiperhidratación) o subcapsular posterior. Se denomina incipiente cuando la opacidad y el trastorno visual son mínimos, madura cuando la opacidad es total y el déficit visual severo, e hipermadura, cuando se produce disolución y licuefacción de las fibras, perdiendo su estructura y apareciendo de color gris homogéneo. La cápsula puede presentar pliegues y el núcleo caer hacia abajo.
  • 31. MUY IMPORTANTE En un paciente de edad avanzada que progresivamente desarrolla disminución de la agudeza visual que empeora con luz intensa y mejora de la presbicia, debemos sospechar catarata senil. COMPLICACIONES. Si la catarata no se opera, pueden aparecer las siguientes complicaciones: • Iridociclitis. Por salida de las proteínas del cristalino a través de la cápsula. • Glaucoma secundario agudo o facomórfico. El cristalino capta agua y aumenta de volumen, provocando un aplanamiento de la cámara anterior y un cierre angular con aumento de PIO. • Glaucoma facolítico. Proteínas cristalinianas que pasan a cámara anterior y provocan una obstrucción de la malla trabecular. • Luxación del cristalino. DIAGNÓSTICO. El diagnóstico general de cataratas se hace provocando midriasis farmacológica y observando a simple vista, apareciendo una leucocoria en los estadios avanzados; o bien observando mediante oftalmoscopia directa a unos 30 cm, apreciándose manchas oscuras sobre el reflejo rojo del fondo del ojo, o impidiendo ver este reflejo cuando son maduras; o bien con lámpara de hendidura, que permite ubicar la opacidad dentro del cristalino. TRATAMIENTO. El tratamiento es exclusivamente quirúrgico. La técnica más utilizada se denomina facoemulsificación 27.- Paciente femenino de 45 años que presenta palpitaciones, debilidad, palidez de tegumentos, taquicardia, glositis, parestesias dístales y después ascendentes; y que cursa con ataxia. Bh con Hb de 8; VCM 110 fL; leucocitos y plaquetas discretamente disminuidos. Lo primero que pensaría es: a) Infección crónica. b) Leucemia. c) Hemorragia. d) Deficiencia de cobalamina. Farreras. Medicina interna. Cap 206. Decimoquinta edición 2004, Pp. 1652-1653. Dado que la cobalamina en el trofismo adecuado de la piel y las mucosas y el mantenimiento de la mielinizaciòn adecuada, pueden aparecer alteraciones digestivas y neurológicas, como glositis (de Hunter) y trastornos gastrointestinales inespecíficos (flatulencia, digestiones pesadas), aunque algunos pacientes presentan diarrea que sólo cede con el tratamiento con cobalamina. Las manifestaciones neurológicas se deben a degeneración axonal y desmielinizaciòn de los cordones medulares posteriores (degeneración combinada subaguda). 28.- Femenino de 62 años con diagnóstico de miastenia gravis. De los siguientes medicamentos el que está dirigido al manejo de esta patología es: a) Neostigmina b) Quinidina c) Sumatriptán d) Succinilcolina Manejo farmacológico: Existen diferentes pautas terapéuticas dirigidas a contrarrestar los síntomas de la enfermedad o el mecanismo inmunológico. Los fármacos utilizados son: - Inhibidores de la Acetilcolinesterasa (Neostigmina, Piridostigmina). Dirigidos al manejo sintomático de la MG, mejorando la fuerza motora pero no la progresión de la enfermedad. Su
  • 32. mecanismo de acción es la inhibición reversible de la acetilcolinesterasa, lo cual genera un aumento de ACh en la placa motora. La dosis a utilizar es variable y debe modificarse en distintas etapas de la enfermedad, incluso siendo frecuente no lograr un efecto uniforme en los diferentes grupos musculares en un mismo paciente. El objetivo por lo tanto será utilizar la dosis mínima con la que se genere la mejor respuesta clínica. El efecto se obtiene de 30 minutos a 2 horas de la administración y tiene una duración de hasta 6 horas. Las dosis recomendadas de Piridostigmina son de 15-60 mg cada 4-6 horas vía oral y de Neostigmina 0,5-2 mg/kg cada 4-6 horas intramuscular. Las reacciones adversas asociadas son: dolor abdominal, hipersalivación, aumento de las secreciones respiratorias y bradicardia y se relacionan con el efecto colinérgico generado, por lo que es necesario administrar concomitantemente atropina2,3. Ponsetia JM: Miastenia Gravis. Manual Terapéutico. Barcelona; Springer Verlag Ibérica, 1995 2.- Ponsetia JM, Espina E, Armengola M: Diagnóstico y Tratamiento de la Miastenia grave. Med Clin (Barc) 2000; 115: 264-70. 3.- Drachman DB: Myasthenia gravis. N Engl J Med 1994; 330: 1797-810 4.- Andrews PI: Autoimmune myasthenia gravis in childhood. Semin Neurol 2004; 24: 101-10 5. - Anlar B: Juvenile myasthenia: diagnosis and treatment. Paediatr Drugs 2000; 2: 161-9. 6. - Gajdos P: Myasthenic syndrome. Diagnosis trends. Rev Prat 2000; 50: 419-23 7. - 29.- Femenino de 34 años la cual labora en lavandería, hace varios años presenta enrojecimiento de los pliegues proximales de las uñas de varios dedos de las manos, que ocasionalmente le supuran. El diagnóstico más probable de la paciente es: a) Dermatitis de contacto. b) Paroniquia candidiásica crónica. c) Liquen plano. d) Dermatoficia. Infección por Candida del lecho ungueal que se presenta con mayor frecuencia como una onicolisis asociada con paroniquia, aunque también se observa la destrucción completa del lecho ungueal y la erosión de la zona distal y lateral de las uñas de los dedos, sin distrofia ungueal total. La perionixis candidiásica se trata de la inflamación con eritema, edema, dolor y salida ocasional de pus blanquecino escaso y espeso del reborde ungueal de uno o varios dedos de la mano. Con frecuencia, se acompaña de onicopatía (discoloración, onicolisis, distrofia total, etc.). Predisponentes: Todas las labores manuales que generen humedad. (muy frecuente en amas de casas, trabajadores de restaurantes, lavanderas, etc.)
  • 33. BIBLIOGRAFÍA: 1. Crespo Erchiga V. Protocolo diagnóstico de contaminantes. En "Micología para dermatólogos" Ed. Janssen, Madrid, 1994, pp:49-70. 2. Crespo V, De Luís B, Delgado V, Crespo A y Vera. Espectro etiológico de las onicomicosis en nuestro medio. CO7. II Congreso Nacional de Micología. Santiago de Compostela. Junio, 1994. 3. Crespo Erchiga V, Delgado Florencio V y Martínez García S. Micología dermatológica. Ed. M.R.A. Barcelona, 2006. 4. Daniel III CR. The Diagnosis of Nail Fungal Infection. Editorial Arch Dermatol 1991;127:1566- 1567. 5. Delgado Florencio V. Protocolo de identificación de dermatofitos. En "Micología para dermatólogos". Ed. Janssen, Madrid, 1994, pp:27-41. 6. Delgado Florencio V. Estrategia en el diagnóstico y tratamiento de las micosis superficiales. Ed. Aula Médica, Madrid, 1994. 7. Delgado V, Abad Romero-Balmas J, Armijo Moreno M y Dulanto F. Scopulariopsis brevicaulis como agente de onicomicosis. Actas Deermo-Sif. 1976; 9-10:693-700. 8. English MP. Nails and fungi. Br J Dermatol 1976; 94:697-701. 9. Fevilhade de Chauvin M. Onicomicosis. Dermatología práctica. 1994; 9:1-2. 30.- Femenino que desde hace varias semanas le han aparecido de forma eruptiva unas máculas y placas eritematosas en el tronco, refiere que hubo una lesión más grande que precedió a las demás. Las lesiones presentan una descamación fina en la periferia y son dis- cretamente pruriginosas. No existe afectación palmoplantar. La serología luética es negativa. EL tipo de pitiriasis más probable ES: a) P. liquenoide crónica. b) P. rubra pilaris. c) P. rosada. d) P. versicolor La pitiriasis rosada es una enfermedad exantemática autolimitada de causa desconocida que cursa con lesiones maculopapulosas de aspecto asalmonado en áreas proximales de las extremidades y en tronco, donde se distribuyen de forma paralela a las líneas de la hendidura (Stulberg DL, 2004).
  • 34. Existen algunas características que sugieren una etiología vírica (pródromos, predominio estacional, evolución variable, ausencia de recaídas, hallazgos histológicos...) que algunos autores relacionan con el herpes virus humano 7, aunque otros lo desmienten (Chuh A, 2004). Afecta con más frecuencia a niños mayores y jóvenes adultos (10-35 años) y es ligeramente más frecuente en mujeres (Chuang TY, 1982). El cuadro típico se inicia con una lesión redondeada u oval, de 4-8 cm. única, generalmente situada en el tronco, de color asalmonado, discretamente descamativa y en ocasiones ligeramente pruriginosa, llamada “madre”. Puede acompañarse de síntomas generales leves: astenia, anorexia, odinofagia, cefalea y artralgias. En 1-2 semanas esta lesión se torna mas escamosa y clara en el centro al tiempo que aparecen otras de menor tamaño que se diseminan por las extremidades (áreas proximales) y por el tronco distribuyéndose simétricamente en forma arbórea a lo largo de las hendiduras costales. En la espalda es más evidente (“signo del árbol de Navidad”) (Stulberg DL, 2004) La evolución de las lesiones es similar a las de la lesión “madre” desapareciendo en 5-6 semanas, aunque en algunos casos puede permanecer más tiempo y en otros verse áreas hiper o hipo pigmentadas durante algún tiempo. La tasa de recurrencia a los 5 años es inferior al 2% (Chuang TY, 1982). Pitiriasis Rosada Chuh A, Chan H, Zawar V. Pityriasis rosea--evidence for and against an infectious aetiology. Epidemiol Infect. 2004 Jun;132(3):381-90. Chuh AA, Chan HH. Prospective case-control study of chlamydia, legionella and mycoplasma infections in patients with pityriasis rosea. Chuh AA, Dofitas BL, Comisel GG, Reveiz L, Sharma V, Garner SE, Chu F. Interventions for pityriasis rosea. Cochrane Database Syst Rev. 2007 Apr 18;(2):CD005068. Chuh AA. Quality of life in children with pityriasis rosea: a prospective case control study. Pediatr Dermatol. 2003 Nov-Dec;20(6):474-8. Hsu S, Le EH, Khoshevis MR. Differential diagnosis of annular lesions. Am Fam Physician. 2001 Jul 15;64(2):289-96] Eur J Dermatol. 2002 Mar-Apr;12(2):170-3. Miranda SB, Lupi O, Lucas E. Vesicular pityriasis rosea: response to erythromycin treatment. J Eur Acad Dermatol Venereol. 2004 Sep;18(5):622-5
  • 35. 31.- Femenino de 39 años, acude a consulta refiriendo mareo a la bipedestación. Antecedentes: DM2 de 10 años de evolución bajo tratamiento médico. Exploración física: TA 105/60mmHg sentada y de pie 95/60mmHg, FC 76 x´, FR 16 x´. Usted solicita ES. Reportando: Na 164 mEq/l, osmolaridad urinaria 210 mOsm/l, osmolaridad sérica 330 mOsm/l, sodio urinario 45 mOsm /l. El diagnóstico de primera elección en este paciente es: a) Diabetes juvenil b) Diabetes insípida nefrogénica. c) Diabetes insípida central. d) Diabetes periférica Diabetes insípida nefrogénica: Se caracteriza por la falta de respuesta tubular a la AVP, en presencia de concentraciones normales de la misma. Causas: alteraciones del receptor V2, uso de litio, hipercalcemia, hipopotasemia, enfermedades renales que interfieren en la concentración de orina. Consecuencias: Poliuria hipoosmótica (>40ml/Kg./d) (<200 mOsm/Kg.), tendencia a la deshidratación hipertónica (285-290mOsm/Kg.), con sodio de 140-145mEq/l, exceso de sed en personas con conciencia conservada, la ingesta de sodio es igual a la secreción Hernando L, Nefrología Clínica, Ed. Panamericana, 3ªEdición, 2008, Pág. 42-43 32.- Mujer de 37 años. Acude a consulta por presentar cefalea, cansancio e irregularidades menstruales con ritmo de 36 a 50 x 2-3 días. No se ha podido embarazar después de 18 meses de actividad sexual regular. No tiene antecedentes importantes. EF: Campos visuales normales, tiroides aumentada de tamaño una vez y aumentada de consistencia, no tiene galactorrea. Resto normal. Laboratorio: química sanguínea, Bh y electrolitos normales. Prolactina 47 ng/dL (< 25), perfil tiroideo: TSH 18 mUI/ml, T4t: 50 nmol/L (57.9 a 154.4), T4L: 7.7 pmol/L (9 a 24), T3T: 1.06 nmol/L (1.2 a 2.9), T3L: 1.96 pmol/L (3 a 6.31) La etiología más frecuente de este problema es: a) Tiroiditis autoinmune crónica b) Microadenoma hipofisario c) Resistencia a la insulina d) Enfermedad de Graves Inicialmente el hipotiroidismo se diagnosticaba mediante la cuantificación por técnicas de Radio Inmuno Análisis (RIA) de las hormonas circulantes triyodotironina y tiroxina; el proceso era lento y sometido a muchos factores de error que hacían su sensibilidad y especificidad poco confiables. Posteriormente, se desarrollaron técnicas para la medición de la TSH hipofisiaria igualmente mediante el RIA lo que mejoró en forma importante la sensibilidad para el diagnóstico de esta enfermedad; sin embargo, los niveles de detección de la prueba se encontraban en el orden de 1 µIU/ml lo que hacía que la prueba no fuera sensible para valores menores de 1 µIU/ml. Debido a esto se crearon técnicas de segunda generación mediante la cuantificación de TSH por anticuerpos monoclonales y RIA, el IRMA (Immuno Radiometric with Monoclonal Antibodies) que permitió detectar valores de TSH en rangos de 0.1 µIU/ml; posibilitando desde entonces diagnosticar pacientes con hipertiroidismo primario; pero con la limitante de que para esta técnica era imposible detectar valores de TSH menores de 0.1 µIU/ml por lo que se creo la medición de TSH mediante quimioluminiscencia o métodos enzimáticos, es decir las técnicas de tercera generación, las cuales pueden detectar valores de TSH de 0.01 µIU/ml; con lo que se logra el espectro ideal para una prueba de laboratorio que tiene la capacidad de diagnosticar tanto la hipofunción como la hiperfunción(20).
  • 36. Además el avance no sólo fue en la medición de TSH sino también en las hormonas tiroideas que han evolucionado simultáneamente con la TSH y ya se miden incluso las fracciones libres de hormonas y las fracciones totales, lo que ha facilitado el manejo de estos pacientes. Gracias a esta evolución en técnicas de laboratorio, el diagnóstico de hipotiroidismo primario es bastante sencillo. Niveles de TSH superiores al valor máximo de la técnica serían diagnósticos de la disfunción; pero no es tan fácil. Cuando tenemos un paciente con toda la sintomatología del hipotiroidismo y la TSH se encuentra elevada el diagnóstico es obvio; pero podemos tener pacientes con síntomas muy inespecíficos como depresión y con examen físico normal a quienes se les encuentran valores de TSH por encima del límite superior y con hormonas tiroideas normales. Se trata de un hipotiroidismo o es un valor ligeramente elevado ocasional de una persona sana(21). Igualmente tenemos otra circunstancia que ha sido descrita con mayor frecuencia: pacientes con valores de TSH en el límite superior normal y con dislipidemia a quienes se les da tratamiento con hormonas tiroideas y su dislipidemia se corrige manteniendo valores de TSH en rangos normales. Todas las circunstancias anteriores han hecho que aparezca en el hipotiroidismo primario la expresión de hipotiroidismo subclinico, que ha sido objeto de reuniones y congresos dedicados exclusivamente a este tema. La sociedad Europea de Tiroides hace algunas recomendaciones para el manejo de esta situación que se consideran útiles como guía (Tabla ). Tabla. Enfoque del paciente con disfunción tiroidea de acuerdo a los niveles de hormona estimulante de la tiroides (TSH). Si TSH < 0.4m U/L Si TSH 0.4 a 2.0 mU/L Si TSH 2.01 a 5.0mU/L Si TSH > 5.0 mU/L Medir T3 y T4 totales o libres para diagnostico de hipertiroidismo. Normal, Repetir cada cinco años Medir T4 libre y anticuerpos antitiroideos Dar tratamiento para hipotiroidismo 1. Si AAT (-) y T4 libre es normal repetir screening cada año. Si TSH es > 4.0mU/l en dos ocasiones dar tratamiento 2. Si AAT (+) y/o T4 libre esta baja o normal baja tratar si TSH es mayor de 3.0 mU/l y observar a los otros Tomado de Koutras DA. Subclinical hypothyroidism. En G. Hennemann, E.P. Krenning, Thyroid International Merck KGaA, Darmstadt 1999 (3), 6-9
  • 37. HIPOTIROIDISMO PRIMARIO: congénito: agenesia autoinmune postablativo tiroiditis subaguda deficiencia de yodo SECUNDARIO tumor hipofisiario iatrogénico TERCIARIO 33.- Se trata de masculino de 11 años de edad sin antecedentes de importancia para el padecimiento actual. Inicia hace 14 días con rinorrea hialina, tos en accesos, adinofagia, cefalea y febrícula. Es atendida por médico quién administra manejo sintomático con discreta mejoría. Hace 5 días inicia con dolor dental,la rinorrea se torna mucopurulenta con incremento de eventos de tos de predominio nocturno. A la exploración presenta faringe hiperémica con descarga posterior, halitosis, narinas con moco purulento. El diagnóstico más probable es: a) Faringoamigdalitis bacteriana b) Sinusitis aguda c) Difteria d) Faringoamigdalitis viral SINUSITIS. Las inflamaciones de los senos paranasales constituyen una afección frecuente, aunque generalmente banal, y con frecuencia son una extensión de la patología de las fosas nasales. Aproximadamente un 5 % de la población padece en algún momento una sinusitis crónica. El seno más afecto es el maxilar, seguido del etmoides, frontal y esfenoidal: Durante el periodo pediátrico los senos más afectados son los etmoidales. CLASIFICACIÓN: - Sinusitis aguda: Duración menor a 4 semanas - Sinusitis subaguda: Duración: 4 a 12 semanas